届高三语文试卷 (菁华3篇)

首页 / 语文 / | 2022-11-12 00:00:00

届高三语文试卷1

  一、现代文阅读

  阅读下面的文字,完成下列小题。

  英国*日前宣布拟推出新规,要求无人机操作员必须“实名注册”,还要参加类似驾驶理论考试的安全意识测试,试图借问责制从源头强化管理,规范无人机使用。

  被喻为“上帝之眼”的无人机不仅在军事、工业勘测、商业配送等领域功用丰富,在民用领域也越来越受到青睐。据统计,20xx年全球无人机的产量将接*300万架,比20xx年增长39%,市值超过60亿美元。然而在技术和市场迅速发展的同时,安全隐忧也在不断增加。

  上个月,一架无人机出现在伦敦盖特威克机场跑道上,导致数十架飞机无法降落,机场也短暂关闭。从无人机面世至今,全球各地不断有其干扰民航客机,造成后者延误、迫降甚至返航等情况。同时,无人机可能带来的对私人领域的侵犯、个人隐私的窥视也引发担忧。各国*因此不断规划实施对无人机的管理,不过,宽严幅度则相距甚远。

  在法国,一名少年因使用无人机拍摄城市全景即被起诉,理由是可能造成对他人的危害。根据法国目前颁发的法规,除机场、军事区等敏感地区禁用民用无人机,城市及周边地区,包括公路、公园、沙滩等公共场所上空同样禁用无人机。即使是私人区域,未经允许也不得随意对他人及其所有物进行拍摄,更不得以商业目的进行传播。在澳大利亚、新西兰,类似严格的举措也在施行。

  相比之下,爱尔兰等国家的管理措施相对宽松,爱尔兰仅要求重量在1000克以上的无人机进行登记。英国交通部的研究也显示,400克以上的无人机就有可能对商用飞机的驾驶舱玻璃造成损伤。

  毋庸置疑,维护公共及私人安全领域不受市场盲目逐利的侵害,乃是监管的第一大功能。但是,采取合适的力度使之不损害行业发展的前景、不打击研发的积极性也是判断监管是否适当的重要标准,像英国运输部官员马丁·卡拉南所指出的,“要试图通过无人机新规优先保护公众,同时最大程度发挥无人机的潜力”。去年底,在美国的监管严规拒绝亚马逊公司测试无人机送货之后,英国作为“全新的自动化技术方面全球最为积极的国家之一”,欣然接受了亚马逊的测试请求,而且亚马逊也只是在英国测试类似产品的“众多大公司之一”。

  以互联网为代表的新兴产业在发展中新的业态不断涌现,特别是以移动互联网、物联网、云计算大数据为代表的新一代信息技术还带来商业模式、服务模式的更新与比拼。对新生事物的发展要有更多的包容,更多的鼓励和支持,而有效的监管一方面应该成为引导规范新兴行业发展的利器,另一方面则应成为帮助*衡其商业潜力与人们安全需求之间的天*。

  完善的监管方案本身也是一种宝贵的“发明”,其价值终会在时间的检验中得以呈现。

  (摘编自李应齐《给无人机立规矩》)

  1.下列关于原文内容的理解和分析,不正确的一项是

  A. 英国*试图从源头强化管理,规范无人机的使用,拟要求无人机操作员“实名注册”。

  B. 无人机被喻为“上帝之眼”,*几年来,技术和市场迅速发展,安全隐忧也在不断增加。

  C. 各国*在不断规划实施对无人机的管理,不过,美洲和欧洲的宽严幅度相距甚远。

  D. 有效的监管方案既要引导规范新兴行业发展,又要*衡它的商业潜力与人们的安全需求。

  2.下列对原文论证的相关分析,不正确的一项是

  A. 文章从英国拟推出新规入手,引出无人机快速发展所带来的安全隐忧问题。

  B. 文章用较多篇幅列举了无人机的安全隐忧,引出各国加强对其监管的问题。

  C. 文章以维护公私安全领域不受侵害为市论前提,阐发了严管无人机的主旨。

  D. 文章引马丁·卡拉南的话,说明监管无人机既要考虑安全,又要有益其发展。

  3.根据原文内容,下列说法不正确的一项是

  A. 要求无人机操作员必须实名注册,就能从源头上对其强化管理,规范其使用。

  B. 盖特威克机场飞机无法降落,是因为跑道上出现的无人机可能对其造成损伤。

  C. 法国禁止在公路、公园等公共场所上空使用无人机,因其可能危害他人权益。

  D. 在新一代信息技术带来商业、服务模式更新背景下,实现有效监管尤为重要。

  阅读下面的文字,完成下列小题。

  车马炮

  丁肃清

  德欣说:“你这匹死马,我暂不吃你。”他说着攻小卒,又攻,再攻……残局上的小卒已是兵临城下。德欣洋洋得意地看着红棋憋在仕脚里的那匹马,确是瓮中之鳖,自己的老将一崴就吃了,可他不吃,他要叫对方输得心服口服。

  杨喜良就剩下车马炮了,而且那马还是匹死马,这使得他捉襟见肘,但仍耐心寻找着机会。

  德欣是车务段段长,杨喜良是老调度,两人私交甚密。他们什么话都能说,一边下棋一边说着车务段的事情。

  旁边还有一位观棋者,叫李来顺,李来顺观棋不语。

  几十天的春运,搞得焦头烂额,难得老朋友一聚。杨喜良说:“你看咱仨,像是这棋,你段长是车,我是这炮,都还厉害,来顺像这匹马。”

  德欣和杨喜良都笑了。李来顺不笑,只静静观棋,在车务段,三十年前是个兵,三十年后还是个兵,尽管他书法绘画,弹拉说唱无所不能,也卖力气,却没有提升,真的就像憋在仕脚的那匹马。可李来顺没有怨言。

  德欣段长说:“稍停,我去撒尿。”

  德欣撒尿去了。

  杨喜良说:“德欣的棋厉害。”

  李来顺说:“我不尿他。”

  杨喜良说:“你吃亏就吃在了这个脾气。”

  论关系李来顺和德欣段长不错,之所以不错,才没有客套,有人没人,不喊段长,就叫他德欣,他想,这才是哥们儿。

  可当了领导,谁没有个自尊心啊。杨喜良说:“德欣工作还是有水*的。”

  李来顺又说:“我不尿他。”

  德欣撒尿回来了,继续下棋。

  李来顺说:“我也去撒尿。”

  趁李来顺撒尿的工夫,杨喜良对德欣说:“咱们段里,数来顺最冤,什么苦活儿累活儿都有他,你可该提拔提拔他。”

  德欣说:“我快退了,我不想管。”

  说得杨喜良不高兴,“啪”地一声敲得很响,“将!”杨喜良在车的后面安上了炮。德欣的将想躲,却躲不了,往哪儿躲都在那只死马的脚下。

  德欣“哎呀”了一声,没想到这盘棋输到了这匹死马上。

  李来顺回来了,大喊:“输得好!我早就看出你要输这一招儿。”

  棋散,仨人叙话。李来顺说:“咱仨,你们都混得好,是车,是炮,我是这死马,可死马也有置之死地而后生的时候。”

  德欣略有感慨:“来顺啊,我对不起哥儿们,可也怪,咱仨人怎么就这么好呢?”

  李来顺说:“咳!这怪啥呀?都吃过苦,你不记得你当司机,我和杨喜良当司炉,从阳泉跑石家庄,四个钟头要填十二吨的碳,那累呀,我和杨喜良都不想活了,是不是呀喜良?”

  杨喜良说:“是,苦是咱们的根本。”

  德欣似有感悟:“这样吧喜良,趁我还没有退,是该把来顺提拔一下,也算了却心事一桩。”

  李来顺“哈哈”地大笑起来:“错了,错了!谁稀罕你那个提拔,倒找我钱我也不干。”

  德欣瞪大了眼睛:“此话怎讲?”

  李来顺说:“这么讲吧,俺李来顺是神仙日子,每天五点起床画画,然后跑步,再然后看日出,看完日出早饭喝一两小酒儿。”

  “咳呀!”杨喜良似悟道:“你李来顺让我想起一句话,袖中乾坤大,壶中日月长,神仙、神仙!”

  德欣无话,又来了下棋的雅兴,“再来一盘!”他愣拽着李来顺下棋,结果连输三盘。李来顺不下了,说德欣段长臭棋篓子。德欣还不甘心。

  李来顺说:“要下,就此一盘了,我让你车马炮。”

  德欣段长犹豫了一下说:“好吧。”

  棋子儿的敲击声又响了起来……

  1.下列对小说相关内容和艺术特色的分析鉴赏,不正确的一项是

  A. 小说讲了车务段三个人下象棋的故事,下棋过程情节起伏,引人入胜,棋局的输赢始终作为主要悬念吸引着读者阅读。

  B. 车务段段长德欣虚荣、狭隘,棋艺最差而不自知,杨喜良比李来顺处世更谨慎,懂得在让棋的过程中迎合领导的感受。

  C. 小说作者擅长运用语言描写推进情节发展,刻画人物形象。下棋者、观棋者的对话虽然简单,却符合不同人物的身份与性格。

  D. 段长与调度在是否提拔李来顺的问题上起初态度不同,但他们后来发现后者生活悠闲、*静,有着洒脱淡泊的人生追求。

  2.小说在刻画李来顺这个形象时,突出了他的哪些形象特征?请简要分析。

  3.小说为什么以“车马炮”为标题?请结合作品进行分析。

  阅读下列材料,完成下列小题。

  材料一:

  “天舟一号”货运飞船于20xx年4月20日19时41分在海南文昌航天发射场由长征七号遥二运载火箭发射升空,“天舟一号”发射获得圆满成功。

  “天舟一号”货运飞船,是由*空间技术研究院(*航天科技集团五院)研制的一款货运飞船,也是*首个货运飞船。“天舟一号”具有与“天宫二号”空间实验室交会对接、实施推进剂在轨补加、开展空间科学实验和技术实验等功能。

  货运系统是*建成空间站需要突破和掌握的关键技术,“天舟一号”将使*具备向在轨运行航天器补给物资、补加推进剂的能力,这一能力,是确保未来*空间站在轨长期载人飞行的基本前提。

  (摘编自《人民日报》20xx年4月21日)

  材料二

  “神舟十号”任务完成后,*载人航天工程全面进入空间实验室和空间站研制阶段。而在空间实验室阶段,将突破并验证推进剂补加技术、再生式环控生保技术等关键技术,为空间站建造奠定基础。

  20xx年9月15日,“天宫二号”发射成功,用于进一步验证空间交会对接技术及一系列空间试验,这标志着我国全面进入空间实验室和空间站任务实拖阶段。

  为了对空间实验室中航天员长期驻留和空间科学实验进行支持,要通过货运飞船进行货物补给。如果说载人飞船是天地往返的载人工具,那么货运飞船就是天地间运货的工具。*的“天舟一号”货运飞船基于神舟飞船和“天宫一号”的技术研发,只运货不运人,货物运载量将是俄罗斯进步号无人货运飞船的3倍,在功能、性能上都处于国际先进水*。

  “天舟一号”货运飞船旨在补给空间实验室以及未来*空间站的推进剂、空气、航天员的饮料食物以及用于维修空间站的更换设备,以延长空间实验室和空间站的运行寿命。

  (摘编自《*》)

  材料三

  “天舟一号”货运飞船由3.35米直径的货物舱和2.8米直径的推进舱组成。货物舱用于装载货物,而推进舱为整个飞船提供动力与电力,推进舱两倒各有一翼太阳能帆板(共三板),后部安装了4台变轨用主发动机。此外飞船还安装了24台姿控发动机。

  “天舟一号”货运飞船全长10.6米,最大直径3.35米,质量13吨,最大上行货物运载量约6.5吨。无论是直径、质量还是运载能力,“天舟一号”都将达到世界先进水*。

  “天舟一号”货运飞船的*地轨道上行运载能力约为6.5吨,高于俄罗斯联邦航天局研制的进步号M型(2.5吨)以及日本航空研究研发机构的H -Ⅱ运载飞船(6.0吨),低于欧洲空间局的自动运载飞船(7.6吨)。下行运载能力约为6.0吨。载荷比即运载货物的质量与货运飞船船体本身的质量之比,“天舟一号”货运飞船的载荷比高达48%,高于日欧的货运飞船。

  (摘编自《*航天报》)

  材料四

  20xx年7月2日晚,长征五号遥二火箭飞行出现异常,发射任务失利。消息一经披露,人们不禁扼腕痛惜。

  航天试验从来没有*坦大道,总是与高风险为伴,可谓“刀尖上的舞蹈”。每每遇到瓶颈和挫折,航天人都承受着常人难以想象的巨大压力,有时甚至会为航天事业献出宝贵的生命。

  惟其艰难,方显勇毅;惟其磨砺,始得玉成。航天探索是勇敢者的事业,只有不惧失败、跌倒后重新爬起、继续砥砺前行的人,才能摘得世界科技高峰上的明珠。

  (摘编自《*航天报》评论员文章《惟其艰难方显勇毅》)

  1.下列对材料相关内容的梳理,不正确的一项是( )

  A. “天宫二号”发射成功,标志着我周全面进入空间实验室和空间站任务的实施阶段。

  B. “天舟一号”货运飞船基于神舟飞船和“天官一号”的技术研发,足*首个货运飞船。

  C. “天舟一号”货运飞船由用于装载货物的货物舱和为整个飞船提供动力与电力的推进舱两部分组成。

  D. “天舟一号”货运飞船只运货不运人,因为货运系统是*建成空间站需要突破和掌握的关键技术。

  2.下列对材料相关内容的概括和分析,不正确的两项是

  A. 航天足高风险事业,航天人承受着巨大压力,甚至献卅宝贵的生命。在失败中砥砺前行,方可登顶世界科技之巅。

  B. 运载“天舟一号”飞船的长征七号遥二火箭今年4月20日发射成功,长征五号遥二火箭今年7月2日发射失利。

  C. 推进剂补加技术等是*载人航天工程全面进人空间实验室和空间站研制阶段需要突破并验证的关键技术。

  D. 如果能及时补给推进剂、空气以及用于维修空间站的更换设备,就能延长空间实验室和空间站的运行寿命。

  E. “天舟一号”的*地轨道上行运载能力介于日本H-Ⅱ运载飞船与俄罗斯联邦航天局进步号M型运载飞船之间。

  3.根据上述材料,概括说明“天舟一号”的主要任务及现实意义。

  二、文言文阅读

  阅读下面的文言文,完成下列小题。

  郑冲,字文和,荥阳开封人也。起自寒微卓尔立操清恬寡欲耽玩经史遂博究儒术及百家之言有姿望动必循礼任真自守不要乡曲之誉由是州郡久不加礼。 及魏文帝为太子,搜扬侧陋①,命冲为文学,累迁尚书郎,出补陈留太守。冲以儒雅为德,莅职无干局之誉,箪食缊袍,不营资产,世以此重之。大将军曹爽引为从事中郎,转散骑常侍、光禄勋。嘉*三年,拜司空。及高贵乡公讲《尚书》,冲执经亲授,与侍中郑小同俱被赏赐。俄转司徒。常道乡公即位,拜太保,位在三司之上,封寿光侯。冲虽位阶台辅,而不预世事。时文帝辅政,*蜀之后,命贾充、羊祜等分定礼仪、律令,皆先谘于冲,然后施行。及魏帝告禅,使冲奉策。武帝践祚,拜太傅,进爵为公。顷之,司隶李熹、中丞侯史光奏冲及何曾、荀顗等各以疾病,俱应免官。帝不许。冲遂不视事,表乞骸骨。优诏不许,遣使申喻。冲固辞,上貂蝉印绶,诏又不许。泰始六年,诏曰:“昔汉祖以知人善任,克*宇宙,推述勋劳,归美三俊。遂与功臣剖符作誓,藏之宗庙,副在有司,所以明德庸②勋,藩翼王室者也。昔我祖考,遭世多难,揽授英俊,与之断金,遂济时务,克定大业。太傅寿光公郑冲、太保郎陵公何曾、太尉临淮公荀顗各尚德依仁,明允笃诚,翼亮*,光济帝业。故司空博陵元公王沈、卫将军钜*侯羊祜才兼文武,忠肃居正,朕甚嘉之。”九年,冲又抗表致仕。明年薨。帝于朝堂发哀,追赠太傅,赐秘器,朝服,衣一袭,钱三十万,布百匹。谥曰成。

  (节选自《晋书·郑冲传》)

  【注】①搜:寻找。扬:选拔、举荐。侧:同“仄”。侧陋:隐伏,卑微,引申指地位低微。②庸:古同“佣”,雇佣,任用。

  1.下列对文中画波浪线部分的断句,正确的一项是

  A. 起自寒微/卓尔立操/清恬寡欲/耽玩经史/遂博究儒术及百家之言/有姿望动/必循礼任真自守/不要乡曲之誉/由是州郡久不加礼/

  B. 起自寒微/卓尔立操/清恬寡欲耽玩/经史遂博/究儒术及百家之言/有姿望/动必循礼/任真自守/不要乡曲之誉/由是州郡久不加礼/

  C. 起自寒微/卓尔立操/清恬寡欲耽玩/经史遂博/究儒术及百家之言/有姿望动/必循礼任真自守/不要乡曲之誉/由是州郡久不加礼/

  D. 起自寒微/卓尔立操/清恬寡欲/耽玩经史/遂博究儒术及百家之言/有姿望/动必循礼/任真自守/不要乡曲之誉/由是州郡久不加礼/

  2.下列对文中加点词语的相关内容的解说,不正确的一项是

  A. 出,指从京城调出,到地方为官。文中郑冲由京官尚书郎补任地方官陈留太守。

  B. 朕,我,我的。秦以前,不论尊卑,皆自称朕;后自秦始皇起,用为皇帝自称。

  C. 致仕,指古代官员自请加职或升职。文中郑冲致仕是想为国家担负更重的责任。

  D. 秘器,指棺木。郑冲去世时,武帝为了表示对郑冲的恩宠,赐给他棺木以安葬。

  3.下列对原文有关内容的概括和分析,不正确的一项是

  A. 郑冲注重修养,作风朴素。任陈留太守时,他把儒雅作为自己追求的道德境界,任职不求美名,吃蔬食,穿麻絮袍子,不营私产,世人因此看重他。

  B. 郑冲洁身自好,受人尊重。做太保时,级别虽已和宰相相同,但他从来不参与世俗的事务;贾充、羊祜等人制定礼仪、法律制度,都是先求教于他。

  C. 郑冲是武帝的心腹之臣。在李熹、史光等人以疾病为由奏请免去郑冲等人的官职时,武帝依然对郑冲坚信不疑,并一次次地驳回他的辞职申请。

  D. 郑冲受到皇帝的称赞。皇帝通过赞美汉高祖刘邦的知人善任,回顾祖辈君臣的通力协作,进而夸赞了郑冲等人的勋劳,并间接表达了赏识之意。

  4.把文中画横线的句子翻译成现代汉语。

  (1)及高贵乡公讲《尚书》,冲执经亲授,与侍中郑小同俱被赏赐。

  (2)昔我祖考,遭世多难,揽授英俊,与之断金,遂济时务,克定大业。

  三、诗歌鉴赏

  阅读下面这首宋词,完成文后各题。

  【中吕】朝天子 卞和①

  薛昂夫

  卞和,抱璞,只合荆山坐。三朝不遇待如何,两足先遭祸。

  传国争符,伤身行货,谁教献与他?切磋,琢磨,何似偷敲破?

  【注】①楚人卞和在荆山发现了一块玉石,献给楚厉王,厉王以为是石头,砍掉了他的左足;武王即位,他又去献,结果又被砍掉右足;文王时,他抱着这块玉石在荆山下痛哭,文王知道后,叫人剖开石头,果然得到宝玉,就命名为和氏璧。

  1.下列对这首元曲的赏析,不恰当的两项是( )

  A. 这支曲子上片运用了和氏璧的典故,点明卞和因献璞玉而身遭不幸,在荆山下痛哭。

  B. “传国争符”是说秦王将和氏璧刻为传国玉玺,导致后来许多野心家为此争战不休。

  C. “行货”,真的东西;“伤身行货”,是随此璧虽是真的,但使卞和伤身,是不祥之物。

  D. 结句是作者的愤激之词:与其让他们拿着璞玉去切磋琢磨,还不如偷偷地将它敲碎。

  E. 这首元曲先是介绍事件,然后揭示事件的惨痛教训,最后直抒胸臆,表达自己看法。

  2.卞和献璞的故事一向被看作才士不遇的悲剧,卞和一向为人们深切同情,这支曲子却有不同的看法,请简要分析。

  四、名句名篇

  补写出下列句子中的空缺部分。

  (1)以乡愁为主题的诗词中,多以明月寄托愁思,李白在《闻王昌龄左迁龙标遥有此寄》中有这样两句:“______,______。”

  (2)荀子在《劝学》中用行千里路作比喻表达成事需善于借助外力的语句是“______,______,______”。

  五、选择题

  下列各句中加点成语的使用,全都不正确的一项是

  ①1954年出生的清河镇木版年画第21代传人王圣亮,脸庞瘦削,头发稀疏,与当地农民别无二致,但每当和记者谈到木版年画时,他就显得很健谈。

  ②出游之前,看看百度热力图便能知道哪里人满为患、哪里万人空巷,避免仅在看过旅行攻略后就出发、却发现看海变成看人海的尴尬。

  ③他多次跑省、县有关部门,上下其手,发动单位力量,筹措项目资金,在他的带领下,全村立即启动了16公里村道延伸建设工程。

  ④我国对文化的传承态度不应是胶柱鼓瑟,而应是尊重文化传统,注重社会实际,注重变通、创新、开放、多元,注重在基础上的传承。

  ⑤鲁迅先生对于友人尤其是青年的爱护可以说是无所不为,这一点在后来的很多回忆文章中都谈到了,这也是先生身上最为可贵的品质之一。

  ⑥于是,以培养孩子某些特殊能力为目的的各种特长教育应运而生,如书法班、美术班、舞蹈班、音乐班等等,应有尽有,不一而足。

  A. ①③⑥ B. ①④⑥ C. ②③⑤ D. ②④⑥

  下列各句中,表达得体的一句是

  A. 如今人的“网行”和文的“网行”都是“无人驾驶”般潜行,心下感慨不知不觉换了人间。鄙人虽廉颇老矣,好在尚能饭,弄微信不亦乐乎。

  B. 春阳和煦,浙江省杭州市西湖区的有机荼基地,头茶终于开摘了。一年尝鲜就在这几天,当我赶到时,相约的老茶友们已经恭候我多时了。

  C. 公司经理给开发部门开会时说:“我们要借重自己在市场上关于投资的专业知识,来发现不同领域的投资方向,以达到公司多元化经营的目的。”

  D. 热浪翻滚的酷暑天气,朋友特意做了拿手菜“凤鹅烧冬瓜汤”招待我们,并不断嘱咐:“你家小女最爱吃这道菜了,一定要带孩子来呀!”

  六、语言应用

  在下面一段文字横线处补写恰当的语句,使整段文字语意完整连贯,内容贴切,逻辑严密,每处不超过15个字。

  爱因斯坦最推崇西方古典音乐,人们常常看到这样一个动人的情景:_________。弹钢琴者是量子论的创始人普朗克,演奏小提琴的,_________。普朗克的量子论和爱因斯坦的相对论共同构成了本世纪物理学科的两大支柱,在科学上面,他们共同描绘了物理学的一幅优美壮丽的图景,在音乐艺术中,_________。

  仿照示例,感受和理解下面两个面面形象的内在涵义,请为它们分别设汁面外音一要求中心明确,至少运用一种修辞下法。

  示例:

  画面:一个女孩在“有需要可自取”的零钱箱前驻足,把自己手中的硬币放了进去。

  画外音:你相信良善,提供便利;我心怀感恩,传递爱心。

  面面一:一个行动缓慢的老人脱下帽子,向停车礼让他的司机深深鞠躬致谢。

  面外音一:_____________________________________________________________。

  面面二:四川九寨沟地震中,前方落石滚滚,一个身着迷彩服的士兵与人们逆向奔跑,跑向救灾前线。

  面外音二:_____________________________________________________________。

  七、材料作文

  阅读下面的材料,根据要求写作。

  7月11日,在第十届海外高层次人才座谈会上,87岁的袁隆*以流利的英语完成了一次长达20分钟的演讲。他卓越的科技贡献和精彩的演讲内容获得了现场观众数次热烈的掌声。国际友人称颂这位“当代神农氏”培育的杂交水稻,是*继指南针、火药、造纸、活字印刷之后,对人类作出的“第五大贡献”。袁隆*堪称“*骄傲”!

  深圳有一个名叫袁媛的7岁小女孩,在家里煤气泄漏、父母昏倒在浴室的危急时刻,她处乱不惊,果断打开浴室门,借助衣架捅开高高的窗户,迅速关上煤气,跑到客厅拨打了110、120求救电话,简洁准确地自报位置,使救援人员在第一时间赶到了现场,从而成功挽救了父母的生命。袁媛也因此当选为年龄最小的“*骄傲”。

  对“*骄傲”,你有怎样的思考?请结合自己的体验与感悟,写一篇文章。要求:①选好角度,明确文体(诗歌除外),自拟标题;②不要套作,不得抄袭;③不少于800字。

届高三语文试卷2

  一、名句名篇

  按要求填空。

  (1)念去去、千里烟波,____________。(柳永《雨霖铃》)

  (2)落花水香茅舍晚,__________________________(《寿阳曲·远浦帆归》)

  (3)同样是杜甫的诗,《江南逢李龟年》中“正是江南好风景,落花时节又逢君”与《登楼》“_____________________”一联所用手法相似。

  二、

  按要求选择。

  (1)小王不幸遭遇车祸,情绪低落,以下句子适合用来鼓励他战胜困难的一项是(_____)。

  A.沉舟侧伴千帆过,病树前头万木春。

  B.删繁就简三秋树,领异标新二月花。

  C.九层之台,起于累土。

  D.玉不琢,不成器。

  (2)把下列画线句放回原文序号处,最通顺的一项是(_____)。

  20xx年3月3日,《科学》杂志发表了题为《*许昌发现的更新世晚期古老型人类头骨》的论文。该文称在*许昌发现了生活在10.5万年至12.5万年之前的“许昌人”,①这是一种新的古老型人类,②*古人类学家在*十年中开展了大量野外调查、发掘和化石研究工作,先后在湖北郧西黄龙洞、湖南道县福岩洞,安徽东至华龙洞等地发现了珍贵的古人类化石,③从而对*更新世中、晚期人类演化的认识进一步深入。④

  发现“许昌人”的灵井旧石器时代晚期遗址,在20xx年和20xx年两次出土人类头骨化石,这些发现被评为当年的十大考古发现之一,遗址已被*公布为国家级文物保护单位。

  A.①B.②C.③D.④

  三、现代文阅读

  (一)阅读下文,完成第下列小题。

  全球化能力及评估

  ——经合组织《面向包容世界的全球化能力》报告述评

  ①随着全球化的深入发展,各国间的联系不断增强,人们需要与来自不同学科、文化的人合作,理解不同的文化和价值,解决复杂的问题,并创造经济与社会价值。面对全球化所带来的机遇和挑战,年轻一代需具备新的能力。多年来,教育界一直在讨论新形势下学生应具备哪些新的能力,以及如何培养这些能力,以帮助学生在未来的工作和生活中获得成功。

  ②20xx年5月14日,经合组织向七国教育部长提交了一份题为《面向包容世界的全球化能力》的报告。该报告首次提出了“全球化能力”评估。经合组织计划将该评估纳入20xx年PISA测评,评估学生的“全球化能力”。

  ③全球化能力是一个复杂的学*目标,可被分解为一些独立的、可测的学*目标。经合组织将全球化能力的宏观范畴解析为“维度”,每个“维度”包含不同“成分”。经合组织指出,全球化能力是指能够批判地从多角度分析全球和跨文化问题的能力;是能够理解差异如何影响自己和他人的感知、判断和想法的能力;是能够在尊重他人的基础上,以开放、适宜、有效的方式和不同背景的人交流的能力。按照此定义,全球化能力有着多重维度,包含不同文化背景的人建立有效、相互尊重关系所需的三个维度:知识和理解力、技能、态度。

  第一个维度是知识和理解力,指个体应具备相关知识和理解能力,能够应对全球化和跨文化发展所带来的挑战与机遇。

  第二个维度是技能,指个体为实现特定目标应具备的复杂的、组织良好的思维或行为能力。能够分析并进行批判性思考;能够与他人进行适当有效的沟通;能够理解他人的想法;能够调整自身感觉或行为,以适应新要求和新环境。

  第三个维度是态度,指_______________________________________________________________。

  ④态度基于价值观而形成。价值可定义为个体对所追求的理想目标持有的总体信念。价值观超越具体的行为和语境,能够影响个体的态度、判断和行为。在全球化能力框架中,尊重他人的尊严和文化多样性能够帮助学生正确处理与他人的差异、相关信息,也是他们做出明智判断的重要参考之一。

  ⑤测试内容和方式。20xx年PISA测评旨在建立一个独立的复合量表来测量学生运用知识理解、认识关系和观点,以及批判地思考特定目标或跨文化问题的能力。该量表将主要建立在全球化能力的认知项目上,包括全球问题的知识和理解力、跨文化知识和理解力、分析和批判性思维三方面评估内容。知识、理解和批判性思维是紧密联系的,构成了学生的跨文化思维能力。

  ⑥全球问题的知识和理解力意味着学生要熟悉一些跨越国界的重要问题,以及能够理解全球范围内的问题、趋势、体系的相互关系。在全球化知识领域,尽管学生能接触适量的信息,但若仅仅被动地汲取这些信息而不理解其深层意义及与其他知识的联系,容易对其产生误解。因而,全球问题的知识和理解力无法仅靠事实性知识获得,还要通过发现不同信息的意义及它们间的联系而*得。同时较强的理解能力表现为灵活的思维,以及良好的学*、组织知识的能力。

  ⑦跨文化知识和理解力,包括关于自身文化、其他文化,以及各文化间异同的知识。作为理解他人的视角之一,跨文化理解能够帮助人们区别特性和共性,以及在跨文化语境下理解不同观点间的关联方式。理解其他价值观,可以深化和改变自己的价值观,但并不意味着一定要接受它们。

  ⑧分析思维是指运用有逻辑的、系统的和有序的方法解决问题的能力,包括理解文本每个要素的意义的能力,以及审查这些要素间的联系和差异的能力。一些研究案例在国际化课堂中经常被用于教学。以案例研究为基础的教学方法能够帮助学生有逻辑地、系统地进行思考。案例研究应优先选择15岁学生熟悉的情境和题干材料,以促进学生积极参与测试项目。例如,一名14岁的秘鲁土著学生用古老的方言唱著名的歌曲,进而成为网络名人,而这种古老的方言在秘鲁一直遭受社会歧视。研究者可据此设计开放性问答题目,让学生思考这位少年积极保护本族语言和文化的动机,而相关多项选择题可以测试学生能否理解语言作为文化的一种表现形式的作用,以及主流语言和少数民族语言间的竞争。

  ⑨对我国教育的启示。要提高学生的全球化能力,首先要提高教师的全球化能力。因此,在教师培训中,我国应增加有关全球问题、跨文化的内容,帮助教师跟进国际发展趋势,树立国际化理念。

  ⑩全球问题主要涉及国际制度、主要历史事件和趋势,以及与全球主题相关的法律和政策框架,如气候变化和全球变暖、健康(例如流行病)、人口增长、移民、空气污染、国际冲突、贫困、人口老龄化等。面对这些问题,教师可以组织学生思考、辩论等,让学生在潜移默化中关注全球问题,逐渐认同人类的普遍价值。

  学校可以开展跨文化教育,让学生深入了解世界各地文化的差异,促进不同文化的理解和交流;加强与社会文化团体的联系,邀请社会文化团体到学校开展各类文化演出。有条件的学校可以推动学生参与国际交流,如跨国交流访学。

  教师可以通过文本、多媒体技术提供跨文化交流案例,引导学生在思考、交流、讨论中逐步意识到不同文化的差异和特性,进而学会尊重、包容不同文化;尊重他人,学会倾听不同观点,学会参考不同文化背景的人的观点。还可以通过让学生扮演来自不同文化背景的角色,让其更深入地体会不同文化,学*如何跟不同文化背景的人沟通、交流和合作。

  1.用简明的语言解释“全球化能力”这个概念。

  2.依据“图1”补充第三个维度的信息。

  3.结合文意,把以下概念填入方框内。

  4.第⑧段所举“秘鲁土著学生唱歌”的例子,目的分析正确的一项是

  A.阐明提高学生运用逻辑的、系统的和有序的方法解决问题的能力。

  B.阐明安全教学法能帮助提升学生有逻辑地、系统地分析思维的能力。

  C.阐明安全所设计的问题能引发学生思考保护本族语言和文化的动机。

  D.阐明测试学生能否理解语言作为文化的表现形式在传播中所起作用。

  5.以第⑥段为例,分析本文的语言特点。

  6.概括本文内容,据此判断本文适合的读者对象。(130字左右)

  (二)阅读下文,完成下列小题。

  孤独

  1、①大部分时候我妈独自一人生活。在阿克哈拉村,她的日常安保措施如下:在房子后墙上多挖一个后门,一旦有坏人闯入,就从后门撤退;若坏人追了上来,就顺着预先靠在后门外的梯子爬上屋顶;若是坏人也跟着爬上来,就用预先放在屋顶上的榔头敲他的头……此外,还有椅垫下藏刀子,门背后放石灰等诸多细节。她老人家国产连续剧看太多了。

  ②她说:“能不害怕吗?就我一个人。”

  2、③说来也奇怪,像我妈这么胆小的人,到了荒野里,一个人守着一大块地,生活全面敞开。再也没有墙壁了,也没有后门、梯子和榔头……却再也不提害怕的事了。

  ④她说:“怕什么怕?这么大的地方,就我一个人。”

  3、⑤真的再没有人了。在戈壁滩上,走一个小时也没遇到一个人。如同走了千百万年也没遇到一个人。不但没有人,路过的帐篷或地窝子也没有炊烟,眼前的土路上也没有脚印。四面八方空空荡荡。站在大地上,仿佛千万年后独自重返地球。

  ⑥关于地球的全部秘密都在风中。风声呼啸,激动又急迫。可我一句也听不懂。它拼命推我攘我,我还是什么都不明白。它转身撞向另一场大风,在我对面不远处卷起旋风,先指天,后指地。我目瞪口呆,仿佛真的离开地球太久。

  ⑦风势渐渐*息。古老的地球稳稳当当悬于宇宙中央。站在地球上,像站在全世界的至高点,像垫着整颗星球探身宇宙。日月擦肩而过。地球另一侧的海洋,呼吸般一起一伏。

  ⑧眼下唯一的人的痕迹是向日葵地。秧苗横*竖直,整齐茁壮。我走进去寻找我妈。又寻找赛虎和丑丑。地球上真的只剩我一人。

  ⑨我回到家,绕着蒙古包走一圈。突然看到一只鸡在附*的土堆旁踱步,并偏头看我。这才暗舒一口气。

  4、⑩我妈说:“我有时候想唱歌,却一首也想不起来。有时候突然想起来了,就赶紧唱。有时候给赛虎唱,有时候给兔子唱。”

  赛虎静静地听,卧在她脚边抬头看她。眼睛美丽明亮,流转万千语言。兔子却心不在焉,跳跳走走,三瓣嘴不停蠕动。

  兔子尾随她走向葵花地深处。兔子的道路更窄,兔子的视野更低。世界再大,在兔子那里也只剩一条深不见底的洞穴。而我妈高高在上,引领兔子走在幽深曲折的洞穴世界里。我妈不唱歌的时候,洞穴前不见头,后不见底。我妈唱歌的时候,洞穴全部消失。兔子第一次看到天空和海洋。

  劳动纯洁而寂静。我妈心里惦记着该锄草的那块地,惦记着几天后的灌溉,惦记着还没买到的化肥。所有这些将她的荒野生活填得满满当当。她扛着铁锨从地东头走到地西头,心里一件一件盘算。突然一抬头,看到了世上最美丽的一朵云。她满满当当的荒野生活瞬间裂开巨大的空白。她一时间激动又茫然。她想向世上所有人倾诉这朵云的美丽。她想:在倾诉之前,得先想好该怎么说。于是她就站在那里想啊想啊。云慢慢变化,渐渐*凡。她心中的措辞却愈加华美。她又想唱歌。仍旧想不起一首。这时她发现兔子不见了。她想,兔子和云之间肯定有某种神秘的联系。至少它们都是白的。

  赛虎也是白的。但它是不安之白,退避之白。它有无限心事。它总是不被允许进入葵花地。因为它的腿受过重伤,我妈不忍心它走动太多。她对它说:“不许跟着我,就在这里自己玩。我一会就回来接你。”它似乎听懂了,原地卧下。我妈边走边回头望。它一动不动凝视她,乖巧得*乎悲哀。它是黑暗之白,破碎之白。我妈无数次离它远去,也无数次转身重新走向它,抱起它,一同深入葵花地深处。

  5、我做好了饭,在蒙古包里等我妈回家。等着等着就睡着了。哪怕睡着了,也能清晰感觉到置身睡眠中的自己是何等微弱渺小。睡眠是地球上第二巨大的事物。第一巨大的是安静。我在梦中起身,推开门,走向远处的葵花地。走了千百万年也没能抵达。千百万年后我独自醒来。饭菜凉了,我妈仍然没有回家。

  6、吃饭的时候我妈再一次称赞:“这里真好!一个人也没有!”

  我说:“那出门干嘛还锁门”

  她语塞三秒钟:“关你屁事。”

  1.为什么第⑩到段反复写妈妈“唱歌”?

  2.扣住“凝视”一词,分析第段画线句的表达效果。

  3.结合第段的内容,自选一种表现手法,分析其作用。

  4.从构思的角度赏析文章结尾三段。

  四、诗歌鉴赏

  (三)阅读下面的诗歌,完成第下列小题。

  咏怀

  阮籍①

  壮士何慷慨,志欲成八荒。

  驱车远行役,受命念自忘。

  良弓扶乌号②,明甲有精光。

  临难不顾生,身死魂飞扬。

  岂为全躯士?效命寺战场。

  忠为百世荣,义使令名彰。

  垂声谢后世,气节髟有常。

  【注】①阮籍:三国时魏国人,“建安文学”重要作品阮瑀的儿子。②乌号:良弓口名。

  1.对作者与嵇康等七人并称的雅号判断正确的一项是

  A.建安七子B.邺中七子C.竹林七贤D.正始七贤

  2.对本诗赏析不正确的一项是

  A.“壮士”句,用“何”字,直抒胸臆,奠定全诗基调。

  B.“驱车”句,描绘出征,承上启下,推动情节的发展。

  C.“岂为”句,巧用反问,强调得语气,增强诗歌气势。

  D.“忠为”句,采用互文手法,交代了远赴战场的原因。

  3.清人方东树认为这首诗“雄杰壮阔”,结合具体诗句加以赏析。

  五、文言文阅读

  (四)阅读下文,完成第下列小题。

  ①公孙贺字子叔,北地义渠人也。贺祖父昆邪景帝时为陇西守以将军击吴楚有功封*曲侯著书十余篇。

  ②贺少为骑士,从军数有功。自武帝为太子时,贺为舍人,及武帝即位,迁至太仆。贺夫人君孺,卫皇后姊也,贺由是有宠。元光中为轻车将军。军马邑。后四岁,出云中。后五岁,以车骑将军从大将军青出,有功,封南窌侯。后再以左将军出定襄,无功,坐酎金,失侯。复以浮沮将军出五原二千余里,无功。后八岁,遂代石庆为丞相,封葛绎侯。时朝廷多事,督责大臣。自公孙弘后,丞相李蔡、严青翟、赵周三人比坐事死。石庆虽以谨得终,然数被谴。

  ③初,贺引拜为丞相,不受印绶,顿首涕泣,曰:“臣本边鄙,以鞍马骑射为官,材诚不任宰相。”上与左右见贺悲哀,感动下泣,曰:“扶起丞相。”贺不肯起,上乃起云,贺不得已拜。出,左右问其故,贺曰:“主上贤明,臣不足以称,恐负重责,从是殆矣。”

  ④贺子敬声,代贺为太仆,父子并居公卿位。敬声以皇后姊子,骄奢不奉法,征和中擅用北军钱千九百万,发觉,下狱。是时,诏捕阳陵朱安世不能得,上求之急,贺自请逐捕安世以赎敬声罪。上许之。后果得安世。安世者,京师大侠也,闻贺欲以赎子,笑曰:“丞相祸及宗矣。南山之行不足受我辞,斜谷之木不足为我械。”安世遂从狱中上书,告敬声与阳石公主私通,及使人巫祭祠诅上,且上甘泉当驰道埋偶人,祝诅有恶言。下有司案验贺,穷治所犯,遂父子死狱中,家族。

  节选自《汉书·公孙贺传》

  1.写出下列加点词在句中的意思。

  (1)然数被谴()(2)从是殆矣()

  2.为下列句中加点词语选择释义相同的一项。

  (1)父子并居公卿位

  A.安居乐业B.岁月不居C.居官任职D.后来居上

  3.第①段画线部分断句正确的一项是

  A.贺祖父昆邪/景帝时/为陇西守/以将军/击吴楚/有功封*曲侯/著书十余篇。

  B.贺祖父/昆邪景帝时为陇西守/以将军击吴/楚有功封*曲侯/著书十余篇。

  C.贺祖父昆邪景帝/时为陇西守/以将军击吴楚/有功封*曲/侯著书十余篇。

  D.贺祖父昆邪/景帝时为陇西守/以将军击吴楚有功/封*曲侯/著书十余篇。

  4.把下面的句子译成现代汉语。

  下有司案验贺,穷治所犯,遂父子死狱中,家族。

  5.根据第③段判断公孙贺不接受丞相之职的直接原因是

  A.公孙贺担心自己出生边远地,见识浅陋,不利管理国家。

  B.公孙贺担心自己靠军功做官,没有全面治理国家的才能。

  C.公孙贺担心自己像前面三位丞相一样被责罚,处危险中。

  D.公孙贺担心自己在英明君主的监管之下,难以施展才能。

  6.概括公孙贺的品格,并结合内容加以分析。

  (五)阅读下文,完成第下列小题。

  枕中十书序

  (明)袁宏道①

  ①人有言曰:“胸中无万卷书,不得雌黄人物。”然书至万卷,不几三十乘乎?除张司空外更几人哉!予于汉刘向、唐王仆射、宋王介甫、苏子瞻见之,然自子瞻迄今又三百余岁□,予于杨升庵、李卓吾②见之。或说卓秃翁,孟子之后一人。予疑其太过。又或说为苏子瞻后身,以卓吾生*历履,大约与坡老暗符,而卓老为尤惨。

  ②予昔令吴时,与卓吾游黄鹄矶,语次及著述书,李卓吾便点首曰“卓老子一生都肯让人。难著书则吾实实地有二十分胆量,二十分见识,二十分才力,若信得过否?”予唯唯。遂诘之曰:“尔数部中,谁是最得意者?”卓吾曰:“皆得意也,皆不可忽也。《藏书》,予一生精神所寄也;《焚书》,予一生事迹所寄也;《说书》,予一生学问所寄也,别有十种,约六百余纸,于中或集诸书,或附已意,此予一生神勇,游戏三昧所寄也,尚未终册,完当请门下校之。”自是分袂,伊南我北,卯酉相望。不数年,卓吾竟以祸殒,惜哉!

  ③已酉,予主陕西试事毕,复谢,天子恩命,夜宿三教寺。寺高敝筐中,获其稿读之,不觉大叫惊起。招提老僧,执光相顾。予遭询曰:“是稿何处得来,束之高阁?”老僧曰:“乡者温陵卓吾被逮时寄我物也。嘱以秘之枕中,毋令人见。今人已亡,书亦安用!”予曰:“嘻,奇哉!不意今日复睹卓吾也,卓吾其不死矣!”惜书前后厄于鼠牙,予以熹受卓吾之祝,故于燕居时续而全之,付冰雪阅而订之,藏之名山,俟有缘者梓而寿之。

  【注】①袁宏道:文学家,反对“文必秦汉,诗必盛唐”的风气,提出“独抒性灵,不拘格套”的性灵学说。②李卓吾:名赞,号卓吾,明代思想家、文学家,奉州学派的一代宗师。晚年被诬下狱,自制于狱中。

  1.填入第①段方框内虚词恰当的一项是

  A.也B.矣C.乎D.欤

  2.第①段写“坡老”的用意是。

  3.对第②段画浪线的句子理解正确的一项是

  A.李卓吾著书立说特别真诚用心,从不让人。

  B.李卓吾著书胆量见识和才力,超过一般人。

  C.李卓吾著书更讲究信用,超过常人二十分。

  D.李卓吾除著书外,其他方面都可以谦让人。

  4.从句式的角度,赏析第②段画直线部分的句子。

  5.分析作者的写作意图。

  六、材料作文

  随着国门打开,经济发展和文化交流的不断增强,现代生活方式层出不穷:传统生活方式面临种种挑战,人们处于难以抉择的境地。

  对“传统生活方式面临挑战”的现象谈谈你的看法。

  要求:(1)自拟题目;(2)不少于800字。

届高三语文试卷3

  一、现代文阅读

  阅读下面的文字,完成下列小题。

  “驼铃古道丝绸路,胡马犹闻唐汉风。”古代丝绸之路架起了一座交流物产、连通人心的桥梁,对我国各民族交流融合、对东西方经济文化交往都起到了十分重要的作用。

  古代丝绸之路大体有草原道、绿洲道、茶马道以及海上道四条。除了汉族,北方和西北游牧民族也是丝绸之路的重要开拓者,他们的马队和骆驼队踏出了一条横贯欧亚大陆的草原丝路。他们的迁徙浪潮、相互交往以及游牧经济特点,使其自然而然地成为古代丝绸之路上的重要角色。

  继月氏、匈奴之后,鲜卑、吐谷浑、吐蕃、回纥、党项等民族,都曾和丝绸之路结下不解之缘,有的甚至一度控制了草原道和绿洲道,成为经营东西方贸易的主角。公元439年,鲜卑建立的北魏*统一了我国北方,使丝绸之路自汉代以来再度繁荣起来。北魏、西夏占据河西走廊后,吐谷浑控制的“青海道”和吐蕃控制的“青唐道”成为中原和南方通往西域的通道。因此,“青海道”又称“吐谷浑道”,“青唐道”又称“吐蕃道”。再看回纥,其与唐朝贸易换回的绸绢,除了供贵族享用,还通过“草原道”大量转输到西方。“安史之乱”后,吐蕃完全占据了河西走廊及陇右地区,传统的丝绸之路东段受到阻遏,唐朝和西域各国的交往一度绕道回纥居住地。因此,这一时期的草原丝路有“回纥道”之称。

  在肯定我国古代北方和西北游牧民族为丝绸之路的开拓与繁荣做出重要贡献的同时,更要充分认识中原王朝的主体作用。张骞出使西域之后,汉、唐、元、明各朝代为了经营西域,保障丝绸之路畅通,在丝绸之路沿途设置馆舍以提供食宿,建立都护府、都督府等以加强治理。这些措施对保障丝绸之路的畅通和安全具有决定性作用。

  丝绸之路密切了我国古代民族关系,也密切了东西方关系。中原、江南以及巴蜀的名茶不仅输入西域、青藏高原与漠北,也输入西方。在西夏与宋朝的贸易中,“惟茶最为所欲之物”。同时,西域和中亚、欧洲的物产和文化也传入内地,今天内地随处可见的石榴、葡萄、大蒜、菠菜等,都是汉朝时从西域传入内地的。到了唐朝,丝绸之路的东端终点长安、洛阳成为西城文化的荟萃之地,上至宫廷、下至百姓无不受西域文化的影响,穿胡服、食胡饼、听胡音成为时尚。到了宋朝,*的茶叶和丝绸被大量贩运到西方,西方的玛瑙、琥珀、琉璃等被大量转售到*。元朝时,*的雕版与活字印刷术、火药等通过丝绸之路传到欧洲,欧洲和***的天文、数学、医药、建筑等科学技术也传入*。

  中华民族是诸多民族相互接触、融合形成的多元统一体,在其形成过程中,丝绸之路是重要的沟通渠道与交流舞台。丝绸之路像一条纽带,联结着农耕文明和游牧文明,联结着内地汉族人民与边疆少数民族人民。当前,深入推进“一带一路”建设,续写古代丝绸之路的传奇,它不仅会继续描绘我国各民族交流融合五彩斑斓的画卷,还会进一步架起东西方各民族交流交往的桥梁,推进沿线国家和地区的经济发展。

  1.下列关于原文内容的表述,不正确的一项是

  A. 鲜卑、吐谷浑、吐蕃等少数民族有的曾一度控制了丝绸之路的草原道和绿洲道,成为当时经营东西方贸易的主角。

  B. 北方和西北游牧民族与汉族一起用马队和骆驼队踏出了一条横贯欧亚大陆的草原丝路,成为古代丝绸之路上的重要角色。

  C. 丝绸之路的东端终点长安和洛阳在唐朝时成为西域文化的荟萃之地,当时的朝野上下无不受到西域文化的影响。

  D. 深入推进“一带一路”建设,有助于推进沿线国家和地区的经济发展,促进我国各民族的融合及东西方各民族的交往交流。

  2.下列理解和分析,不符合原文意思的一项是

  A. 古代丝绸之路大体上有草原道、绿洲道、茶马道、海上道四条,汉族及北方、西北的游牧民族都是其重要的开拓者。

  B. “青海道”“青唐道”和草原丝路之所以又称“吐谷浑道”“吐蕃道”和“回纥道”,是因为它们曾分别被吐谷浑、吐蕃和回纥控制过。

  C. 古代丝绸之路的开拓与繁荣,不能否定我国古代北方和两北游牧民族所做出的贡献,更不能忽视中原王朝的长期经营和有效治理。

  D. 丝绸之路联结着农耕文明与游牧文明、内地汉民族与边疆少数民族,对中华民族这个多元统一体的形成起了巨大的促进作用。

  3.根据原文内容,下列说法不正确的一项是

  A. 古代丝绸之路在汉、北魏、唐、宋、元等朝代比较繁荣,但因战乱等因素的影响,它在历史上也有过衰落的时候。

  B. 在西夏和宋朝的贸易中,“惟茶最为所欲之物”,这说明当时西夏人已有饮茶的*惯,可见物产上的互通有无也可改变人们的生活方式。

  C. 从生活中常见的胡桃(核桃)、胡瓜(黄瓜)、胡豆(蚕豆)、胡萝卜等农产品的名称可以看出,它们很可能是从西域传入中原的。

  D. 丝绸之路是一条连通东西方的重要商贸之路和文化交流之路,元朝以前丝绸之路以物产交流为主,而元朝以后则以科技文化交流为主。

  (一)文言文阅读

  阅读下面的文言文,完成下列小题。

  任忠,字奉诚,小名蛮奴,汝阴人也。少孤微,不为乡党所齿。及长,谲诡多计略,膂力过人,尤善骑射,州里少年皆附之。侯景之乱,忠率乡党数百人,随晋熙太守梅伯龙讨景将王贵显于寿春,每战却敌。土人胡通聚众寇抄,忠与主帅梅思立并军讨*之。会京城陷,忠坚戍晋熙,旋复京。侯景*,授荡寇将军。

  王琳立萧庄,署忠为巴陵太守。华皎之举兵也,忠预其谋。及皎*,高宗以忠先有密启于朝廷,释而不问。五年,*北伐,忠将兵出西道,击走齐历阳王高景安于大岘,逐北至东关,克其东西二城。进军蕲、谯,并拔之。径袭合肥,入其郛。进克霍州。以功授员外散骑常侍,封安复县侯,邑五百户。十一年,加北讨前军事,进号*北将军,率众步骑趣淮南路。周遣王延贵率众为援忠大破之生擒延贵后主嗣位进号镇南将军给鼓吹一部入为领军将军,加侍中,改封梁信郡公,邑三千户。出为吴兴内史,加秩中二千石。

  及隋兵济江,忠自吴兴入赴,屯军朱雀门。后主召萧摩诃以下于内殿定议,忠执议曰:“兵家称客主异势,客贵速战,主贵持重。宜且益兵坚守宫城,遣水军分向南豫州及京口道,断寇粮运。待春水长,上江周罗殉等众军,必沿流赴援,此良计矣。”众议不同,因遂出战。及败,忠云:“陛下唯当具舟楫,就上流众军,臣以死奉卫。”后主信之,敕忠出部分。忠辞云:“臣处分讫,即当奉迎。”后主令宫人装束以待忠,久望不至。隋将韩擒虎自新林进军,忠乃率数骑往石子岗降之,仍引擒虎军共入南掖门。台城陷,其年入长安,隋授开府仪同三司。卒,时年七十七。

  1.下列对文中画波浪线部分的断句,正确的一项是( )

  A. 周遣王延贵/率众为援忠/大破之/生擒延贵/后主嗣位进号镇南将军/给鼓吹一部

  B. 周遣王延贵率众为援/忠大破之/生擒延贵后/主嗣位进号/镇南将军给鼓吹一部

  C. 周遣王延贵率众为援/忠大破之/生擒延贵/后主嗣位/进号镇南将军/给鼓吹一部

  D. 周遣王延贵率众/为援忠/大破之生/擒延贵/后主嗣位/进号镇南将军/给鼓吹一部

  2.下列对文中加点词语的相关内容的解说,不正确的一项是( )

  A. 乡党,指乡里、家乡、乡族朋友;古代五百家为党,一万二千五百家为乡,合而称乡党。

  B. 太守,官名,秦置郡守,汉景帝时改名太守,为一郡最高的行政长官,隋初以州刺史为郡长官,宋只用作知府、知州的别称,明清时专指知府。

  C. 路,*古代一种行政区划,相当于现在的省或地区;如辛弃疾《永遇乐·京口北固亭怀古》中“烽火扬州路”的“路”就是这个意思。

  D. “邑”和“秩”都是古代帝王对有功的臣子的赏赐,其中“邑”属于赏赐给臣子的禄米粮食,“秩”是分封给臣子的封地。

  3.下列对原文有关内容的分析和概括,不正确的一项是( )

  A. 侯景作乱,任忠随主帅击败叛军和土匪的攻击,为坚守国土、收复京城立下汗马功劳,被皇帝授予荡寇将军的头衔。

  B. 陈朝军队北伐,任忠从西路出击,击败历阳王高景安,将其追赶到东关,因为有功被授员外散骑常侍,封安复县侯,邑五百户。

  C. 隋军渡江南侵,任忠主张坚守,不宜出击,等待敌人粮断援尽,我军后援到达,一定会打破敌军。但众人没有采纳任忠的建议,仓促出战,导致失败。

  D. 任忠向后主表示出城部署好军队后就立即迎接后主,结果任忠向隋将韩擒虎投降,并带领韩擒虎的军队一起进入南掖门。

  4.把文中画横线的句子翻译成现代汉语。

  (1)少孤微,不为乡党所齿。及长,谲诡多计略,膂力过人。

  (2)忠云:“陛下唯当具舟楫,就上流众军,臣以死奉卫。”后主信之,敕忠出部分。

  阅读下面一首宋词,完成下列小题

  感皇恩(陆游)

  小阁倚秋空,下临江渚。漠漠孤云未成雨。数声新雁,回首杜陵何处。壮心空万里,人谁许①!

  黄阁②紫枢,筑坛③开府。莫怕功名欠人做。如今热计,只有故乡归路。石帆山脚下,菱三亩。

  【注】①许,赞赏、信任。②黄阁,宰相官署,借指宰相;紫枢,宋代戎服多用紫色,故以紫枢指枢密使。③筑坛,用了汉高祖设坛场拜韩信为大将的典故;开府,开幕府,置僚属。

  1.下面对这首词的赏析,不恰当的两项是( )( )

  A. 上阙写景视觉与听觉结合,远*上下结合,从秋空到江渚到孤云到雁声,营造了冷清开阔的意境。

  B. 杜陵原为西汉宣帝陵寝,在长安附*,而长安又是汉唐旧都,词人用借代修辞手法来借指南宋都城。

  C. 上阙以写景起而以抒情终,下阙先抒情后写景,或因景生情,或因情见景,景语即情语,情景交融。

  D. 结句“石帆山脚下,菱三亩”,描绘出一幅江南水乡的美丽图景,表现了词人对理想生活的向往。

  E. 此词通过用典,丰富了作品的内涵,引发读者联想,也较好地表达了词人内心深沉的感慨。

  2.简要分析这首词中词人所抒发的复杂情感。

  二、文言文阅读

  补写出下列句子中的空缺部分。

  (1)在《醉翁亭记》中,表达太守既能同醉,又能“自醒”体察万物,体现作者志趣的句子是:“_______________________________________。”这两句表达了欧阳修的豁达和与民同乐的感情。

  (2)荀子《劝学》中用蟹借穴而居来比喻说明学*者用心浮躁的一句是“____________”

  (3)李白在《蜀道难》中以浪漫主义手法,着力描绘了秦蜀道路上险峻而瑰丽的山川,如“___________________________________________________”写出了漩涡飞转,瀑布飞泻的气势。

  三、诗歌鉴赏

  下列各句中加点成语的使用,全都不正确的一项是( )

  ①在儒家传统中,孔子和孟子总是形影相吊:既有大成至圣,则有亚圣;既有《论语》,则有《孟子》。

  ②从众星捧月到被冷落一旁,一些退休官员认为周围的一切都改变了,甚至出现了许多不适应的状况。

  ③我头上树木的宏伟、四周灌木的纤丽、脚下花草的纷繁使我目不转睛,不知道应该观赏还是赞叹。

  ④作者希望读者从这粗枝大叶的讨论中,领略运用文字所应有的谨严精神,养成读写的好*惯。

  ⑤改革开放以来,我们的物质生活发生了很大的变化,但也不能用一饭千金的方式去挥霍浪费。

  ⑥在亘古而苍茫的宇宙间,路遥《*凡的世界》发出的虽是*凡的声音,但这种声音荡气回肠。

  A. ②③⑥ B. ①④⑤ C. ①③⑤ D. ②④⑥

  四、情景默写

  下列各句中,没有语病的一项是( )

  A. 《三体》被不少学者、媒体人乃至创业者、企业家热捧的原因,是它打破了严肃文学与通俗文学界限,其中呈现的“中华形象”和宏大视野使它走出了纯文学的局限空间造成的。

  B. 以美国为代表的市场化征信模式发展自从开始成熟,征信机构向信息提供者付费获取信息,制作如个人信用报告及个人信用评分等个人征信产品,使用者需付费获得这些产品。

  C. 创建文明城市,要注重发挥文化的滋养涵育作用,不仅要凭借建筑风格提升城市品位,还要用人文精神增添城市魅力。

  D. 虽然国家对农村文化建设投入了不少资金,但目前我国农村文化仍旧处于有阵地却缺乏后续文化建设设施完善。

  五、选择题

  下列句子用语使用得体的`一项是( )

  A. 拙作《老子补正》新成,惶恐以赠,尚祈雅正,不吝赐教。

  B. 书信:毕业之后,学生垂念师恩。值此春节到来之际,谨祝恩师节日快乐,万事如意!

  C. 值此中秋佳节,令女出阁之庆,聊备薄酒,以飨众亲朋好友。

  D. 请柬:新居落成,我明天搬迁,为答谢您的祝贺,特于府上备下薄酒,恭请光临。

  在下面一段文字横线处补写恰当的语句,使整段文字语意完整连贯,内容贴切,逻辑严密,每处不超过15个字。

  一些有关筷子的*俗至今流传,比如,①__________________________。 首先,要会摆放筷子。筷子要整齐地摆放在进食者的右手边,不要将一双筷子一反一正并列摆放,也不要将筷子用于进食的一端朝桌外摆放。其次,②_________________________。不可取位过高,这样容易被认为清高孤傲,也有远离父母家门的嫌疑;③_______________________,这样容易被认为笨拙,也显得缺乏教养。在举筷夹菜时,不要乱翻,更不要将筷子伸到别人面前的碗盘中夹菜。

  根据下面图表提供的信息回答问题,每处不超过15字,不得出现数字。

  从表中可以看出,20xx年至20xx年五年间我国在初、中、高等教育经费投入方面变化比较明显的特点是:_____________;__________________。但是,也可以看出我国在初、中、高等教育经费投入方面一直存在一个问题,那就是: _______________。

  六、语言应用

  阅读下面的材料,根据要求写一篇不少于800字的文章。

  王铎 绘

  荆轲竟然是女的,“诗仙”李白变成了刺客,名医扁鹊是用毒高手……这样的角色错位会不会让你大吃一惊?据媒体报道,一款名为“王者荣耀”的手机游戏时下大热。角色名字都取自*历史和传说中的著名人物,可在游戏中,从形象到内容都与历史或传说完全不符合。对此,是否要全盘否定,到底应该怎么看?

  读完上述材料,你有何感想?请综合材料的内容,写出自己的思考。

  要求选好角度,确定立意,选好文体,自拟标题;不要套作,不要抄袭。


届高三语文试卷 (菁华3篇)扩展阅读


届高三语文试卷 (菁华3篇)(扩展1)

——届九年级下语文试卷 (菁华3篇)

届九年级下语文试卷1

  一、选择题

  下面语段中加点字的注音全都正确的一项是( )

  西溪之胜,独在于水。水道如巷、鱼塘栉比,具有净化水质、蓄洪防旱和维护生物多样性等功能,尽显“芦锥几顷界为田,一曲溪流一曲烟”的绮丽风光。西溪之美,美在生态。湿地内芦白柿红、竹翠梅香、鹭栖燕翔,拥有“十里梅花,百顷芦荡,千点白鹭,万年翠竹”的美景。西溪之重,重在文化。伫立西溪,饱览前人留下的大批诗文词章和“龙舟胜会”“竹林挖笋”等诸多传统民俗,浓厚的文化气息扑面而来。

  A、jié qǐ xī zhù B、zhì qǐ qī zhù

  C、zhì yǐ qī chù D、jié qǐ xī chù

  下列句子中没有错别字的一项是( )

  A、在职业生涯的第20个赛季,洛杉矶湖人队科比·布莱恩特正式宣布他将在本赛季结束后退役,这引发了微信朋友圈持续的刷屏感慨。

  B、*明确*战略方针和总体布署,推动*工作取得显著成效。

  C、西湖秀美的风景妇儒皆知,一到周末,在杭城读书的大学生常常邀上挚友前往西湖游玩。

  D、“不就是等一场久违的雪吗?简直就是小提大作!”穿丝袜的摩登女笑得得意的背后藏着更多的是阴险。世界迎合了她魅惑的性感,把她水性杨花的个性表现得淋漓尽致。

  下列句子中加点的词语使用恰当的一项是( )

  A、《捉妖记》能在一众商业大片之中脱颖而出,这与其精良的制作是分不开的。

  B、个别同学对他这次考试取得的进步表示怀疑,他却忍辱负重,依然刻苦学*。

  C、惟有牢记血的教训,严抓安全监管,严惩失职渎职者,才能有效遏止安全事故,保障公众生命安全。

  D、市汉字听写大赛前夕,老师喋喋不休地嘱咐我们一定要胆大细心。

  下列各句中,没有语病的一句是( )

  A、武亦姝在最后一场比赛中,与陈更、李宜幸、张淼淼强强PK,以317分获得攻擂,最后战胜彭敏,成为冠军。

  B、戛纳电影节将于20xx年5月在杭州举办MIPCHINA国际影视内容高峰论坛,这是戛纳电视节品牌活动首次登陆*。

  C、网络之所以让那么多的网友着迷的重要原因,是因为他们下单后输入账号密码时基本没有感觉到是在花钱。

  D、杭州举办20xx年第19届亚运会,将推动奥林匹克运动的发展,同时将加强杭州的国际知名度,促进杭州经济、社会的全面发展。

  二、情景默写

  请根据提示的语境内容,填写出相应的古诗文名句。

  ①鬓微霜,又何妨!__________,___________?(《江城子 密州出猎》)

  ②臣本布衣,躬耕于南阳,__________,___________。(《出师表》)

  ③_____________,_________,将军白发征夫泪。(《渔家傲 秋思》)

  ④刘方*《月夜》中表现春气萌发,蛰虫涌动的诗句是:__________,__________。

  ⑤温庭筠《商山早行》中由十个名词构成,蕴含六种景物,表现了早行之早的句子是:__________,__________。

  ⑥辛弃疾《破阵子 为陈同甫赋壮词以寄之》中,辛弃疾用“__________,__________”两句概括了自己的最高理想,表达了爱国激情和雄心壮志。

  三、名著阅读

  文学常识和名著阅读。

  (1)阅读下面的文字,在括号内填写恰当的内容。

  阅读名著能拓宽视野,启迪人生。我们读中外作品,从《我的叔叔于勒》中的①_______、《变色龙》中的奥楚蔑洛夫、《范进中举》胡屠户中的身上了解 “变色龙”的特点;读吴承恩的《西游记》,能让我们真切地感受到②________(外号+人名)在大闹天宫的过人英勇;读鲁迅先生的《从百草园到三味书屋》《藤野先生》《③________》,我们徜徉在《朝花夕拾》的世界里,了解先生青少年时期的生活经历与体验。

  (2)阅读下面一段话,完成下面的题目。

  某班同学在阅读《西游记》时,对唐僧这个人物有两种不同看法,为此语文老师组织了一场辩论。假如你是正方,针对反方辩词该怎么说?

  反方:我方认为,唐僧胆小懦弱、本性昏愚、忠奸不分;他不辨是非,总是听信猪八戒的挑唆;他对待妖怪心慈手软,对待除妖有功的孙悟空却用紧箍咒来折磨;历尽千辛万苦到达西天后,面对传经者索要的“人事”,他却不知反抗,唯唯诺诺,实在可气。

  正方:____________________

  四、综合性学*

  请根据下面的材料和具体要求完成任务。

  4月4日,20xx年“国际安徒生奖”在全球最大的童书展会博洛尼亚书展上揭晓,*儿童文学作家曹文轩获得该奖项,这是*作家第一次获得此项殊荣。

  获奖词:“曹文轩的作品读起来很美,书写了关于悲伤和苦痛的童年生活,树立了孩子们面对艰难生活挑战的榜样。”

  质疑的声音:《*》刊登了一名高中生评价曹文轩作品的文章。作者认为:“曹文轩笔下的苦难似乎只是单纯地为了表现人物情感的,缺乏更为深刻的含义。而这种含义原本是可以在作品中表现出来的,但是他作品中大量的极具美感的段落反而阻碍了这种表达。”“没有快乐只追求感动的儿童文学,无疑会对儿童的心理造成不利的影响,这不是儿童文学的本质,也不是儿童文学应该的样子。”

  (1)初阳文学社团课上要就以上两种不同的看法开展讨论,请你准备发言,表达自己的观点并简述理由。

  (2)初阳文学社想邀请曹文轩先生为同学们做视频讲座,和大家分享他的创作经验,时间是本周五下午三点,地点在我校二楼报告厅,请你以初阳文学社社长的身份,向曹文轩先生发出书面邀请。

  五、现代文阅读

  阅读下面的文章,完成下列小题。

  晶莹的泪珠

  陈忠实

  我手里捏着一张休学申请书朝教务处走去。

  我要求休学一年。

  我敲响了教务处的门。获准以后便推开了门,一位年轻的女教师正伏在米黄色的办公桌上,手里拿着木杆蘸水笔在一本表册上填写着什么。“老师,给我开一张休学证书。”

  她抬起头来,诧异地瞅了我一眼,然后拿起我的申请书看着。她很快看完了,抬起头来问:“就是你写的这些理由吗?”

  “就是的。”

  “不休学不行吗?”

  “不行。”

  “亲戚全都帮不上忙吗?”

  “亲戚……也都穷。”

  “可是……你休学一年,家里的经济状况也不见得能改变,一年后你怎么能保证复学呢?”

  于是我就信心十足地告诉她我父亲的精确计划:待到明年我哥哥初中毕业,父亲谋划着让他投考师范学校,师范学校的学杂费和伙食费全由国家供给,据说还发三块零花钱。那时候我就可以复学接着念初中了。

  她轻轻舒了口气,拉开抽屉取出公文本在桌子上翻开,用那支木杆蘸水笔,在墨水瓶里蘸上墨水后又停下手,问:“你家里就再想不出办法了?”我看着那双充满忧郁的眼睛,突然意识到因为我的休学致使她心情不好。她*时就是在教务处做些抄抄写写的事,我和她几乎没有说过话,甚至至今也不知道她的姓名。我便说:“老师,没关系,我年龄小。她说:“白白耽误一年多可惜!”随之又换了一种口吻说:“我知道你的名字,也认得你。每个班前三名的学生我都认识。”我的心情忽然灰暗起来,我没有开口。

  她终于落笔填写了公函,取出公章在下方盖了,又在切割线上盖上一枚合缝印章,吱吱吱撕下并不交给我,放在桌子上,然后把我的休学申请书抹上糨糊,贴在公文存根上。

  她做完这一切才重新拿起休学证书交给我,叮嘱道;“装好,别丢了。明年复学时拿着来找我。”

  我把那张休学证书折叠了两番装进了口袋。她从桌子那边绕过来,又从我的口袋里掏出来塞进我的书包里,说:“明年这阵儿你一定要来复学。”我向她深深地鞠了躬,就走出门去。忽然听到背后一声“等等”,她跟上我,和我并排在廊檐下的台阶上走着。我忽然心情很不好受,在争取得到了休学证书后,我很不愿意看见同班同学熟悉的脸孔,便低了头匆匆走起来,凭感觉可以知道她也加快了脚步,几乎和我同时走出学校大门。

  校门外拥来一拨偏远地区的学生,熟悉的同学便连连问我:“你来得早,报到了吧?”我含糊地笑笑就走过去了,想尽快远离正在迎接新学期的洋溢着欢乐气氛的学校大门。她又喊了一声“等等”。

  我停住脚步。她走过来拍了拍我的书包:“别把休学证书弄丢了。”我点点头。她又安慰我说:“我同意你的打算,休学一年不要紧,你年龄小。”

  我抬起头来,猛然看见那双睫毛很长的眼睛里充满了泪水,像雨雾中正在涨溢的湖水,泪珠在眼眶里打着旋儿,晶莹透亮。我瞬即垂下头避开她的目光,要是再在她的眼睛处多驻留一秒,我肯定就会嚎啕大哭。我低着头咬着嘴唇,脚下盲目地拨弄着一块碎瓦片来抑制情绪,一股热辣辣的酸流从鼻腔倒灌进喉咙里去。我终于扬起头鼓着劲儿说:“老师……我走咧……”

  她的手轻轻搭上我的肩头:“记住,明年的今天来报到复学。”

  我看见两滴晶莹的泪珠从她眼睫毛上滑落下来,缓缓流过一段,就在鼻翼两边挂住。我再次虔诚地向她鞠了一躬,然后转身走了。

  25年后,父亲在弥留之际,对坐在他身边的我说:“我有一件事对不住你……”

  我惊讶得不知所措。

  “我不该让你休那一年学!”

  我浑身颤栗,久久无言。我高中毕业名落孙山,回到乡村,陷入无边无际的彷徨苦闷,那时我曾经猴急似的怨天尤人:“全都倒霉在休学那一年……”我1962年毕业恰逢*经济最困难的年月,高校招生名额大大减少,我们班剃了光头,四个班也仅仅考取了几个人,而上一年我们学校有50%的学生考取了大学。父亲说:“错过一年……让你错过了20年……而今你还算熬出点名堂了……”

  我猛然想起休学那年,出校门时那位女教师溢满眼眶又流挂在鼻翼上的晶莹的泪珠。我对已经跨进黄泉路上半步还向我忏悔的父亲讲了那一串泪珠的经历,父亲听后喃喃地说:“可你……怎么……不早点给我……说这女先生哩……”

  我今天把40年前的这一段经历写出来,对自己算是一种虔诚祈祷。当今,各种欲望膨胀成一种强大的浊流冲击着所有大门窗户和每一个人的心扉,我企望自己的泪泉,如女教师那饱含晶莹泪珠的泪泉,不至于堵塞,更不敢枯竭,那是滋养生命灵魂的泉源,也是滋润民族精神的泉源哦……

  1.阅读全文,请用词语概括女教师在给“我”办休学证书过程中的心理变化过程。

  2.阅读文中画线句子,简析人物心理。

  (1)我的心情忽然灰暗起来,我没有开口。

  (2)我向她深深地鞠了躬,就走出门去。

  3.父亲在弥留之际对“我”所说的话有什么作用?请结合文意分析。

  4.“40年前”和“今天”作者对女教师泪珠的感受有何不同?从作者“今天”的感受中你获得了怎样的感悟?

  阅读下面的文章,完成下列小题。

  切莫握着鼠标忘了笔杆

  在教育部举行的第十届“全国推广普通话宣传周”新闻发布会上,教育部副部长、国家语委主任赵沁*指出,现在很多学生写字时只知道那个字的大概形状,但是具体怎么写就不会了,而用电脑这个字很快就能敲出来。长期使用电脑导致学生提笔忘字,这与一个书法大国很不匹配。专家呼吁,在网络时代,不能“握着鼠标忘了笔杆”。

  曾几何时,一个人写字的水*意味着他的修养和素质。而E时代的到来,电脑打字的书写方式,已悄悄颠覆了人们传统的手写文字的*惯。面对着曾经熟悉的文字,手指放在键盘上,就知道该按哪个钮,但拿起笔,许多字却印象模糊写不出来。这种“手写时代”老年人才会经常出现的“提笔忘字”现象,在年轻人的身上表现得愈来愈明显,被人戏称为“电脑失写症”。

  一项针对在校大学生的调查显示,超过半数大学生对电脑等数码多媒体有依赖感,遇到结构稍微复杂一点的汉字就要借助手机和电脑来“救助”。①在某人才招聘会上,一些应聘大学生的电子简历装帧精美,电脑编排功夫非同一般,但他们的汉字书写简直没法看,甚至有人连填表书写“聘任”的“聘”字都要抬头看招聘现场的大标语。去年,某媒体记者在IT行业聚集区街头随便找行人做了个调查,让他们写出“尴尬”一词,结果十余位过路的大学生、高三学生和公司白领等,竟无一人能正确写出来,着实令人“尴尬”了一回。

  众所周知,汉字博大精深,底蕴丰厚,承载了中华民族的文明和智慧,是世界文化的重要组成部分,是我们民族的象征。不论是方正的楷书、俊秀的小篆,还是遒劲的草书,其兼具象形和表意的特点及蕴含的思想和文化内涵,即便再发达的科技也无法模拟和取代。国外的专家学者认为:每个汉字都是一个“集成电路”,在开发智力上是没有止境的;认识的汉字越多,联想就越丰富。正因为如此,日本、韩国、新加坡等国家的一些中小学课程中,都将“书法”列入必修课。

  在很多外国人都把汉字当成一门艺术,积极地学*中文的今天,国人却因电子书写的推广,让“握着鼠标忘了笔杆”成为另类景观,不仅提笔忘字,而且提笔写错字的现象比比皆是,还呈现出一种越来越年轻化甚至是儿童化的趋势,实在是一种悲哀。这样的“写字困境”频现,不仅隐藏着学校教育特别是语文教育、中文教育的失落与迷失,也意味着传统的汉字书写艺术及其所传承的*语言文化也在离我们渐行渐远,值得我们反思和重视。

  当务之急,有必要将汉字作为一种“国粹”来进行保护,让每一个公民对这种民族文化敬畏有加。青年人尤其是大学生,应在熟练掌握多媒体技术的同时,重拾传统学*方式,培养阅读图书、手写文字*惯;我们的中小学校也要与时俱进,在研究语言教学的技术手段、推进板书教学的同时,抓好学生的“手写”训练,不仅要求学生用手写作业,更要在我们庞大而运转自如的考试机器里,加入关于写字的内容。如此,“提笔忘字”的“别扭”才会淡出我们的视野。

  1.第一段引述国家语委主任赵沁*的话,其作用是什么?

  2.根据文意分点概括“握着鼠标忘了笔杆”的危害性。

  3.文中画线语句采用什么论证方法?有什么表达作用?

  4.根据全文内容,概括本文作者所要阐述的主要观点。

  六、文言文阅读

  阅读下面的文言文,完成下列小题。

  司空图

  司空图,字表圣,河中虞乡人。图,咸通末擢进士,礼部侍郎王凝特所奖待俄而凝坐法贬商州图感知己往从之。凝起拜宣歙观察使,乃辟置幕府。召为殿中侍御史,不忍去凝府,台劾,左迁光禄寺主簿,分司东都。卢携以故宰相居洛,嘉图节,常与游。携还朝,过陕虢,属于观察使卢渥曰:“司空御史,高士也。”渥即表为僚佐。会携复执政,召拜礼部员外郎,寻迁郎中。

  黄巢陷长安,将奔,不得前。图弟有奴段章者,陷贼,执图手曰:“我所主张将军喜下士,可往见之,无虚死沟中。”图不肯往,章泣下。遂奔咸阳,间关至河中。景福中,拜谏议大夫,不赴。后再以户部侍郎召,身谢阙下,数日即引去。

  图本居中修山王官谷,有先人田,遂隐不出。作亭观素室,悉图唐兴节士文人。名亭曰休休,作文以见志曰:“休、美也,既休而美具。故量才,一宜休;揣分,二宜休;耄而聩,三宜休;又少也惰,长也率,老也迂,三者非济时用,则又宜休。”因自目为耐辱居士。其言诡激不常,以免当时祸灾云。

  每岁时,祠祷鼓舞,图与闾里耆老相乐。王重荣父子雅重之,数馈遗,弗受。尝为作碑,赠绢数千,图置虞乡市,人得取之,一日尽。时寇盗所过残暴,独不入王官谷,士人依以避难。

  朱全忠已篡,召为礼部尚书,不起。哀帝弑,图闻,不食而卒,年七十二。

  1.下列句子中加点词的解释错误的一项是( )

  A、俄而凝坐法贬商州(由……而获罪) B、属于观察使卢渥(通“嘱”,叮嘱)

  C、不获从,又还河中(跟从) D、悉图唐兴节士文人(图画)

  2.对文中画波浪线的部分断句正确的一项是( )

  A、礼部侍郎王凝特所奖待/俄而凝坐法/贬商州图感知/己往从之。

  B、礼部侍郎王凝特所奖/待俄而凝坐法贬商州/图感知己/往从之。

  C、礼部侍郎王凝特所奖待/俄而凝坐法贬商州/图感知己往/从之。

  D、礼部侍郎王凝特所奖待/俄而凝坐法贬商/州图感知己/往从之。

  3.用现代汉语写出下面文言句子的意思。

  ①我所主张将军喜下士,可往见之,无虚死沟中。

  ②王重荣父子雅重之,数馈遗,弗受。

  4.司空图是个怎样的人?请结合文意评析。

  七、诗歌鉴赏

  阅读下面这首古诗,完成下列小题。

  登金陵凤凰台①

  [唐]李白

  凤凰台上凤凰游,凤去台空江自流。

  吴宫花草埋幽径,晋代衣冠成古丘。

  三山②半落青天外,一水③中分白鹭洲。

  总为浮云能蔽日,长安不见使人愁。

  (注)①凤凰台:在南京凤凰山。一说天宝年间,李白被逐出长安,到金陵游历时写下此诗。②三山:山名,在南京*江边。③一水:指秦淮河。

  1.颔联中的__________、_________,可以看出写的是六朝古都南京的历史遗迹。

  2.这首诗蕴含了诗人怎样的丰富的思想情感?

  八、材料作文

  阅读下面的文字,按要求作文。

  两人想体验登上泰山之巅的感觉,于是相约一起去爬泰山。第二天清晨,二人便来到泰山脚下,开始一天的旅程。走到半山腰时,其中一人由于体力不支便停了下来了,他对他的伙伴说:“你往上爬吧,我就在这等你。”夕阳落山时,伙伴下来了说道:“真遗憾,你没爬到山顶,你不知道站在泰山山顶是多么美妙。”半路停下来的人说:“真遗憾!你没在山腰逗留,你不知道泰山的景色多么迷人。”

  上面的材料引发了你怎样的联想和思考,请写一篇600字至800字的文章。

  (注意)①题目自拟,立意自定。②除诗歌外,文体自选。③不得抄袭、套作。④文中不得出现你所在学校的校名,以及教职工、同学和本人的真实姓名。

届九年级下语文试卷2

  一、选择题

  下列词语中加点字注音完全正确的一项是( )

  A、风骚(sāo) 睿智(ruì) 恻隐(cè) 越俎代庖(zǔ)

  B、诉讼(sòng) 狡黠(xiá) 腼腆(diǎn) 恃(sì)才放旷

  C、名讳(huì) 旁骛(wù) 佝偻(lǚ) 面面相觑(qǖ)

  D、矫(jiǎo)正 戏谑(nüè) 陨(yǔn)落 舐犊之情(shì)

  下列词语中没有错别字的一项是( )

  A、阔绰 恪尽职守 入坐针毡 怒不可竭

  B、沉缅 仓海桑田 周到如砥 孜孜不倦

  C、赈灾 吹毛求疵 莫衷一是 粗制烂造

  D、臆测 重蹈覆辙 顶礼膜拜 万恶不赦

  下列语句中加点的成语使用正确的一项是( )

  A、妈妈回家后把买回来的鱼称了又称,才发现她买的鱼被偷工减料了,足足少了半斤。

  B、我更高兴的更安慰的是:多少过分的谀词与夸奖,都没有使你丧失自知之明,众人的掌声、拥抱,名流的赞美,都没有减少你对艺术的谦卑!

  C、这个演讲装腔作势,貌似很有权威,实则东拉西扯,内容空洞,没有中心,令听众莫衷一是。

  D、面对伟大祖国70年的辉煌成就,40年的改革巨变,每一个*人无不为之长吁短叹,倍感骄傲与自豪!

  下列句子中没有语病的一项是( )

  A、“一带一路”这个战略构想,举世瞩目,影响深远,实现它是中华民族兴衰的关键。

  B、*年来,以“朗读者”为代表的一批文学文化类节目广受欢迎,正是因为它们引导人们放慢生活节奏,倾听内心的声音,顺应和满足了人们对精神生活的渴望。

  C、*军事动作片《战狼2》日前成功登顶*电影市场票房冠军,广泛引发国际社会关注。

  D、斑马线上之所以交通事故频发的原因,是由于驾驶员和行人的交通安全意识不强造成的。

  下列各句中,标点符号使用正确的一项是( )

  A、老王只有一只眼,另一只是“田螺眼”,瞎的。乘客不愿坐他的车,怕他看不清,撞了什么。

  B、“那就用不着白费工夫再上那儿去问了,”奥楚蔑洛夫说:“这是条野狗!用不着白费工夫说空话了。”

  C、网络技术对艺术传统的冲击不容忽视,对新艺术形式的催生已初露端倪。人们不得不思考,高科技的发展将导致艺术的沉沦?还是会迎来新时代的文艺复兴?

  D、*年来,随着经济的发展,城市的扩大,人口的猛增和生活质量的提高,城市垃圾不断增加,“城市垃圾处理”已成为环境保护的一大难题。

  依次填入下面横线处的句子,最恰当的一项是( )

  无论是读书还是写作,我们都在经历着一个前所未有的喧嚣时刻。 ①这些人的书和作品可以一读再读。

  ②能够保持一份清醒和独立,在读书中去伪求真,去芜存精,并不是一件容易的事。

  ③能够留下的,是鲁迅,是《红楼梦》,是《牡丹亭》《聊斋志异》,是雨果和陀思妥耶夫斯基……

  ④它们不会随着时光的流逝而变旧,它们犹如日出,每一次出现都是夺目的。

  ⑤我的枕畔,也曾有过名声显赫却难以卒读的书,但它们很快就从我的记忆中消失了。

  A、③①⑤④② B、②⑤③①④ C、④①②③⑤ D、②⑤④①③

  下列说法有误的一项是( )

  A、《儒林外史》是我国清代一部长篇讽刺小说,主要描写封建社会后期知识分子及官绅的活动和精神面貌。作者吴敬梓,清代小说家。

  B、《出师表》的“表”,是古代的一种特殊文体。这篇表写于诸葛亮第一次出师伐魏之前。在表文中,诸葛亮既给后主刘禅提出了治国之策,也陈述了对先帝的“感激”之情和“兴复汉室”的决心。

  C、戏剧,是一种综合的舞台艺术,它把文学、表演、绘画、雕塑、音乐、舞蹈等多种艺术综合成为一种独立的艺术样式。可以按不同的标准分类,按照表现形式,可以分为现代剧、历史剧、神话剧、科学幻想剧、童话剧等。

  D、《孔乙己》是鲁迅先生自己最喜欢的一篇短篇小说,他笔下的“孔乙己”和吴敬梓笔下的“范进”都是受封建科举毒害的读书人。

  二、综合性学*

  阅读下面的材料,回答后面的问题。

  气候变暖打乱二十四节气

  材料一20xx年底,我国申报的“二十四节气”成功入选人类非物质文化遗产代表名录。作为*人通过观察太阳周年运动,认知一年中时令、气候、物候等方面的变化规律所形成的知识体系,二十四节气不仅指导着*的传统农业生产和人们的日常生活,而且是传统历法体系的重要组成部分,至今已沿用20xx多年。在国际气象界,这一时间认知体系被誉为“*的第五大发明”。

  二十四节气起源于我国古代的黄河流域。古人通过观察、记录等,将太阳周年运动轨迹划分为24等份,每一等份有15天,为一个“节气”。冬至是最早被制定出的一个节气,之后又陆续确定了其他二十三个节气。人们将其编成了一首节气歌,“春雨惊春清谷天,夏满芒夏暑相连,秋处露秋寒霜降,冬雪雪冬小大寒”,流传至今。

  材料二

  *些年,全球变暖趋势明显,对二十四节气反应的气候特征又带来哪些影响?*科学院大气物理研究所副研究员钱诚等人研究发现,在全球变暖大背景下,每个节气的增温幅度都十分显著,季节循环整体抬升,立夏越来越早,立冬越来越晚。同时,满足某个节气气温标准的天数增减也呈两级分化趋势。

  钱诚和其他研究人员基于全国549个气象站的气温记录,经过降噪处理,以1961年到1990年的30年*均值确定了每个节气日的阈值温度。所谓阈值温度,简单来说,就是30年间每个节气日的全国*均气温。从气温记录来看,最冷的节气是大寒,阈值温度为-3、51℃;最热的节气是大暑,阈值温度为23.59℃。

  1月5日起,进入小寒节气。小寒节气的阈值温度为-3、50℃,即全国*均气温达-3、50℃及以下就符合小寒节气的气候特征。通过对1961至20xx年气温记录的研究显示,符合小寒、大寒气候特征的天数越来越少。统计显示,在上世纪60年代,最多的一年有51天符合大寒气候特征。1987年后,天数减少趋势更加明显。20xx-2007年冬季是一个暖冬,当年冬天没有一天气温达到大寒阈值。1998至20xx年10年间*均的大寒天数为14、0天/年,而20世纪60年代为32、4天/年,减少了56、8%。

  大寒不寒,与之相反的则是大暑更热。分析显示,1997年以后,气温季节循环中最热的时段(22℃以上)明显抬升。1997年到20xx年,符合大暑气候特征的天数为年均36、1天,而在上世纪60年代则为年均20天,增加了81、4%。

  材料三专家表示,虽然二十四节气的时间已经固定;但从历史发展来看,二十四节气的内涵是动态的、不断丰富的。*现代以来,农民也会根据气温、降水、物候的变化不断赋予其新的内涵,动态地修改和完善与节气相关的农谚。从这一角度看,正是由于二十四节气动态变化的内涵,它依然是当前人们生产、生活的参考。

  1、下列说法有误的一项是( )

  A、在国际气象界,“二十四节气”被誉为“*的第五大发明”。

  B、在全球变暖大背景下,惊蛰、小满节气的增温幅度都十分显著,季节循环整体抬升,立夏变化不明显,但立冬越来越晚。

  C、所谓阈值温度,就是一段时间内每个节气日的全国*均气温。

  D、*现代以来,农民也会根据气温、降水、物候的变化不断动态地修改和完善与节气相关的农谚。

  2、根据材料二的图表我们可以得出的结论是( )

  A、*几十年来,满足大寒阈值气温的天数基本不变。

  B、*几十年来,满足大寒阈值气温的天数呈增加趋势。

  C、*几十年来,满足大寒阈值气温的天数呈减少趋势。

  D、*几十年来,满足大寒阈值气温的天数有的时段上升,有的时段下降。

  3、下列判断有误的一项是( )

  A、“古人通过观察、记录等,将太阳周年运动轨迹划分为24等份”一句中加点的“等”字体现了说明语言的准确性和严密性。

  B、材料二的小标题可以拟作“大寒不寒 大暑更热”。

  C、材料三说明的主要内容是“二十四节气对当前人们的生产、生活依然有很重要的参考价值”。

  D、“1998至20xx年10年间*均的大寒天数为14、0天/年,而20世纪60年代为32、4天/年,减少了56、8%”一句运用了列数字和打比方的说明方法。

  三、情景默写

  诗文名句默写填空

  (1)__________________________,只有香如故。 (陆游《卜算子·咏梅》

  (2)__________________________,朝廷之臣莫不畏王,四境之内莫不有求于王。(《邹忌讽齐王纳谏》)

  (3)范仲淹在《渔家傲·秋思》中,表示自己和征夫们想家却不甘无功而返的矛盾心理的句子是_______,________________。

  (4)《关雎》中抒发求之不得的忧思的句子_______________,________________。

  四、诗歌鉴赏

  诗歌鉴赏

  南乡子·登京口北固亭有怀

  辛弃疾

  何处望神州?满眼风光北固楼。千古兴亡多少事?悠悠。不尽长江滚滚流。

  年少万兜鍪,坐断东南战未休。天下英雄谁敌手?曹刘。生子当如孙仲谋。

  1、从内容和结构上看,“年少万兜鍪,坐断东南战未休”一句有什么作用?

  2、词句“天下英雄谁敌手?曹刘。生子当如孙仲谋”运用了哪两个典故?

  五、名著阅读

  (题文)名著阅读

  我的任务很艰巨,还因为他们在内心一定感到这种想法是公*的,也一定本来就意识到他们如处在我的地位,也一样会做我希望做的事——最后他们让步了,同意把事情交付公断。被选中的仲裁人是奥利弗先生和一位能干的律师。两位都与我的意见不谋而合。我实现了自己的主张。

  选段出自《________》,作者是______,“我实现了自己的主张”具体指的是________。

  六、综合性学*

  综合性学*

  1998年7月31日,经*批准设立广安市。为庆祝广安市建市二十周年,学校要制作一本《时间的痕迹——老广安的记忆》的册子,请你完成以下任务。

  【任务一】方言拾萃

  广安方言形象幽默,富有韵味。请从下表提供的四个方言词语中选择一个,说说它的妙处。

  广安方言

  含义

  妙处

  惊抓抓

  指一个人在面对其他人看来不足为奇的事时,表现出国语惊慌与害怕的样子。

  以尖叫和手突然张开来表现人的惊慌与害怕,形象而生动。

  ①灯儿晃

  无事闲逛。这里的灯是指旧时悬挂起来或手提的照明灯。

  选:____________

  表达效果:______________________

  ②撑花儿

  雨伞。撑,张开。

  ③搭扑趴

  摔跟头。搭,方言读dá,摔。

  ④巴心巴肠

  全心全意。巴,紧贴。

  【任务二】老物记忆

  同学们拍摄了下面一组广安“老玩意儿”的照片,请你从中选择一张,参考示例,给它配上一句优美的话。(要求:体现物件特征,用上修辞,句式不限)

  示例:摇椅——吱吱呀呀,唱着儿时的歌。

  “老玩意儿”照片:木梳 茶壶 折扇 灯笼

  ________________________________________________________________

  【任务三】旧事回顾

  编辑“老广安的故事”,需要采访在广安土生土长的李爷爷,请你以小记者的身份用电话联系他。你会怎么说?(校名、人名用“xx”代替)

  _________________________________________________________

  七、现代文阅读

  记叙文阅读

  窗前的树

  张抗抗

  ①我的窗前有一棵树。

  ②那是一棵高大的洋槐。树冠差不多可达六层的楼顶。粗壮的树干与三层的阳台相齐,碧绿而茂密的树叶部分正对着我的四楼的窗户。

  ③坐在我的书桌前,一树浓阴收入眼底。从春到秋,由晨至昏,任是着意的或是不经意抬头,终是满眼的赏心悦目。

  ④那树想必已生长了多年。我们还没有搬来的时候,它就站立在这里了。或许,我还没出生的时候,它就已成为一棵树了。就因为它的缘故,我们曾真心希望能拥有这个单元的一扇窗。后来果真如愿,我们从此天天享受着它的清凉与恬静,很是满足,很觉幸福。

  ⑤洋槐在春天,似乎比其它的树都沉稳些。杨与柳都已翠叶青青,它才爆出米粒般大的嫩芽,只星星点点的一层隐绿,悄悄然绝不喧哗。又过了些日子,忽然就挂满了一串串葡萄似的花苞,又如一只只浅绿色的蜻蜓缀满树枝——当它张开翅膀跃跃欲飞时,薄薄的羽翼在春日温和的云朵下染织成一片耀眼的银色。那个清晨你会被一阵来自梦中的花香唤醒,那香味甘甜淡雅、撩人心脾却又若有若无。你寻着这馥郁走上阳台,你的精神为之一振,你的眼前为之一亮,顿时整个世界都因此灿烂而壮丽:满满的一树雪白,袅袅低垂,如瀑布倾泻四溅。银珠般的花瓣在清风中微微飘荡,花气熏人,人也陶醉。

  ⑥夏日的洋槐,巍巍然郁郁葱葱,一派的生机勃发;骄阳下如华盖蔽日,烈焰下送来阵阵清风。夏日常有雨,暴雨如注时,偏爱久久站在窗前看我的槐树——它任凭狂风将树冠刮得东歪西倒,满树的绿叶呼号犹如一头发怒的雄狮,它翻滚,它旋转,它颤栗,它**。曾有好几次我以为它会被风暴折断,闪电与雷鸣照亮黑暗的瞬间,我窥见它的树干却始终岿然。大雨过后,它轻轻抖落身上的水珠,那一片片细碎光滑的叶子被雨水洗得发亮,饱含着水分,安详而*静。

  ⑦那个时刻我便为它幽幽地滋生出一种感动,自己的心似乎也变得干净而澄明。雨后清新的湿气萦绕书桌徘徊不去,我想这书桌会不会是用洋槐木做成的呢?否则为何它负载着沉重的思维却依然结实有力。

  ⑧洋槐伴我一春一夏的绿色,到秋天,艳阳在树顶涂出一抹金黄。不几日,窗前已被装点得金碧辉煌。秋风乍起,金色的槐树叶如雨纷纷飘落,我的思路便常常被树叶的沙沙声打断。我明白那是一种告别的方式。它们从不缠缠绵绵凄凄切切,只是痛痛快快利利索索地向我挥挥手,连头也不回。它们离开了槐树就好比清除了衰老抛去了陈旧,是一个必然,一种整合,一次更新。它们一日日稀疏凋零,安然地沉入泥土,把自己还原给自己。他们需要休养生息,一如我需要忘却所有的陈词滥调而寻找新的开始。所以凝望这棵斑驳而残缺的树,我并不怎么觉得感伤和悲凉——我知道它们明年还会再回来。

  ⑨冬天的洋槐便静静地沉默。它赤裸着全身一无遮挡,向我展示它的挺拔与骄傲。或许没人理会过它的存在,它活得孤独,却也活得自信,活得潇洒。寒流摇撼它时,它黑色的枝条俨然如乐队指挥庄严的手臂,指挥着风的合奏。树叶落尽以后,树叉间露出一只褐色的鸟窝,肥硕的喜鹊啄着树枝喳喳欢叫,几只麻雀飞来飞去到我的阳台上寻食,偶尔还有乌鸦的黑影匆匆掠过,时喜时悲地营造出一派生命的气氛,使我常常猜测着鸟们的语言,也许是在提醒着我什么。雪后的槐树一身素裹银光璀璨,在阳光还未及融化它时,真不知是雪如槐花,还是槐花如雪。

  ⑩四季的洋槐便如一幅幅不倦变幻的图画,镶入我窗口这巨大的画框。冬去春来,老槐衰而复荣、败而复兴,重新回来的是原来那棵老槐;可是,我知道它已不再是原来的那棵槐树了——它的每一片树叶、每一滴浆汁,都由新的细胞、新的物质构成。它是一棵新的老槐树。

  ⑾年复一年,我已同我的洋槐度过了六个春秋。在我的一生中,我与槐树无言相对的时间将超过所有的人。这段漫长又真实的日子,槐树与我无声的对话,便构成一种神秘的默契。

  1、结合文章内容,用简洁的语言概括老洋槐一年四季不同的特点。

  2、文章第⑦段开头部分写道:“那个时刻我便为它幽幽地滋生出一种感动。”“我”感动的原因是什么?

  3、品析文中划线语句。

  又过些日子,忽然就挂满了一串串葡萄似的花苞,又如一只只浅绿色的蜻蜓缀满树枝──当它张开翅膀跃跃欲飞时,薄薄的羽翼在春日温和的云朵下染织成一片耀眼的银色。

  4、作者描写老洋槐,采用了哪些艺术手法?

  议论文阅读

  传统文化:从“有意思”到“有意味”

  尹宁玮

  ①20xx年春节期间,以古诗词为主题的电视节目《*诗词大会》红遍大江南北。这档被誉为诗词界“饕餮盛宴”的节目宛如一股清流,让看腻了歌手选秀、明星综艺等娱乐节目的观众们耳目一新。“冷”知识在“热”背景中复活,深刻启示着我们: 。这档节目火爆异常,让人多多少少有些意外。实际上,老百姓对于古典诗词的喜爱,是一以贯之的,但需要用恰当的方式来进一步激活。诗词大会正是因为暗合了社会中本就潜藏着的了解传统文化的需求,将电视节目模式和诗词文化传播相结合,才成为了所谓的“荧屏清流”。

  ②从“关关雎鸠”的诗经泛起点点轻波涟漪,到“落月摇情满江树”“万紫千红总是春”的诗词惊艳、百花齐放,再到“江山代有才人出,各领风骚数百年”的迁客骚情,中华诗词文化绵延千年却始终熠熠生辉。特别是随着*年来文化环境和文化传统得以逐步修复,出现诗词热是自然而然的现象。可以说,古典诗词的“热”,源于文化传统固有的“美”。

  ③但也有人可能会疑惑:日常生活中我们已不使用文言文了,为什么还需要古典诗词?特别是在当前娱乐化和商业文化流行的语境下,应如何透过古诗词热的现象看待其本质,该怎样传承古诗词,让它富有当代性和生命力?这俨然已成为绕不开、躲不过的话题。

  ④必须指出,对待古诗词的功效,不能直接采取实用主义的、功利化的态度;倡导古诗词的阅读、朗诵,更不是简单粗暴的“崇古”“复古”。中华文化源远流长,积淀着中华民族最深层的精神和追求,代表着中华民族独特的精神标识。从这个意义上讲,古典诗词是*人的精神礼赞。守望“诗心”,守的是“诗”也是“心”,更是民族的“根”和“魂”,是实现*梦的强大自信源泉。

  ⑤当钢筋混凝土的厚重感逼迫着我们加快脚步奔跑,当《我是歌手》等综艺节目占据着屏幕太多的空间,我们更需要古典诗词的熏陶,需要它们在日常生活中的潜移默化。古典诗词凝聚了大量美好、文雅、高贵的文字,可以帮助我们克服对生命焦虑、不安、琐碎,让我们能看到“诗和远方”。尽管时空相隔,但古诗文中蕴含的情感是贴*当下的。倾听古人的声音,触及前辈的思想,感悟圣哲的意境,这正是诗词大会所传递的正能量。

  ⑥但优秀的精神文化仍需要良好的传承载体。《大学》有云:“苟日新,日日新,又日新。”文化的“年轻化”与文化发展的新社会环境、新创制模式紧密相关。*诗词大会的成功,证明了“曲高”的古诗词也能引来大众的“应和”,关键是能否找到符合大众文化追求、生活节奏和欣赏偏好的方式,顺应传统文化的传播规律,创新传播形式。

  ⑦因此,守望“诗心”,亦需做好传承与展望。传统诗词穿越时代而仍有浸润心灵、启迪人心的力量,不仅意味着需要让传统文化与现代生活更好对接,更意味着传播方式需要随着时代变迁而推陈出新,创造性地使之转为现代型,从而实现传统文化从“有意思”到“有意味”的文化认同升级。

  1、结合文意,在文章第①段画线处填上恰当的一句话。

  2、概括本文的中心论点。

  3、文章第⑥段运用了什么论证方法?主要论述了什么?

  4、文中的“有意思”和“有意味”分别指什么?请简要概括。

  八、文言文阅读

  文言文阅读

  【甲】陈胜者,阳城人也,字涉。吴广者,阳夏人也,字叔。陈涉少时,尝与人佣耕,辍耕之垄上,怅恨久之,曰:“苟富贵,无相忘。”佣者笑而应曰:“若为佣耕,何富贵也?”陈涉太息曰:“嗟乎!燕雀安知鸿鹄之志哉!”

  二世元年七月,发闾左適戍渔阳,九百人屯大泽乡。陈胜、吴广皆次当行,为屯长。会天大雨,道不通,度已失期。失期,法皆斩。陈胜、吴广乃谋曰:“今亡亦死,举大计亦死;等死,死国可乎?”陈胜曰:“天下苦秦久矣。吾闻二世少子也,不当立,当立者乃公子扶苏。扶苏以数谏故,上使外将兵。今或闻无罪,二世杀之。百姓多闻其贤,未知其死也。项燕为楚将,数有功,爱士卒,楚人怜之。或以为死,或以为亡。今诚以吾众诈自称公子扶苏、项燕,为天下唱,宜多应者。”吴广以为然。

  【乙】初,钜鹿张角自称“大贤良师”,奉事黄老道畜养弟子跪拜首过符水咒说以疗病;病者颇愈,百姓信向之。角因遣弟子八人使于四方,以善道教化天下,转相诳惑。十余年间,众徒数十万,连结郡国;自青、徐、幽、冀、荆、杨、兗、豫八州之人,莫不毕应。遂置三十六“方”。“方”犹将军号也。大“方”万余人,小“方”六七千,各立渠帅。讹言“苍天已死,黄天当立,岁在甲子,天下大吉”。以白土书京城寺门及州郡官府,皆作“甲子”字。

  中*元年,大方马元义等先收荆、杨数万人,期会发于邺。元义数往来京师,以中常侍封谞、徐奉等为内应,约以三月五日内外俱起。未及作乱,而张角弟子济南唐周上书告之,于是车裂元义于洛阳。灵帝以周章下三公、司隶,使鉤盾令周斌将三府掾属,案验宫省直卫及百姓有事角道者,诛杀千余人;推考冀州,逐捕角等。角等知事已露,晨夜驰敕诸方,一时俱起。皆著黄巾为标帜,时人谓之“黄巾”,亦名为“蛾贼”。**以祠天。角称“天公将军”,角弟宝称“地公将军”,宝弟梁称“人公将军”。所在燔烧官府,劫略聚邑;州郡失据,长吏多逃亡。旬日之间,天下响应,京师震动。

  1、解释下列句中加点的词。

  (1)若为佣耕(________)

  (2)期会发于邺(_________)

  (3)度已失期(__________)

  (4)未及作乱(___________)

  2、下列各组加点词意义和用法相同的一项是( )

  A、项燕为楚将,数有功/众徒数十万

  B、百姓信向之/辍耕之垄上

  C、以中常侍封谞、徐奉等为内应/等死,死国可乎

  D、今有人于此/车裂元义于洛阳

  3、用“/”给文中句子断句,划出三处。

  奉 事 黄 老 道 畜 养 弟 子 跪 拜 首 过 符 水 咒 说 以 疗 病

  4、用现代汉语说说下面句子的意思。

  (1)扶苏以数谏故,上使外将兵。

  (2)角因遣弟子八人使于四方,以善道教化天下。

  5、甲文陈胜对当时形势进行分析后,提出了“_________”的策略口号;乙文黄巾军起义时提出的口号是“_____________________________。”

  6、结合文章内容,简要谈谈陈胜、张角在起义前有什么相似的做法。

  九、材料作文

  阅读下面的文字,根据要求写作文。

  印度哲学大师奥修说:“玫瑰就是玫瑰,莲花就是莲花,只要去看,不要比较。”一味的比较最容易动摇我们的心志,改变我们的初衷。而比较的结果,使人不是自卑,就是自傲,总之是流于*庸。也有人说:“没有比较就没有鉴别。”只有通过比较,才能明确自己的优劣高低,确定自己的奋斗目标。

  读完上述材料,你有什么感悟?写一篇不少于600字的文章。

  要求:角度自选,立意自定,题目自拟,文体自选(除诗歌外)。

届九年级下语文试卷3

  一、选择题

  下面语段中加点字的拼音正确的一项是( )

  每当瑞雪初霁,站在宝石山上向南眺望,白堤横亘,西湖银装素裹。断桥石桥拱面的冰雪已消融,桥的两端却还覆盖在皑皑白雪下。依稀可辨的石桥身似隐似现,而涵洞中的白雪熠熠生光,这与桥面的灰褐色形成反差,远望去似断非断,故称断桥。伫立桥头,放眼四望,远山*水,尽收眼底。

  A. qì gèn kái chù B. jì gèn ái zhù

  C. jì yuán ái zhù D. qì yuán kái chù

  下列各句中,没有别字的一项是( )

  A. 乡村振兴面临的最大问题并非资金,而是人才。既便如此,招引人才也要使尽浑身解数。

  B. “有爱就要说出来。”虽说老生长谈,但却很少有人能真正做到,尤其是对自己的父母。

  C. 针对*兵马俑访美拇指被盗一事,我们与美方展览承办单位进行了严正交涉,对这种损坏文化遗产的破坏和偷窃行为表示强烈愤概和谴责。

  D. 在实现梦想的过程中,如果我们没有莫言和村上春树那样心无旁骛而默默坚持数十载的毅力,那么就选择做一个踏踏实实的普通人吧。

  下列句子中加点的词语使用恰当的一项是( )

  A. 有自知之明的人善于对他人作出正确的评价,能明辨是非,知道如何针对队友的薄弱之处,及时提出弥补的措施。

  B. *和谐号真空超级高铁即将入海,一条开往法国科幻小说家凡尔纳笔下“海底两万里”的奇幻之旅即将开动。

  C. 去年暑假前后,我国多地出现抢购买房的现象,以致一些不法商人乘机哄抬价格,许多地区出台了一系列的政策,坚决打击不法行为,稳定房价。

  D. 一看到那两个盗窃国家文物、毁坏国家文物的强盗,他不由地意气风发,挣脱开众人的胳膊,抬起脚踹了那两人几脚。

  下列各句中,没有语病的一项是( )

  A. 春节期间,为了防止酒驾事件不再发生,杭州市各级执法部门投入了较多的警力,加大了巡查整治的力度,取得了显著的成绩。

  B. 20xx年9月4日-16日在浙江杭州举行的第13届全国*会,是将全国大中*会合并后举办的全国性首次*会,也是迄今为止浙江省承办的最大规模体育运动会。

  C. 水生态治理是一项长期而艰巨的任务,我们要树立以生态修复、循环利用为核心的科学治水观念,加强区域间联手、联动、联治的理念是不容置疑的。

  D. 英国首相特雷莎·梅说:“未来让我感到激动,不仅因为未来会有更多中英贸易往来,更是因为两国可以强强联合,创造更辉煌的成功。”

  二、句子默写

  古诗文名句填空。

  (1)__________,鬓微霜,又何妨!(苏轼《江城子·密州出猎》)

  (2)了却君王天下事,__________。(辛弃疾《破阵子》)

  (3)__________,舍鱼而取熊掌者也。(《鱼我所欲也》)

  (4)__________,出则无敌国外患者,__________,然后知生于忧患而死于安乐也。(《孟子》两章)

  (5)“__________,________________。”黄金台相传是战国时燕昭王所筑,他渴望贤才,置千金于台上,延请天下奇才。诗人借用这个典故,既与战争的地点相吻合,又是对天子和守边的将士浴血奋战、视死如归的英雄气魄的称颂。

  (6)英雄是什么?英雄就是文天祥在《过零丁洋》中的“人生自古谁无死,留取丹心照汗青”的大义凛然;英雄就是诸葛亮在《出师表》中“__________,__________”的临危受命。

  三、名著阅读

  文学常识与名著阅读。

  (1)阅读下列语段,在横线上填入恰当的内容。

  英国哲学家培根说:“读书足以怡情,足以傅彩,足以长才。”读西汉史学家司马迁所写的《史记》,我们会被其中 (填写人物)“王侯将相宁有种乎”的石破天惊的发问而震撼;读 (填国籍)作家雨果的演说《纪念伏尔泰逝世一百周年的演说》,我们清晰听到了雨果对人类社会永远需要的正义的呐喊;读俄罗斯田园诗人 (填人名)的《夜》,我们感受到夜的静谧与美好,窥见内心的安适与宁静。

  (2)阅读《西游记》片段,回答问题。

  金箍棒,浑铁棍,变脸不以朋友论。那个说:“正怪你这猢狲害子情!”这个说:“你令郎已得道休嗔恨!”那个说:“你无知怎敢上我门?”这个说:“我有因特地来相问。”

  ①上面这段文字中的“这个”和“那个”分别是指谁?

  ②“猢狲害子情”具体指什么事?为什么说“令郎已得道”?“有因特地来相问”的“因”又是指什么?

  四、语言应用

  根据以下材料,请你以环卫工人的名义,给材料中的学生所在的学校写一封120字左右的表扬信(只写正文)。

  20xx年1月25日,杭州三名中学生在家吃晚饭时看到新闻报道环卫工人辛苦清扫积雪。他们就自发组织,在家熬了暖暖的姜汤送给夜晚工作的环卫工人们。

  五、现代文阅读

  阅读下面的文章,完成下列小题。

  《洞》

  我叫马达赫。你别听错了,不是马大哈,我很聪明。

  晚上,我和老婆下班回家一看,糟了:客厅里那台液晶大屏幕彩电不见了,房里的手提电脑,还有那只二百斤重的保险箱全都不翼而飞了!……厨房的墙上有一个大洞!墙的那边是404。

  警察找人了解情况。我住403,对门402住着刘伯,刘伯说,上午九点多钟,来了几个民工。开门进了404,之后就从里面传来了巨大的砸墙声,他被吵得受不了,就过去敲门,那伙人开了半扇门跟他说话,这时墙上已砸开了一个小洞,刘伯看见了,问他们:“你们这是干什么?”他们说:“装修。”

  我问刘伯:“他们在我的墙上打洞,您不觉得有问题吗?”

  刘伯说:“那是你们两家的墙,关我什么事啊?”404的对门是401,里面住着王婶,王婶告诉警察说,她上街买菜,回来的时候正好碰上那伙人往外搬东西:彩电,电脑,保险箱……

  我老婆问她:“那套房子里没人住,你不知道吗?你看见他们搬那么多东西出来,不觉得奇怪吗?”

  王婶笑着说:“是啊,我也觉得奇怪,可这关我什么事呢?人家愿意买套房子当仓库,我管得着吗?”

  还有一个王阿毛,在这里设点收破烂,每天这个时候看着人来人往,据他说,快十点钟的时候,楼道的大门口停了辆车,有人在往车上搬东西,他问是谁家搬家?他们中有人随口答道:“403。”他当时就觉得不对:搬家都要处理很多破烂,怎么没叫他去收?他认识我,看见我们上班去了,他想:怎么搬家也不留人在家里?

  “这么说,你已经发现问题了。”警察问他,“可你为什么不报警呢?”

  他说:“这关我什么事啊?小区有保安,保安要查的。”

  保安又是怎么让那一车东西在他的眼皮底下出去的呢?这事谁也搞不清楚,因为小区有好几个出口,那些保安个个都说与他无关。

  我们回到家里,望着那个墙洞叹气,老婆说:“他们全都说对了,东西不是他们的,全与他们无关,只与我们有关,我们活该倒霉!”

  这时,我想起了几个钟头前的一幕。

  早上九点钟,我们下楼去上班,在楼道的大门口,见防盗门外站着几个陌生男人,我开了门,他们就乘机一拥而入。

  他们是干什么的?早些时候,附*不断有人家被盗。小区管理处提醒住户,要提高防范意识,看到有陌生人进入楼道,要仔细盘问,发现异常情况要立即报告。我叫住了他们,问:“喂,你们是干什么的?”

  他们中的一个说:“搞装修的。”

  “谁家装修?怎么不找房主开门?”

  那人说:“404。房主不在。”

  404?奇怪了,那套房子一直没有卖出去,昨天房产商还带人来看了房子呢!怎么今天就来装修了?我觉得不对,应该报告管理处,上了公共汽车,我就掏出手机来打电话,打一次,那边正在通话,再打,还是不通。这时老婆说:“算了,别打了,省几个电话费。就算他们是贼,关我们什么事呢?他们偷谁家也不能偷我们家。我们家的防盗门很坚固,谁也撬不开。我们家的防盗锁很先进,谁也打不开。就算打开了也不要紧,门锁连着自动报警装置呢,贼一进门,警察马上就知道了。你怕什么?”

  我笑了,是啊,我怕什么?进来的是不是贼,关我什么事呢?

  那时,我也是这么想的……

  1.小说中“我”的聪明体现在哪里?请分点简要回答。

  2.指出下列句子中加点词语所反映的人物心理。

  (1)王婶笑着说:“是啊,我也觉得奇怪,可这关我什么事呢?人家愿意买套房子当仓库,我管得着吗?”

  (2)我笑了,是啊,我怕什么?进来的是不是贼,关我什么事呢?

  3.小说中多次出现“关我什么事”,有什么作用?请简要分析。

  4.小说标题“洞”含义深刻,意蕴丰富,请结合全文陈述你的观点并联系生活实际谈谈你的想法。

  阅读下面的文章,完成下列小题。

  ①*期发布的一份“无现金趋势报告”称,现金社会的发展已经到了顶点,在不久的将来,包括*在内的许多国家都将进入“无现金社会”(事实上,“无现金生活”并不是指整个社会完全没有现金,而是指一种以电子支付为主的经济模式,全社会现金使用率极低,人们可以无障碍地使用电子支付方式进行消费。)。传统的现金支付不再跟得上社会发展的速度。要知道,每一次现金收付*均需要26秒钟,每一张纸币*均带有18万个细菌,每生产15000张纸币要砍掉一棵树龄20年的树。为了成功实现《巴黎气候变化协定》中人类在20xx年逐渐将碳足迹降至接*零的目标,人类需要一种全新的生活方式,“无现金”的生活就是其中之一。

  ②“92一代”是*目前最早完成“无现金生活”的社会群体,在他们看来,问题已经不是要不要现金,而是怎样在“无现金”的世界里提升自己的信用等级。蚂蚁金服的免押金信用服务已经覆盖全国381座城市。有专家预测,在未来的“无现金生活”中,信用将会取代押金,而消灭现金支付的不是支付公司,而是信用。凭借芝麻信用的积分,人们可以免押金直接租借雨伞、充电宝或租房、租车、住酒店等。也许你觉得这些都很超前,其实,全国*两千万人已经用过各种免押金信用服务。不出几年,*的大部分城市将成为信用城市。谁能够累积更好的信用,谁就能更好地在“无现金社会”获得更便利舒适的生活体验。

  ③杜莉就是一个芝麻信用的“高级玩家”。1993年出生的她,已经是杭州某互联网公司的品牌经理。作为支付宝的资深用户,杜莉意识到她的每一笔无现金支付,都是一次信用积累。杜莉现在的日常生活已经完全处于无现金的状态。每天早上,杜莉来到小区门口,这里已经整齐地停好了一排共享单车。她用手机一扫就能骑着车去上班。上班路上,顺便用支付宝在卖早点的李大妈那里买一份营养早餐。中午,杜莉和同事一起去吃饭时,连钱包也不用带。吃完饭,再也不用问半天谁有零钱,大伙儿直接用手机AA制结账,方便利落。

  ④其实,早在20xx年2月,支付宝就号称要用5年时间把全*推进到“无现金社会”。面对支付宝的“无现金移动支付体验活动”,另一家移动支付巨头微信支付也立刻跟进,推出直接对标的“鼓励金”活动。在互联网大并购时代之后,这样“薅(hāo)羊毛”的机会对于老百姓来说已经越来越少了。当然,有鼓励就难免带来歧视,拒收现金便是例子。

  ⑤商家拒收现金的新闻其实早已见诸报端,只是未受重视。随着这几年虚拟经济与实体经济的融合,阿里巴巴提出“新零售”,要用技术重构线下零售形态,重构超市,于是办起了盒马鲜生会员店。这种不收现金,只能通过APP付款的超市新业态,却在上海等地被碰壁老人屡屡投诉。很多老人别说使用盒马鲜生的App了,连智能手机都不会用。他们或与子女同行购物,或付现金给其他顾客,再由其他顾客帮助用盒马鲜生App结账付款。

  ⑥目前,虽然*绝大多数人手中都有一部智能手机,但这个“绝大多数人”主要指的是*的劳动人口,还有占总人口数35%的接*5亿的老人和小孩,他们很难享受到移动支付的便利。

  ⑦除了“不方便”,移动支付常年的支付补贴也让不会使用它的人在“经济上”处于不*等地位。以电影票为例,线上购票渠道的价格受补贴影响一直大幅度低于线下售票渠道的价格。马云曾说,20xx年打车软件集体大战时,的士司机为了获得各打车软件提供的补贴优惠,只接通过软件打车的乘客,这导致像他母亲这样的老年人的利益受到很大损害,连基本的打车服务都享受不到。

  ⑧20xx年8月,作为*移动支付行业领先者的蚂蚁金服CEO井贤栋率先呼吁:“无论是使用现金,还是银行卡或电子钱包的消费者,都应被提供他们最需要的服务。我们提倡给世界带来更多的*等机会,也包括每个人在支付选择上的*等。我们一直相信,技术的进步不会剥夺这个世界的温暖。”

  (选自《新周刊》总第490期)

  1.请概括本文的主要观点。

  2.阅读下列句子,结合文意,回答括号中的问题。

  (1)在互联网大并购时代之后,这样“薅(hāo)羊毛”的机会对于老百姓来说已经越来越少了。(说说句中加点词语的表达效果。)

  (2)这导致像他母亲这样的老年人的利益受到很大损害,连基本的打车服务都享受不到。(结合上下文,说说“这”在文中的含义。)

  3.联系上下文,指出第③段所用的论证方法并分析其作用。

  4.你怎么看待“无现金生活”?请结合全文并联系你的生活体验作具体分析。

  六、文言文阅读

  阅读下面的文言文,完成下列小题。

  古称操舟者为“长年”。王长年自少有胆勇,渔海上。

  嘉靖己未,倭①薄会城②大掠,长年为贼得,挟入舟。同执者男妇十余人,财物珍奇甚众。

  长年既执,时时阳为好语媚贼,酋长③亲信之;又业已入舟,则尽解诸执者缚,不为防。长年乘间谓同执者曰:“若等思归乎?能从吾计,且与若归。”皆泣曰:“幸甚!计安出?”长年曰:“贼舟还,将抵国,不吾备。今幸东北风利,诚能醉贼,夺其刀,尽杀之。”皆曰:“善!”会舟夜碇④海中,令诸妇女劝贼酒。贼度*家,喜甚。饮大醉,卧相枕藉。妇人收其刀以出。长年手巨斧余人执刀尽斫五十余贼断缆发舟行抵岸。长年既尽割贼级,因私剜其舌,另藏之,挟金帛,并诸男妇登岸。

  将归,官军见之,尽夺其级与金。长年秃而黄须,类夷人⑤,并缚诣镇将所,妄言捕得贼。镇将大喜,将斩长年,并上功。镇将,故州人⑥也。长年急,乃作乡语,历言杀贼奔归状。唶⑦曰:“若言斩贼级,岂有验乎?”长年探怀中藏舌示之。镇将验贼首,皆无舌。诸军乃大骇服。用长年为裨将,谢不欲。则赐酒,鼓吹乘马,绕示诸营三日,予金帛遣归。长年今尚在,老矣,益秃,贫甚,犹操渔舟。

  (选自朱国桢《涌幢小品》,有删改)

  (注释)①倭:指倭寇,古代对日本的称呼。此指十四至十六世纪劫掠我国江浙一带和朝鲜沿海的日本海盗集团。②会城:指福建省会福州。③酋长:倭寇的头目。④碇dìng:系船的石墩。这里指抛锚休息。⑤类夷人:相貌类似倭寇。⑥故州人:原本是同州人,指同乡。⑦唶zè:大喊。

  1.下列句子中加点词语意思相同的一项是( )

  A. 长年为贼得 必为有窃疾矣

  B. 贼度*家 度已失期

  C. 妇人收其刀以出 徒以有先生也

  D. 用长年为裨将,谢不欲? 秦王色挠长跪而谢之

  2.对文中画波浪线的部分断句恰当的一项是( )

  A. 长年手巨斧/余人执刀/尽斫五十余/贼断缆发/舟行抵岸。

  B. 长年手巨斧/余人执刀/尽斫五十余贼/断缆发舟/行抵岸。

  C. 长年手巨斧余人/执刀尽斫/五十余贼/断缆发舟行/抵岸。

  D. 长年手巨斧余人/执刀尽斫/五十余/贼断缆发舟/行抵岸。

  3.用现代汉语写出下列句子的意思。

  ①若等思归乎?能从吾计,且与若归。

  ②唶曰:“若言斩贼级,岂有验乎?”

  4.王长年是一个怎样的人?请结合文意简要分析。

  七、诗歌鉴赏

  (题文)阅读下面这首词,完成下列小题。

  减字木兰花竞渡

  (宋)黄裳

  红旗高举,飞出深深杨柳渚。

  鼓击春雷,直破烟波远远回。

  欢声震地,惊退万人争战气。

  金碧楼西,衔得锦标第一归。

  1.(小题1)元宵节、端午节、中秋节、重阳节等都是*的传统节日,这首词描会的是????????? (填写节日名称)的场景。

  2.(小题2)有评论者认为词中“飞”改为“冲”更好。你认为有道理吗?请主说理由。

  八、材料作文

  阅读下面的文字,按要求作文。

  一个人若是没有热情,他将一事无成,而热情的基点正是责任心。——列夫·托尔斯泰

  尽得最大的责任,就是大快乐;尽得小的责任,就是小快乐。你若是躲,倒是自投苦海,永远不能解脱了。——梁启超

  请自拟题目,按以下要求作文:

  ①要有真情实感。②自定立意,自选文体(诗歌除外)。③不少于600字。④不得抄袭套作,不得出现真实的人名、校名和地名。


届高三语文试卷 (菁华3篇)(扩展2)

——届高三上学期期末考试语文试卷 (菁华3篇)

届高三上学期期末考试语文试卷1

  一、现代文阅读

  阅读下面的文字,完成下面各小题。

  我国书法理论诞生较早,始见于汉代。崔瑗所谓“观其法象,俯仰有仪”,蔡邕所谓“书肇于自然”的书法评论,可算是书法理论的萌芽。此后,“自然”二字出现频率极高,且在不同时段、不同理论家的认知中有着不同的内涵。

  汉代的书论中,“观物取象”意识普遍存在。在此观念下,当时的书论家不但关注汉字点画形状的书写方法,而且注重以自然物象来对应说明点画的形状与面貌。到了魏晋,书论对点画的描述更加微观细致。卫夫人《笔阵图》中提到“横如千里阵云”“点如高峰坠石”,以自然之形比附书法之形;王羲之《笔势论十二章》中的“屈脚之法,弯如角弓之张”,则揭示了书法点画形状与自然物象之间的相似性。这种从形状上把自然物象与书法点画紧密相连的理论,属于第一自然——“眼中自然”。在后世有关“永字八法”的讨论中,这种理论被进一步细化,但对自然内涵的拓展上并没有多大进展。用这一自然概念理解书法,具体可感,真实可信,但这种一一对应的关系毕竟有限。因为,它只能停留在书法具体点画外形的层面,一旦超出点画外形,进入到点画姿态以及整个字的造型与姿态问题时,这种以物对应的办法就陷入了尴尬。于是,书法理论的构建中就出现了“第二自然”,即“胸中自然”。

  “胸中自然”基于生命意识与书法审美。魏晋南北朝书论中用自然物象来喻说书家的个人风格,并以此来表达不同书家作品中流露出来的生命意象,如梁武帝说“王羲之书字势雄逸,如龙跳天门,虎卧凤阙”,“韦诞书如龙威虎振,剑拔弩张”。到了唐代,孙过庭《书谱》中讲到书法“同自然之妙有,非力运之能成”,这就是说,书法与自然的关系不是简单的视觉感官上的相似性,而是自然与人心妙合的产物,需要感受与体悟。

  怎样将胸中的“第二自然”自然而然地表达出来呢?这就进入了基于法道观念与心性表现的“第三自然”——“手中自然”。宋代以后,在尚意思潮的影响下,书法的主要追求已经不是妙悟自然,更不是眼观自然,而是心性的自然流露,以及表达方式上的“自然而然”。苏轼的书写状态“我书意造本无法,点画信手烦推求”,便是他本人真性情、真情感的流露。此类作品,字如其人,人如其字,一切都自然而然。自然而然即为自由自在,是人生的最高境界,也必然是书法的最高境界。

  历代书家正是在观察、体悟、回归自然的过程中,提炼了书法的点画样式,丰富了书法的生命意象,升华了书法的审美境界,终将书法内化为书家的人格修养和心性表达。

  (摘编自史忠*《*古代书论中的“自然”变迁》)

  1.下列关于原文内容的表述,不正确的一项是

  A. 汉代崔瑗的“观其法象,俯仰有仪”、蔡邕的“书肇于自然”的书法评论,可证明我国书法理论诞生很早,不会晚于汉代。

  B. “眼中自然”的理论虽然在自然内涵的拓展上没有很大进展,但影响了后世有关“永字八法”的讨论,并且被进一步细化了。

  C. 汉代的书论家已经普遍具备了“观物取象”的意识,比如,他们注重汉字点画形状的书写方法,且注重以自然物象来对应说明。

  D. 用“眼中自然”的理论理解书法,但毕竟有局限,因为它只能描述字的点画外形,很难描述整个字的造型与姿态。

  2.下列理解和分析,不符合原文意思的一项是

  A. 构建“胸中自然”的理论,目的是破解“眼中自然”的尴尬,也反映了书法理论从强调书法的“形”到强调“态”的转变。

  B. “胸中自然”强调书法的“态”,包含了从点画本身的姿态以及从点画到整字再到整篇所孕育的生命跃动,内涵更加丰富。

  C. 从魏晋南北朝到唐代,书法理论家们逐渐意识到了“胸中自然”所反映的生命意识,将它自然而然地表达出来则是在宋代以后。

  D. 孙过庭《书谱》认为,书法与自然的关系是自然与人心妙合的产物,需要感受和体悟,而不仅仅是视觉感官上具有相似性。

  3.根据原文内容,下列理解和分析不正确的一项是

  A. 我国书法理论诞生时就出现了“自然”地观念,不同时段和不同理论家都在丰富着这一观念,最终达到“自然而然”的最高境界。

  B. 梁武帝对书法的评论不仅包含点画本身的形态,还有从点画到整字再到整篇所孕育的生命跃动,因而能直接唤起人们的审美联想。

  C. 苏轼说“我书意造本无法,点画信手烦推求。”这表明在尚意思潮影响下,他的书法只有心性的自然流露,而全不在乎点画形状了。

  D. 历代书家对书法的认识由注重点画样式,到注重生命意象和审美境界,再到注重心性表达,最后终于将书法内化为书家的人格修养。

  阅读下面的文字,完成下面小题。

  *人祭祖的历史十分悠久。到目前为止,可以追溯到的最早的祭祀行为是由甲骨文中有关祖先祭祀的文字记载的,如“报”“又”“岁”等。这些祭祀性文字表明祭祀是商王朝为了祓禳避祸、祈福求吉而举行的。他们需要定期举行祭祀活动,虽然仅通过部分甲骨文文字无法了解当时祭祀的具体情况,但至少可以得知殷人已经开始祭祀他们的祖先,并将祖先视为可祈福求助的神灵。这和《说文解字》对“祭”的解释相吻合,“祭,祭祀也。从示,以手持肉”。祭字本意就是用手持肉,献给神灵,沟通神人关系。

  到了周代,祭祖礼仪更增加了稳固国家统治和维持社会稳定的政治含义。从社会结构上看,周代是典型的宗法社会。周王自称天子,是“大宗”,同姓诸侯尊其为大宗子。这种宗法关系直接体现在宗庙设置上。即通过国家礼制严格限定不同身份群体的庙数差异,彰显他们的社会等级差异。虽然周代创制的这种宗庙体制没有被后世延续,但祭祖的文化传统却经久不衰,成为后世维系家族人伦关系、巩固国家统治的重要凭据。礼有仪有义,仪是外在表现形式,义是内在价值意蕴,两者相互依存,不可分割。

  将孝道观念和祭祖礼仪完美结合起来的是汉代诸儒。可以说汉儒对*祭祖礼仪发展的历史贡献是重大的,他们找到了*人重祭的本源——孝,并进行了深度的理论阐发和身体实践,使祭礼最终获得了自己的灵魂支撑。自“罢黜百家,独尊儒术”之后,孝作为儒家主要的思想观念,成为汉王朝立国的重要思想根基,“导民以孝,则天下顺”。汉朝通过优待孝子、不孝之罪入刑律、举孝廉、颁布养老令、推行三年丧制、诵读《孝经》等方式,将孝文化提升到稳固国家政治统治的高度。同时,汉代统治者将孝道作为一种普世性价值观,着力推广到政治、经济、法律、教育等社会各个层面,并逐渐形成了汉代以家族伦理为核心,并逐步向社会伦理和政治伦理扩展的孝文化系统。

  汉代以后,*人的祭祖礼仪既有了规范的礼仪表现形式,又继续追寻着以孝为核心的价值取向,自此长期活跃在人们的节日礼仪生活中,但是在传统社会,礼被认为是区分社会等级身份的工具。根据“礼不下庶人”的古礼规定,庶民长期被排斥在国家礼仪制度之外。庶民即寻常百姓,他们是不拥有任何政治、文化特权的底层。所以,先秦至隋唐时期,祭祖礼仪长期是贵族独有的特权,庶民祭祖行为被排斥在国家礼制范畴之外,只能以民间*俗的方式延续着。这种两分的局面在宋代出现了转机,当时受科举制度推行引起的社会阶层结构的调整,使士庶阶层的严格界限被打破,祭祖礼仪逐渐变成不同阶层共享的文化权利。

  *祭祖礼仪发展过程中的一件大事是《家礼》的出现,它深刻地改变了*祭祖礼仪的历史发展轨迹和形态面貌。《家礼》是一代通儒朱熹编写的冠、婚、丧、祭四礼指南。他认为,祭礼问题不限于礼仪本身,也关系着家族形态、社会文化发展和国家基层管理等一系列重要问题。他建议人们不用过于拘泥《仪礼》《书仪》等繁琐的礼仪要求。为了保证祭礼的可操作性,他对传统祭礼仪节进行了大胆的细化、分割、提炼,最后按照祭祀时间、地点、人物、器物、仪节五个主要方面,制定了祭礼基本模式。这个模式也成为宋明以来*人举行祭祖礼仪的标准范式。

  (选自邵风丽《祭祖礼仪的文化传统》,有删改)

  1.下列关于原文内容的表述,不正确的一项是

  A. 迄今可追溯的*人最早的`祭祀行为就是祭祖,甲骨文中的祭祀性文字说明殷人通过祭祀祖先来避除灾祸、祈求庇佑。

  B. 汉代儒生对“孝”进行理论阐发,并且身体力行,将孝道观念与祭祖礼仪完美结合,使祭祖礼仪有了灵魂支撑。

  C. 随着儒家地位在汉朝的确立,孝成为了当时立国的重要思想基础,并慢慢发展为以家族伦理为核心、向社会和政治伦理扩展的孝文化系统。

  D. 在古代很长的一段时期里,普通百姓无法进行国家礼制范畴内的祭祖活动,这一现象在科举制推行之后才有了改观。

  2.下列理解和分析,不符合原文意思的一项是

  A. 商朝举行祭祀活动,向被他们看作神灵的祖先祈福求助,这与《说文解字》中对“祭”字的解释是一致的。

  B. 周代是宗法社会,不同身份群体的庙数有差异,直接体现为周天子自称“大宗”,同姓诸侯尊其为大宗子。

  C. 孝道在汉代能成为一种渗透到社会各方面的普世性价值观,与统治者在立法、选才、教育等方面的大力推广有关。

  D. 周人传承下来的祭祖文化有维系家族人伦关系、巩固国家统治的重大意义,朱熹对祭礼问题的看法与此有共通之处。

  3.根据原文内容,下列说法不正确的一项是

  A. 虽然*人的祭祖传统年代久远,但借助出土文物、各类文献等资料,我们有可能了解到商代祭祀的具体情况。

  B. 祭祖礼仪在周代有稳固国家统治和维持社会稳定的政治含义,这充分说明“礼”不仅是一种外在形式,也是一种价值内涵。

  C. 晋代李密《陈情表》中所说的“圣朝以孝治天下”与汉代“导民以孝,则天下顺”的思想都强调了孝在国家治理方面的重要性。

  D. 宋代以前的祭礼较为繁复,不具备可操作性,因而朱熹对其进行了大刀阔斧的改革,祭礼的基本模式才由此制定。

  阅读下面的文字,完成下题。

  感恩节的荆棘花束

  迎着十一月的寒风,推开街边一家花店大门的时候,珊德拉的情绪低落到了极点。一直以来,她都过着一帆风顺的惬意生活。但是今年,就在她怀孕4个月的时候,一场交通事故无情地夺走了她肚子里的小生命。紧接着,她的丈夫又失去了工作。这一连串的打击令她几乎要崩溃了。

  “感恩节?为什么感恩呢?为了那个不小心撞了我的粗心司机?还是为那个救了我一命却没能帮我保住孩子的气囊?”珊德拉困惑地想着,不知不觉中来到一团团鲜花面前。

  “我想订花……”珊德拉犹豫着说。

  “是感恩节用的吗?”店员问,“您一定想要那种能传递感激之情的花吧?”

  “不!”珊德拉脱口而出,“在过去的五个月里,我没有一件事是顺心的。”

  “我知道什么对您最合适了。”店员接过话来说。

  珊德拉大感惊讶。这时,花店的门铃响了起来。“嗨,芭芭拉,我这就去把您订的东西给您拿过来。”店员一边对进来的女士打着招呼,一边让珊德拉在此稍候,然后就走进了里面的一个小工作间里。没过多久,当她再次出来的时候,怀里抱满了一大堆的绿叶、蝴蝶结和一把又长又多刺的玫瑰花枝——那些玫瑰花枝被修得整整齐齐,只是上面一朵花也没有。

  珊德拉狐疑地看着这一切,这不是在开玩笑吧?谁会要没有花的枝子呢!她以为那顾客一定会很生气,然而,她错了。她清楚地听到那个叫芭芭拉的女人向店员道谢。

  “嗯,”珊德拉忍不住开口了,声音变得有点结结巴巴的,“那女士带着她的……嗯……她走了,却没拿花!”

  “是的,”店员说道,“我把花都给剪掉了。那就是我们店里的特别奉献,我把它叫做感恩节的荆棘花束。”

  “哦,得了吧,你不是要告诉我居然有人愿意花钱买这玩意吧?”珊德拉不理解地大声说道。

  “3年前,当芭芭拉走进我们花店的时候,感觉就跟你现在一样,认为生活中没有什么值得感恩的。”店员解释道,“当时,她父亲刚刚死于癌症,家族事业也正摇摇欲坠;儿子在吸毒,她自己又正面临一个大手术。我的丈夫也正好是在那一年去世的,”店员继续说道,“我一生当中头一回一个人过感恩节。我没有孩子,没有丈夫,没有家人,也没有钱去旅游。”

  “那你怎么办呢?”珊德拉问道。

  “我学会了为生命中的荆棘感恩。”店员沉静地答道,“我过去一直为生活当中美好的事物而感恩,却从没有问过为什么自己会得到那么多的好东西。但是,这次厄运降临的时候,我问了。我花了很长一段时间才明白,原来黑暗的日子也是非常重要的。”

  正在这时,又有人走了进来,是一个头顶光秃的矮个子胖男人。

  “我太太让我来取我们的感恩节的荆棘花束——12根带刺的长枝!”那个叫菲利的男人一边接过店员从冰箱里取出来的用纸巾包扎好的花枝,一边笑着说。

  “这是给您太太的?”珊德拉难以置信地问道,“如果您不介意的话,我想知道您太太为什么会想要这个东西。”

  “哦,不介意……我很高兴你这么问,”菲利回答道。“四年前,我和我太太差一点儿就离婚了。在结婚四十多年之后,我们的婚姻陷入了僵局。但是现在,我们总算把问题给解决了,我们又和好如初。这儿的店员告诉我,为了让自己牢记在荆棘花束里学到的功课,她总是摆着一瓶子的玫瑰花枝。这正合我意,因此就捎了些回家。”

  “我诚挚地向你推荐这一‘特别奉献’!”菲利一边付账,一边对珊德拉说。

  “我实在不知道我能不能够为我生命中的荆棘感恩。”珊德拉对店员说道,“这有点儿……不可思议。”

  “嗯,”店员小心翼翼地说,“我的经验告诉我,荆棘能够把玫瑰衬托得更加宝贵。人在遇到麻烦的时候,会更加珍视上帝的慈爱和帮助,我、芭芭拉,还有菲利夫妇,都是这么走过来的。因此,不要恼恨荆棘。”

  眼泪从珊德拉的面颊上滑落,她抛开她的怨恨,哽咽道:“我要买下这样一束带刺的花枝,该付多少钱?”

  “不要钱,你只要答应我把你内心的伤口治好就行了。这里所有顾客第一年的特别奉献都是由我免费赠送的。”店员微笑着递给珊德拉一张卡片。

  珊德拉打开卡片,上面写着:我的上帝啊,我曾无数次地为我生命中的玫瑰而感谢你,但却从来没有为我生命中的荆棘而感谢过你。请你教导我关于荆棘的价值,通过我的眼泪,帮助我看到那更加明亮的彩虹……

  眼泪再一次从珊德拉的脸颊上滑落。

  (有删改)

  1.下列对这篇小说相关内容与艺术特色的分析和鉴赏,最恰当的两项是 ( )

  A. 珊德拉遭遇了一场交通事故,失去了自己的孩子,她的丈夫也失去了工作。因此,她到花店买花,想要使自己的心情好起来。

  B. 不幸的遭遇使店员懂得了生命中“荆棘”的价值,于是她开始售卖荆棘花束。为招揽生意,她决定第一年向顾客免费赠送。

  C. “荆棘能够把玫瑰衬托得更加宝贵”,这朴实无华的话语,却道出店员对人生的深刻领悟,令人深思。文中类似的语言描写不止一处。

  D. 尽管店员一直向珊德拉推荐这种感恩节特别奉献的荆棘花束,芭芭拉和菲利夫妇的经历也证明了花束的意义,但珊德拉始终将信将疑。

  E. 卡片上的“玫瑰”与前文“生活当中美好的事物”遥相呼应,而“荆棘”则象征着生活中遇到的麻烦、挫折、不幸和苦难等等。

  2.小说以“感恩节的荆棘花束”为题,有什么作用?请简要分析。

  3.小说在刻画店员这个形象时,突出了她的哪些性格特征?请简要分析。

  4.小说的主人公,有人认为是珊德拉,有人认为是店员。你同意哪一种看法?请结合全文,谈谈你的看法。

  二、文言文阅读

  阅读下面文言文,完成下列小题。

  郭进,深州博野人。少贫贱,为钜鹿富家佣保。有膂力,倜傥任气,结豪侠,嗜酒蒲博。其家少年患之,欲图杀进,妇竺氏阴知其谋,以告进,遂走晋阳依汉祖。汉祖壮其材,留帐下。

  周广顺初,移淄州。二年,吏民诣观察使举留。是秋,迁登州刺史。会群盗攻劫居民进率镇兵*之部内清肃民吏千余人诣阙请立屏盗碑许之。显德初,移卫州。卫、赵、邢、洺间多亡命者,以汲郡依山带河,易为出没,伺间椎剽,吏捕之辄遁去,故累岁不能绝其党类。进备.知其情状,因设计发擿之,数月间剪灭无余,郡民又请立碑记其事。进尝于城四面植柳,壕中种荷芰,后益繁茂。郡民见之有垂涕者,曰:“此郭公所种也。”

  初,开宝中,太祖令有司造宅赐进,悉用筒瓦。有司言:旧制,非亲王公主之第不可用。帝怒曰:“进控扼西山十余年,使我无北顾忧。我视进岂减儿女耶?亟.往督役,无妄言。”进有材干,轻财好施,然性喜杀,士卒小违令,必寘于死,居家御婢仆亦然。进在西山,太祖遣戍卒,必谕之曰:“汝辈谨奉法。我犹贷.汝,郭进杀汝矣。”其御下严毅若此。然能以权道任人,尝有军校自西山诣阙诬进者,太祖诘知其情状,谓左右曰:“彼有过畏罚,故诬进求免尔。”遣使送与进,令杀之。会并人入寇,进谓诬者曰:“汝敢论我,信有胆气。今舍汝罪,能掩杀并寇,即荐汝于朝;如败,可自投河东。”其人踊跃听命,果致克捷。进即以闻,乞迁其职。时田钦祚护石岭军,恣.为奸利诸不法事,进虽力不能禁,亦屡形于言。进武人,性刚烈,战功高,钦祚以他事侵之,心不能甘,自经死,年五十八,钦祚以暴卒闻。太宗悼惜久之,赠安国军节度,中使护葬。

  (选自《宋史·列传·卷三十二》,有删改)

  1.对下列句子中加点词语的解释,不正确的一项是

  A. 进备知其情状 备:准备

  B. 亟往督役 亟:急切、立刻

  C. 我犹贷汝 贷:宽恕

  D. 恣为奸利诸不法事 恣:肆意

  2.下列对文中画波浪线部分的断句,正确的一项是

  A. 会群盗攻劫/居民进率镇兵*之/部内清肃/民吏千余人诣阙/请立屏盗碑/许之

  B. 会群盗攻劫/居民进率镇兵*之/部内清肃民吏/千余人诣阙/请立屏盗碑/许之

  C. 会群盗攻劫居民/进率镇兵*之/部内清肃民吏/千余人诣阙/请立屏盗碑/许之

  D. 会群盗攻劫居民/进率镇兵*之/部内清肃/民吏千余人诣阙/请立屏盗碑/许之

  3.下列对原文有关内容的概括和分析,不正确的一项是

  A. 郭进倜傥任气,喜欢结交豪杰侠士。有人认为他是祸患,想谋划杀他。竺氏暗中知道了阴谋,把这事告诉了郭进。

  B. 郭进智勇双全,在做卫州官员的时候,带兵剿灭了依山傍河的汲郡亡命之徒,得到了百姓的高度赞美。

  C. 郭进性情残暴,无论是士卒还是家里的婢仆,稍有违背命令,郭进都会置之于死地,因此也被太祖批评。

  D. 郭进大度宽容,能用权谋驾驭他人。对于诬告自己的军校,他也能以国家利益为重,没有立刻处置他。

  4.把文中画横线的句子翻译成现代汉语。

  (1)进控扼西山十余年,使我无北顾忧。我视进岂减儿女耶?

  (2)其人踊跃听命,果致克捷。进即以闻,乞迁其职。

  三、诗歌鉴赏

  阅读下面两首唐诗,完成后面各题。

  独 鹤

  【唐】韦庄

  夕阳滩上立徘徊,红蓼【注】风前雪翅开。

  应为不知栖宿处,几回飞去又飞来。

  鹤

  【唐】褚载

  欲洗霜翎下涧边,却嫌菱刺污香泉。

  海鸥浦雁应惊讶,一举扶摇直上天。

  【注】蓼:草本植物,开淡红色或白色的花。

  1.下列对这首作品的理解与分析,不正确的两项是( )

  A. 两诗写鹤都采用了细节描写:前诗写鹤伫立、展翅的细小动作,后诗写鹤嫌菱刺而不洗濯的细微心理。

  B. 两诗写鹤都采用了烘托手法:前诗以夕阳、沙滩、红蓼为背景烘托,后诗以溪涧、蓝天为背景烘托。

  C. 两诗都采用虚实结合手法:两首诗中鹤的颜色、动作为实写;两首诗都着一“应”字表猜测,是虚写。

  D. 两诗都是托物言志的咏物诗:两诗吟咏鹤的形象,其实都融入了诗人内心的观照,寄托了诗人的志趣。

  E. 两首诗的表达技巧有所不同:前诗运用夸张手法,动静结合;后诗也用夸张手法,只有动态的描写。

  2.两首诗刻画的鹤这一形象所表达的思想情感有什么不同?

  四、情景默写

  补写出下列句子中的空缺部分。

  (1)在《逍遥游》开篇,庄子在写出了鹏不知有几千里长的背之后,又用形象生动的笔法写出了鹏“______________,____________”的壮观气势 。

  (2)《陈情表》中李密用“__________,_______”两句概括写出自己命运坎坷,早年遭遇不幸的人生经历。

  (3)《岳阳楼记》中有“是进亦忧,退亦忧”一句,“进亦忧”在文中指的是“___________”。

  五、选择题

  下列词语中,划线字的读音全都正确的一项是

  A. 针灸(jiǔ) 晕车(yùn) 露马脚(lòu) 曲尽其妙(qǔ)

  B. 桔梗(jié) 精髓(suǐ) 刽子手(kuài) 混水摸鱼(hún)

  C. 压缩(yā) 狡黠(xié) 拧脾气(nìng) 恬不知耻(tián)

  D. 择菜(zhái) 连累(lěi) 紧箍咒(gū) 咎由自取(jiù)

  下列各句中,没有语病的一项是

  A. 总部设在美国西雅图的亚马逊公司已经停止广受争议的维基解密网站提供网络服务器支持,维基解密只得再次将其网站托管给瑞士的一家服务商——班霍夫公司。

  B. 像“*达人秀”这类“零门槛”的选秀节目,让拥有才华和梦想的任何一个普通人,都可以展示天赋和潜能;也让我们懂得*凡人成就自我的关键在于相信梦想,相信奇迹。

  C. 梵高笔下的“向日葵”,不仅仅是植物,而是带有原始冲动和热情的生命体,是希望和自由的化身,让无数人的心灵为之震颤。

  D. 周瑜是一个文武双全的人物,没有他,三国格局都有可能改写,甚至也就没有赤壁之战。只可惜天妒英才,他仅仅活了三十六岁。

  下列各句中加点的成语使用恰当的一项是

  A. 一名外地客人在本市乘的士时遭劫,载他的“的哥”竟坐山观虎斗。昨日,乘客周先生说,他准备将这名“的哥”告上法庭。

  B. *汽车工业落后,没有自己的风格和拿出手的品牌,所以我们要学*外国的先进造车理念和先进的技术,因人成事,使*的汽车早日走向世界。

  C. 现代企业非常注重核心技术参数的保密工作,管理者甚至要求技术人员在关键时刻拿出剖腹藏珠的气概也要保守住自己的商业机密。

  D. 连续的暴雨,造成河道水位猛涨,昨天,省里来电,要求沿河各级部门防患未然。

  六、语言应用

  请用一句话概括下面消息的主要内容。(含标点不超过25个字)

  晨报讯 备受关注的*第一、世界第二高楼上海中心大厦目前已经完工,正在积极准备相关事宜,即将正式开业。最令游客向往的可直达119层观光*台的快速电梯已安装完毕,其速度可达每秒18米,55秒可抵达上海中心119层观光*台。

  上海中心可抵达的最高点为125—126层,设置有风阻尼器。这个神秘的空间内,可以举办一些小型音乐演奏会、艺术展示等。工作人员告诉记者,今后要抵达118层、119层的观光层,必须乘坐时速达每秒18米的超高速观光电梯。如果下行,每秒速度达10米,70秒后可抵达一层。这样的超高速观光电梯一共有三台。

  上海中心大厦相关人员表示,上海中心大厦的观光*台将设置纪念品商店、空中邮局等,还酝酿增设世界超高层建筑史展览等内容。

  如今,广告已逐渐演变成一种文艺形式,有着独特的魅力。广告语常常运用某种修辞增加可读性、趣味性,从而更好地宣传自己的产品。请认真品读下面两则广告词,并说说它们的表达技巧好在哪里。

  ①千里江陵一日还。(江陵汽车) ②格力空调,冷静的选择。(格力空调)

  七、命题作文

  阅读下列文字,按要求作文。

  前几年,央视春晚上的一首歌曲《时间都去哪了》引发了国人的共鸣,有的人感慨“青春都去哪了?”有的人感慨“快乐都去哪了?”有的人感慨“幸福都去哪了?”……还有的人说,面对这些问题,我们要做的不仅仅是感慨。

  以上的材料可能有属于你自己的回忆,也许会引发你的思考和感悟。请以“ 都去哪了?”为题写一篇文章。

  要求:补充完整题目,立意自定,写一篇不少于800字的记叙文。

  注意:1. 符合文体特征;2. 书写工整。

届高三上学期期末考试语文试卷2

  一、现代文阅读

  阅读下面的文字,完成下列小题。

  两河流域的楔形文字、古埃及的圣书字和*的汉字,是世界上最古老的三大自源文字体系,但唯有汉字沿用至今。这不应该是侥幸,而是源自于汉字的特质。

  早期文字大都是象形、表意的,其表音符号很多都是衍生自象形表意符号。即便是汉字,占绝大比例的形声字的声符,也大多来自表意成分。在长期发展中,汉字表音的因素在汉字体系中大大增强,表现出明显的表音化倾向。但为什么汉字的表音化倾向停滞了?为什么表音的声符没有进一步简化、抽象化,像很多拼音文字那样发展成为纯粹表音的字母?汉字的生命力何在?

  若在汉字和汉语的关联中寻找原因,这首先是因为汉字的适用性。汉字能够很好地记录汉语,这是汉字不废的重要原因之一。汉语是词根语,不用改变词首、词尾或词中间的语音来表示语法关系。印欧语言很多“性”“数”“格”都要求有小而频繁的词形变化,这是方块汉字难以如实记录的。汉字作为语素文字,一个字表示一个意思,在语音上也没有什么变动,和汉语匹配十分得当。

  其次是汉字具有时空包容性。汉字的“包容性”是一种巨大的弹性,表现在时间和空间两个方面。

  一是时间包容性。孔子云:“三人行必有我师焉。”我们一看就懂,但如果当年有录音机录下老夫子的话,我们可能难以听懂。因为语音是不断变化的,春秋时代的语音系统和现代普通话的语音系统大不一样。然而,不管语音发生了何种变化,字仍然是这些字,意思始终不会改变,这就是时间的包容性。对于英语、法语或其他拼音文字,几百年前的作品现代人可能就看不懂了,因为语音的变化直接反映到了文字的拼写上。汉字承载了历史悠久、内容丰富且比较容易解读的文献,这也成为汉字不废的重要原因。

  二是空间包容性。同样一张《人民日报》,北京人、上海人、广州人都能毫无障碍地阅读,他们使用同一个汉字系统,字形和字义是一样的,但读音却各行其是。*疆域辽阔,方言繁杂,汉字可以起到统一沟通的作用。汉字及汉字文化的联系使中华民族成为统一的文化共同体。试想,如果历史上使用的就是拼音文字,拼音文字要反映实际语音,那么各地的拼音文字就会各不相同,随着历史的发展,文字的差异会越来越大,就像今天欧洲或印度,语言纷繁复杂。据统计,印度有1600多种语言,宪法规定的联邦官方语言就有18种。可以想见汉字的包容性,在幅员辽阔、人口众多的*,汉字起着何等重要的文化认同作用,产生了何等强大的凝聚力。

  最后是汉字具有文化积聚性。在几千年的历史中,汉字沉淀了大量的历史文化信息,成为历史的活化石,它反映了*早期的生产、商业、政治、战争、家庭、饮食等社会生活及意识形态等各个方面的情况,隐藏着一部古代社会的百科全书,可以作为历史考古的线索。例如买、卖、贵、贱、资、费、财、货、赚、赔等字,都有“贝”字,这反映了早期社会“贝”曾被作为原始货币的历史。

  正是由于上述原因,汉字已经成为中华文化的主要载体和象征,成为不可抛弃的文化遗产。在世界文字之林中,汉字展现出其科学性、艺术性,闪烁着独特的智慧光芒,是人类宝贵的非物质文化遗产。

  (节选自黄国营《*汉字的开放性与生命力》,有删改)

  1.下列关于原文内容的理解和分析,不正确的一项是

  A. 作为世界上最古老的自源文字体系之一,汉字是唯一至今仍在使用的文字,是中华文化的主要载体和象征。

  B. 汉字在发展中曾呈现出明显的表音化的倾向,表音的声符来自于表意成分,但没有进一步抽象化为纯粹表音的字母。

  C. 汉语是词根语,没有“性”“数”“格”等词形变化;汉字是语素文字,与不通过改变语音来表示语法关系的汉语十分匹配。

  D. 在幅员辽阔、人口众多的*,汉字发挥着重要的文化传承和认同作用,具有着强大的凝聚力和生命力。

  2.下列对原文论证的相关分析,不正确的一项是

  A. 文章开头指出汉字能够沿用至今,源于汉字的特质,汉字虽然也有表音化倾向,但这并不是汉字的生命力所在。

  B. 文章在汉字和汉语的关联中寻找汉字不废的原因,那就是汉字具有适用性、时空包容性和文化积聚性特点。

  C. 文章第5、6段分别从时间和空间角度阐述汉字的包容性,将汉字与拼音文字进行比较,是为了证明语音变化不会影响汉文化的传承。

  D. 文章论证上采用总分总结构,开头提出问题,从三方面分析汉字特质,最终得出汉字是人类宝贵的非物质文化遗产的结论。

  3.根据原文内容,下列说法不正确的一项是

  A. 如果汉字发展的表音化倾向没有停滞,发展成纯粹表音的字母,它也有可能像楔形文字、圣书字等一样失去沟通交流的作用。

  B. 虽然汉字的读音可能受时间和地域影响而产生很大的变化,但因字形和字义是确定的,所以不会影响人们的理解。

  C. 如果汉民族历史上使用的是拼音文字,那么各地的拼音文字就会受实际语音影响而各不相同,甚至可能会导致汉语演化成多种语言,而不是方言。

  D. 汉字承载了大量的历史文化信息,是古代社会的百科全书,是*早期社会生活历史考古非常重要的依据。

  阅读下面的文字,完成下列小题。

  老? 人

  崔 立

  ①早上,阳光缓缓地照射在马路上,老人佝偻的背影,缓步地前行中。微风从老人的身边轻轻滑过,都像比老人的速度要快。老人,是要从人行道的一端,走到人行道的另一端。有一个年轻人,急急地走过老人的身边。年轻人走过去时,带起了一阵风,也碰触了老人柔弱的肩膀一下,是风要刮倒了老人,还是年轻人撞到了老人,老人的身子踉跄地顺势往前仰了一下,像是要摔倒,摇摇晃晃地要倒下了,还是艰难地撑住了。匆匆而过的那个年轻人,在老人的眼睛里早已不见了踪影。

  ②又一天,老人走着同样的路,缓缓地,在那条人行道上蹒跚着步子,缓缓行走在人行道的中间。一个女孩,越走越*,很快已经到了老人的跟前。老人站在人行道的中间,挡住了女孩前行的路线,女孩几次想走过去,但空间太窄,还是走不过去。女孩咬了咬牙,想说什么,又憋了回去。反复犹豫,女孩终于是忍不住开了口。女孩说,阿姨,你能让我一下吗?有好几秒的停顿,老人缓缓转过了身,看向女孩。顺势地,女孩在老人转身时多出来的空档,匆匆忙忙地走了过去。女孩真的是太匆忙了,连一声“谢谢”也忘记说出口,人已走出去好远。

  ③……

  ④再一天,老人还在那里行走,缓慢地,继续缓慢地行走在这条路上。①天是有些冷的,哪怕阳光是高高挂着的,也难以抵挡冬日的寒冷。老人缓缓地在行走,不知不觉间,本来围得就有些松的围巾悄无声息地滑落。老人毫无察觉,②老人丝毫感觉不到因围巾掉落后的寒冷,老人本来就很冷,冷与再冷之间差异不大。身边响起一个女人的声音。不知什么时候,女人已经到了老人的身旁。女人喊了声:阿姨!老人听到了,老人是有几秒钟的停顿,老人缓缓地转过身,老人以为还是要让开路,然后有一个人会像阵风般匆匆忙忙地从她身边走过。这次,没有风,要是风,也是空气中的寒风。女人停在了老人面前,女人说,阿姨,这围巾,是您掉的吗?老人看到了女人手上的围巾,*惯性地摸了摸自己的脖子,脖子间空荡荡的。无疑,眼前的这条围巾是老人掉的。老人从女人手上接过了围巾,说,谢谢。老人轻轻地摸索着往脖子里围,老人围得很艰难。女人说,阿姨,我帮您围吧。女人为老人围上了围巾,围得很严实,让那寒风吹不进脖子里。女人还说,阿姨,您是要往前面走吗?我送您过去吧。老人脸上挂起笑容,说,谢谢你。女人扶着老人缓慢地往前走,③天还是冷,冷中却跃动着一丝暖意。是春天快要到来的暖意。

  ⑤这是电视台做的一个节目,老人是电视台请的一位演员,表演得很到位。演员在这条短短的人行道上行走了一个星期,没有人怀疑她是假的老人,也没有人关注过老人。女人是这个星期最后一天出现的。电视台用了一个星期的时间,找到了愿意帮助老人的女人。

  ⑥这个节目很快就在电视台播出了。

  ⑦同时播出的,是*期发生的一个诈骗故事:一个未经证实的儿童募捐,竟引发了全市高达数十万的捐款,若不是**的及时介入,捐款数额还在不断地增长。为什么我们市民的防范意识是如此的薄弱,为什么会有那么多人发出善心善念踊跃捐款呢?如果需要捐款的不是孩子,是老人呢?

  ⑧两个故事放在了一起播放,是刻意,还是无心之举?

  ⑨这个节目什么都没评论,只是原生态地播出了。

  ⑩节目播出的第二天,这个城市一下子多了许多主动帮扶老人的年轻志愿者们。

  (选自《人民日报·海外版》20xx年9月9日,第11版,有删节)

  1.下列对小说相关内容和艺术特色的分析鉴赏,不正确的一项是

  A. 小说中的很多语句看似*淡,内涵却很丰富。如“这次,没有风,要是风,也是空气中的寒风”,暗示女人没有匆匆忙忙走过老人身边。

  B. 小说成功地运用了对比手法,比如,年轻人、女孩与女人的做法形成了对比,再如,电视台两个节目的内容也形成了对比。

  C. 小说注重于细微处写人,老人“缓慢地前行”、“踉跄地顺势往前仰”、“缓缓地转过身”、系围巾“围得很艰难”等动作,表现了扮演老人的演员演技的高超。

  D. 发生在老人身上的故事是电视台的节目策划,是小说出人意料之处,但其中也蕴含着作者对当下社会道徳和人际关系的忧虑与反思,这也是小说的深刻之处。

  2.小说第④段中三次写到“冷”,分别表现了什么意图?请结合小说内容具体分析。

  3.小说结尾“节目播出的第二天,这个城市一下子多了许多主动帮扶老人的年轻志愿者们”的安排是否必要?结合文章内容,谈谈你的观点和看法。

  阅读下面文字,完成下列小题。

  材料一

  为了更有针对性的满足不同旅客的出行要求,*高铁运行时速覆盖200公里、250公里、300公里、350公里四个速度等级,在同一个高铁路网中同时运行如此多速度等级的动车组,在世界上独一无二。今年9月21日,7对“复兴号”动车组在京沪高铁率先实现350公里时速运营,*重新成为世界铁路运营时速最高的国家。*高铁不仅速度快,而且运营规模大,截至20xx年末,我国高速铁路营业里程逾2.2万公里,比20xx年底增长了1.4倍,稳居世界首位。

  在满足人民群众对更加快捷方便的轨道交通需求的同时,*高铁越来越注重布局的均衡,五年来,我国中西部高铁营业里程从20xx年底的0.3万公里增加到20xx年底的1.3万公里,增长了3倍多,在路网中占比超过半壁江山。到20xx年,我国高铁运营规模将达到3万公里,覆盖80%以上的大城市;与公路、民航、水运、城市轨道交通有效衔接,以高铁为大动脉的综合交通运输体系将展现出“交通强国”的英姿。材料二:

  材料三

  20xx年,*高铁旅客发送量超14.7亿人次,*均每天403万名旅客乘坐高铁出行。支撑如此大规模运输量的关键,就是*铁路的自主创新、科技创新实力。也正是因为坚定不移走*特色自主创新道路,走好了科技创新这步先手棋,才迈出了高铁强国的铿锵步伐。

  这是一个世界级的新跨度。作为世界上跨度最大的钢筋混凝土拱桥,北盘江大桥主跨

  445米,距离江面300米,需应对相当于10级狂风的峡谷风速。然而,当动车组以时速

  300公里疾驰而过时,桥面变形只有3毫米,远低于10毫米的世界前沿标准。

  这是一个世界级的新速度。20xx年7月15日,两列*标准动车组以超过420公里的时速在郑徐高铁上交会,创造了高铁列车交会、重联运行速度的世界最高纪录。交会时产生的压力波相当于1*方米瞬时受重*200公斤。如此高的压强,车内的工作人员和体验者却没有任何不适感。

  目前,*铁路在勘察设计、工程建造、高速动车组、列车控制、牵引供电、运营管理、安全保障等领域已经取得一系列自主创新成果,*铁路总体技术水*已迈入世界先进行列,部分技术处于世界领先水*,为其他国家发展高等级高铁提供了有力的技术支撑,充分展现了我们的实力。“从跨越大江大河到穿越山谷溶洞,从有砟到无砟,从有缝钢轨到无缝钢轨,从追求建设完美到兼顾生态及文物保护……世界上没有哪个国家的建设者,会遭遇如此复杂的设计挑战。”中铁第四勘察设计院集团有限公司副院长王玉泽说。作为*“新四大发明”之一,高铁不仅安全、便利、快捷地服务着*旅客,同时也惊艳着世界。

  材料四:有人说“没有高铁,城镇就是散落的珍珠;通了高铁,城镇就是项链上的珍珠。”比如世界上首条高寒地区长距离高铁——哈大高铁运营以来,以哈尔滨、长春、沈阳、大连为中心的“1小时交通圈”从梦想变成现实,东北三省真正融为一体。再如世界上运营里程最长的京广高铁,把珠三角经济圈、长株潭城市圈、武汉城市群、中原经济区及环渤海经济圈等*从南到北的五大经济区紧紧地连在了一起,带动沿线的28个城市间彼此往来不超过8小时,时间距离的缩短让人们的空间感觉更为接*。

  十年前,*首趟时速200公里动车组列车在上海站始发,我国由此进入动车时代;十年后,具有完全自主知识产权的“复兴号”以350公里的世界高铁最高商业运营时速在京沪高铁、京广高铁上飞驰。可以说,*高铁改变着*,震撼着世界。短短十年,*高铁从无到有,从基本成型的“四纵四横”,到规划中的“八纵八横”高速铁路网,高铁带来的美好生活极大地刺激着*人的想象力,相信在不远的将来,这种想象力将会转变成伟大的创造力,实现每个人心中的“*梦”。

  1.下列对材料相关内容的梳理,不正确的一项是

  A. 目前*是世界铁路运营时速最高的国家,*铁路不同等级的运行时速,有针对性地满足了不同旅客的出行要求。

  B. 截至20xx年末,我国高铁运营里程超过2.2万公里,形成了世界上运营规模最大的高速铁路网。

  C. 从20xx年底到20xx年底,我国中西部高铁营业里程增长了3倍多,规模超过全国铁路总营业里程的一半。

  D. 在高铁建设方面,*能提供针对复杂地形情况的、覆盖各种速度等级的,从勘察设计到建设运营的经验,是当之无愧的高铁建设强国。

  2.下列对材料相关内容的分析和评价,不正确的两项是

  A. 根据材料二,到20xx年,我国高铁营业里程将达到3万公里,高铁在铁路运营里程中将继续保持稳定快速增长。

  B. 北盘江大桥建设和*标准动车组两车交会都创造了世界纪录,充分证明了自主科技创新推动*成为高铁强国。

  C. 自主创新实力是*成为高铁强国最重要的保障,正是因为总体技术水*世界领先,高铁被誉为*“新四大发明”之一。

  D. “通了高铁,城镇就是项链上的珍珠”是说高铁路网对*的城市及经济区的联通发挥了重要作用,有力地促进了*经济的增长。

  E. 根据材料四,*高铁从无到有,从“四纵四横”到“八纵八横”的建设奇迹,改变了*,震撼了世界。

  3.*高铁有哪些特点?请结合材料简要概括。

  二、文言文阅读

  阅读下面的文言文,完成下列小题。

  张昭字子布,彭城人也。少好学,善隶书,博览众书,与琅邪赵昱、东海王朗俱发名友善。弱冠察孝廉,不就,与朗共论旧君讳事,州里才士陈琳等皆称善之。刺史陶谦举茂才,不应,谦以为轻己,遂见拘执。昱倾身营救,方以得免。

  孙策创业,命昭为长史、抚军中郎将,升堂拜母,如比肩之旧,文武之事,一以委昭。策临亡,以弟权托昭,昭率群僚立而辅之。权悲感未视事,昭乃身自扶权上马,陈兵而出,然后众心知有所归。昭复为权长史,授任如前。权每田猎,常乘马射虎,虎尝突前攀持马鞍。昭变色而前曰:“将军何有当尔?夫为人君者,谓能驾御英雄,驱使群贤,岂谓驰逐于原野,校勇于猛兽者乎?如有一旦之患,奈天下笑何?”权谢昭曰:“年少虑事不远,以此惭君。”然犹不能已,乃作射虎车,为方目,间不置盖,一人为御,自于中射之。时有逸群之兽,辄复犯车,而权每手击以为乐。昭虽谏争,常笑而不答。

  权于武昌临钓台饮酒大醉权使人以水洒群臣曰今日酣饮惟醉堕台中乃当止耳昭正色不言出外车中坐。权遣人呼昭还,谓曰:“为共作乐耳,公何为怒乎?”昭对曰:“昔纣为糟丘酒池长夜之饮,当时亦以为乐,不以为恶也。”权默然,有惭色,遂罢酒。初,权当置丞相,众议归昭。权曰:“方今多事,职统者责重,非所以优之也。”后孙邵卒,百寮复举昭,权曰:“孤岂为子布有爱乎?领丞相事烦,而此公性刚,所言不从,怨咎将兴,非所以益之也。”乃用顾雍。

  昭容貌矜严,有威风,权常曰:“孤与张公言,不敢妄也。”举邦惮之。年八十一,嘉禾五年卒。遗令幅巾素棺,敛以时服。权素服临吊,谥曰文侯。

  (选自《三国志?吴书?张昭传》,有删节)

  1.下列对文中画波浪线部分的断句,正确的一项是

  A. 权于武昌/临钓台/饮酒大醉/权使人以水洒群臣曰/今日酣饮/惟醉堕台中/乃当止耳/昭正色不言/出外车中坐

  B. 权于武昌/临钓台/饮酒大醉/权使人以水洒/群臣曰/今日酣饮惟醉/堕台中/乃当止耳/昭正色/不言出外/车中坐

  C. 权于武昌/临钓台/饮酒大醉/权使人以水洒/群臣曰/今日酣饮惟醉/堕台中/乃当止耳/昭正色不言/出外车中坐

  D. 权于武昌/临钓台/饮酒大醉/权使人以水洒群臣曰/今日酣饮/惟醉堕台中/乃当止耳/昭正色/不言出外/车中坐

  2.下列对文中加点词语的相关内容的解说,不正确的一项是

  A. 隶书,汉字中常见的一种字体,是两汉时期通行的主要字体,书写效果略微宽扁,讲究“蚕头雁尾”、“一波三折”,对后世书法有不可小觑的影响。

  B. “察孝廉,举茂才”是自上而下选拔人才的察举制的主要科目,孝廉指孝顺父母、品行方正之人。茂才即秀才,汉时因避刘秀讳改称茂才,与后代科举中的“秀才”意同。

  C. 视事,旧时指官吏到职办公。《张衡传》中有“视事三年”语,其中“视事”意即为此。本文中“权悲感未视事”指的是孙权因为哀痛兄丧不能亲自办公。

  D. 幅巾,是指用一块帛巾束首,是一种表示儒雅的装束。用葛布制成,称为“葛巾”,多为布衣庶人戴用;用细绢制成,称为“缣巾”,多为王公雅士戴用。

  3.下列对原文有关内容的概括和分析,不正确的一项是

  A. 张昭年少好学,素有才名。他二十岁时被推荐为孝廉,后来又被推举为秀才,都没有接受,甚至因不**而遭到捕系。

  B. 张昭受孙策礼遇,并被委以重任。孙策待他犹如并肩共事的旧友,军政大事全都托付给张昭办理,临终时又让他辅佐孙权。

  C. 张昭忠心耿耿,直言敢谏。孙权冒险射虎,饮酒作乐,他都毫不留情地批评,孙权都接受了他的批评并立即改正,但也据此认为张昭不堪丞相之职。

  D. 张昭甚有威仪,受百官拥戴。他曾两度被群臣推举为丞相人选,虽然孙权认为他不能胜任丞相之职,但对他仍敬畏有加。

  4.把文中画线的句子翻译成现代汉语。

  (1) 夫为人君者,谓能驾御英雄,驱使群贤,岂谓驰逐于原野,校勇于猛兽者乎!

  (2) 领丞相事烦,而此公性刚,所言不从,怨咎将兴,非所以益之也。

  三、诗歌鉴赏

  阅读下面这首唐诗,完成下列小题。

  野人送朱樱

  杜? 甫

  西蜀樱桃也自红,野人相赠满筠笼。

  数回细写①愁仍破,万颗匀圆讶许同。

  忆昨赐沾门下省②,退朝擎出大明宫。

  金盘玉箸无消息,此日尝新任转蓬。

  [注]①“写”同“泻”,言用水漂洗,这里指将樱桃从一个容器倒入另一个容器。②门下省,杜甫当年任左拾遗,属门下省。

  1.下列对这首诗的赏析,不恰当的两项是(??? )

  A. “西蜀樱桃也自红”的“自”精妙传神,写出西蜀樱桃应季而红,却无人欣赏的落寞,与“映阶碧草自春色”有异曲同工之妙。

  B. “野人相赠满筠笼”,野人,指村农;筠笼,竹篮。村农以“满”篮鲜果“相赠”,足见诗人与邻里相处欢洽。

  C. 颔联上句写诗人担心樱桃碰破,表现了西蜀樱桃娇嫩易破的弱点,*添一丝哀愁;下句写樱桃形体“匀圆”,隐含对果农种植技艺与劳动的赞美。

  D. 全诗可分三层:前两联写“今日”,第三联忆“昨日”,第四联回到“今日”;引起第三联诗人回忆的,是“野人送朱樱”这一小小事件。

  E. 这首诗以“朱樱”为描写对象,采用今昔对比手法,表达了诗人对供职门下省时的生活细节的深情忆念,拓宽了生活层面,增添了感情厚度。

  2.“忆昨赐沾门下省,退朝擎出大明宫。”有人评价此联“门下省、大明宫”两个专有名词的使用,点铁成金,意蕴丰富。试结合此联与尾联诗意简要分析。

  四、情景默写

  补写出下列名篇名句中空缺的部分。

  (1) 陆游在《游山西村》中,用“__________________,__________________ ”两句,描写了庆典将*,音乐不断的热闹景象,也反映了当时淳朴的民风。

  (2) “____________,____________,_____________”,才是真正的逍遥游,也才能达到至人、神人、圣人那样绝对自由的精神境界。

  五、选择题

  下列各句中划线成语的使用,全部正确的一组是(??? )

  ①最*故宫博物院院长单霁翔因为一个演讲视频火了。视频中单院长对故宫的各种藏品如数家珍,被大家誉为有智慧、金句多的“段子手”。

  ②唐朝诗人王维被边塞的奇景震撼,写下了“大漠孤烟直,长河落日圆”的名句。无独有偶,宋代范仲淹在边塞军中也对这样的风光作过描写。

  ③完善社会主义市场经济体制必须有详细周密的总体规划,零敲碎打的改革不能达到我们预想的目标。

  ④短短十天的军训过去了,但我们忘不了教官不胜其烦地重复单调的动作,更忘不了烈日下我们共同奋斗的身影。

  ⑤短暂的20分钟节目浓缩了王华同志对新疆、农四师、家庭的热爱,他在事业上的坚守展现了一名*人冰壶秋月的松柏胸襟。

  ⑥人们普遍认为《简·爱》是一部自传性质的作品,夏洛蒂·勃朗特以鲜明独特的女性视角和叙事风格侃侃而谈,真实而有艺术感染力。

  A. ①②⑤

  B. ①③⑤

  C. ②⑤⑥

  D. ①③④

  下列各句中,没有语病、句意明确的一句是

  A. 12月16日,由*、江苏省人民*主办的第三届*歌剧节将在南京拉开帷幕,来自全国各地的23台歌剧剧目将参演,为历届之最。

  B. 如果人工智能真的如设想的那样,能够创造新思想、模仿人类思维、**生产力,这无疑是自工业革命以来最激动人心的事件。

  C. 在全国各个电视台热播的真人秀节目,或许无法承受复兴传统文化的重任,但至少也应该减少庸俗乃至低俗的打趣的价值定位。

  D. 因不满过去的制度而弃考,因受到当下改革的激励而复考,“白卷考生”徐孟南重返高考考场的举动,某种程度上体现了他对今日高考改革的一种认同。

  阅读下面材料,选出四项推论中正确的一项是(??? )

  心理学家德威克用10年时间对400名五年级小学生进行实验,发现被夸聪明的孩子更容易放弃、缺乏自信、不敢冒险。后来她重复实验时把不同社会阶层都纳入了自己的实验,都发现了相同的结果,不论男女,连学龄前的孩子也未能幸免被表扬后带来的负面效应。

  A. 研究表明,表扬并不一定都具有积极作用,效果因表扬的具体内容和形式而有天壤之别。

  B. 一份关于表扬的综合调查结果显示,被表扬聪明的学生变得更倾向于规避风险和缺乏独立精神。

  C. 一项对积极表扬组和无表扬组的学生的学*成绩对比研究显示,他们的学*成绩差异不显著。

  D. 研究发现,表扬对最聪明的女孩影响最大,她们大部分在一项“失败反应”的测试中崩溃了。

  六、语言应用

  在下面一段文字横线处补写恰当的语句,使整段文字语意完整连贯,内容贴切,逻辑严密。每处不超过20个字。

  长久以来,农村的发展方式相对单一,主要通过第一产业向市场提供农产品。而如今,①____,改变田园的生活。旅游不仅能够重塑农村的面貌, ②____。传统意义上的农业生产是产品思维, ③____,围绕旅游所提供的服务都是从人的需要出发。因此,旅游的参与将会从思维方式上影响着农村经济的发展,让田园生活更加美好。

  找出下面文字在语言表述方面存在的三处错误,按要求完成下列小题。

  邀请函

  亲爱的校友:

  ①光阴荏苒,斗转星移,②汇文中学进入了建校五十周年。为了凝聚校友力量,回顾过去,展望未来,③学校决定于20xx年3月1日隆重举行建校五十年庆典活动。④昔日您以母校为荣,今天母校以您为荣,⑤请您届时务必拨冗出*,⑥与全校师生共襄盛典,⑦同贺母校五十华诞。

  汇文中学校庆委员会

  20xx年1月10日

  (1)有语病的是第______处,修改:____________

  (2)不得体的是第______处,修改:________________

  (3)邀请函中缺少的重要信息是________________________

  七、材料作文

  阅读下面的材料,根据要求写作。

  20xx年12月21日,国家语言资源监测与研究中心、商务印书馆等联合主办的“汉语盘点20xx”结果揭晓,“享”、“初心”、“智”、“人类命运共同体”分别当选年度国内字、国内词、国际字、国际词。一个字,一种生活;一个字,一年记忆。如果用一个汉字或一个汉语词来总结你的20xx 年,你会做怎样的选择呢?可以自主选择字词,也可以参考以下汉字“爱、梦、强、搏、读”作文。

  要求选好角度,确定立意,明确文体,自拟标题,完成写作任务;不要套作,不得抄袭;不得少于800字。

届高三上学期期末考试语文试卷3

  一、选择题

  在下面一段话的空缺处依次填入的词语,最恰当的一组是

  “ ”的社会看上去很有序,但社会体制却是僵化的,人被僵化的体制 ,没有自由发挥的空间,一切都是设定好的、 的。这种“有序”社会是没有活力的社会,当然也是难以向前发展的社会。

  A.死气沉沉 束缚 墨守成规 B.死水一潭 束缚 按部就班

  C.死水一潭 制约 按部就班 D.死气沉沉 制约 墨守成规

  下列各句中加点的词语,都与古典小说、戏曲或传说有关,对应不正确的一项是

  A.看我“七十二变”,爱漂亮没有终点,麻雀也能飞上青天。(《西游记》)

  B.又没有“红娘”式的人物,要理解对方的心思是多么困难啊!(《西厢记》)

  C.老听人家夸赞你,我还以为你是一个“三头六臂”的女英雄呢。(《水浒传》)

  D.你看看你,开了水龙头不记得关,家里都“水漫金山”了。(《白蛇传》)

  填入下面唐诗划线处的句子,最恰当的是一项

  寒窗灯尽月斜晖,佩马朝天独掩扉。 ,白云空长越山薇。

  病中送客难为别,梦里还家不当归。惟有寄书书未得,卧闻燕雁向南飞。

  A.细水浮花归别浦 B.淡烟浮照明星楼

  C.清露已凋秦塞柳 D.绣户夜攒红烛市

  在下面一段文字横线处填入语句,衔接最恰当的一项是( )

  世间事物有真、善、美三种不同的价值,人类心理有知、意、情三种不同的活动。 人生来就有真、善、美的需要,真、善、美具备,人生才完美。

  ①求知、想好、爱美,三者都是人类天性。

  ②人能发意志,就要想好,就要趋善避恶,造就人生幸福。

  ③这三种心理活动恰和三种事物价值相当:真关于知,善关于意,美关于情。

  ④人能动情感,就爱美,就欢喜创造艺术,欣赏人生自然中的美妙境界。

  ⑤人能知,就有好奇心,就要求知,就要辨别真伪,寻求真理。

  A.③⑤②④① B.①⑤②④③

  C.③⑤④②① D.①⑤④②③

  下列关于古代文化常识的表述,错误的一项是

  A.宫、商、角、徵、羽是*古乐基本音阶,起源于春秋时期。

  B.*是礼仪之邦,来往称呼接洽,多使用敬辞,其中陛下、殿下、膝下、足下、在下,都是敬称用语。

  C.“南阳高士吟梁父,西蜀才人赋子虚”,前句说的是诸葛亮,后句说的是司马相如。

  D.“避讳”是我国封建社会特有的现象,“春秋为尊者讳,为亲者讳,为贤者讳”。黛玉因母亲名字有“敏”字,所以写“敏”字时常用它字代替或曲笔,即为“避讳”之举。

  二、文言文阅读

  阅读下面的文言文,完成下面小题。

  寄欧阳舍人书

  曾巩

  去秋人还,蒙赐书,及所撰先大父墓碑铭,反复观诵,感与惭并。夫铭志之著于世,义*于史,而亦有与史异者。盖史之于善恶无所不书,而铭者,盖古之人有功德材行志义之美者,惧后世之不知,则必铭而见之。或纳于庙,或存于墓,一也。苟其人之恶,则于铭乎何有?此其所以与史异也。其辞之作,所以使死者无有所憾,生者得以致其严。警劝之道,非*乎史,其将安*?

  及世之衰,为人之子孙者,一欲褒扬其亲,而不本乎理。故虽恶人,皆务勒铭,以夸后世。立言者既莫之拒而不为,又以其子孙之所请也,于是乎铭始不实。后之作铭者,当观其人。苟托之非人,则书之非公与是,则不足以行世而传后。故千百年来,公卿大夫至于里巷之士,莫不有铭,而传者盖少。其故非他,托之非人,书之非公与是故也。

  然则孰为其人而能尽公与是欤?非畜道德而能文章者,无以为也。盖有道德者之于恶人,则不受而铭之;于众人,则能辨焉。而人之行,有情善而迹非,有意奸而外淑,有善恶相悬而不可以实指,有实大于名,有名侈于实。犹之用人,非畜道德者,恶能辨之不惑,议之不徇?不惑不徇,则公且是矣。而其辞之不工,则世犹不传,于是又在其文章兼胜焉。故曰,非畜道德而能文章者无以为也,岂非然哉!

  然畜道德而能文章者,虽或并世而有,亦或数十年或一二百年而有之。其传之难如此,其遇之难又如此。若先生之道德文章,固所谓数百年而有者也。先祖之言行卓卓,幸遇而得铭,其公与是,其传世行后无疑也。而世之学者,每观传记所书古人之事,至其所可感,则往往衋然①不知涕之流落也,况其子孙也哉?况巩也哉?

  既拜赐之辱,且敢进其所以然。所谕世族之次,敢不承教而加详焉?愧甚,不宣。

  (选自《古文观止》,有删节)

  注:①衋然:伤痛的样子。

  1.对下列加点词的解释,不正确的一项是

  A.生者得以致其严 严:敬意 B.而不本乎理 本:探究

  C.意奸而外淑 淑:善良 D.其传世行后无疑也 传:传播

  2.下列对文中相关内容的解说,不正确的一项是

  A.作者认为铭与史不同,墓志铭应写贤者,而史则善恶皆书。

  B.文中的庙指宗庙,而不是寺庙,是祭祖的房屋,铭文可置于其中。

  C.公卿大夫在文中泛指为官做宰的人,他们都有墓志铭,但流传下来的却很少。

  D.欧阳修在给作者的信中厘清了曾氏家族世系,作者表示会对世系增加充实扩展。

  3.把文中画线的句子翻译成现代汉语。

  (1)立言者既莫之拒而不为;又以其子孙之所请也,于是乎铭始不实。

  (2)犹之用人,非畜道德者,恶能辨之不惑,议之不徇?

  4.结合全文,用自己的话概括曾巩认为墓志铭少有流传的原因。

  三、诗歌鉴赏

  阅读下面这首清词,完成下面小题。

  蝶恋花

  【清】谭献

  庭院深深人悄悄,埋怨鹦哥,错报韦郎到。压髯钗梁金凤小,低头只是闲烦恼。

  花发江南年正少,红袖高楼,争抵还乡好?遮断行人西去道,轻躯愿化车前草。

  1.词的上阕前三句是如何刻画主人公形象的,请简要分析。

  2.词的下阕是如何抒发感情的,试简要分析。

  四、句子默写

  补写出下列名句名篇中的空缺部分。

  (1)_______,子无良媒。(《诗经.氓》)

  (2)汩余若将不及兮,________。(屈原《离骚》)

  (3)________,忽复乘舟梦日边。(李白《行路难》)

  (4)________,黄芦苦竹绕宅生。(白居易《琵琶行》)

  (5)________,则天地曾不能以一瞬。(苏轼《赤壁赋》)

  (6)元嘉草草,封狼居胥,________。(辛弃疾《永遇乐·京口北固亭怀古》)

  (7)_________,衣冠简朴古风存。(陆游《游山西村》)

  (8)_________,为霞尚满天。(刘禹锡《酬乐天咏老见示》)

  五、现代文阅读

  阅读下面的作品,完成下面小题。

  江底之夜

  艾芜

  走下十里多路的山坡,阻挡在面前的,正是一条水势汹汹的江流和排立在江边的一列街屋,都在暮色中渐渐朦胧起来。街口吐出一大群回到远山去的村人,荷着土洋炮和带着红缨子的长矛,倘不看见另一只手还提有竹筐及布口袋,那会把他们这些赶街的人疑为土匪的。

  “真是幸运!”这样想着。因为一个人,又没带武器,却安然地走到了一个可以暂时住宿的地方。

  这儿叫江底,我住在一家临江的马店。兼做店老板和小伙计的,是一位三十来岁的粗女人,衣衫已经补了好些块不配色的疤,三个高矮不齐的孩子和一个尚未满岁的婴儿,时常吵闹着她。

  我的装有书和衣衫的包袱,就由她拿去放着。等我和大孩子小猪一块坐在火堆旁边用心烧吃包谷时,她便走进她那间小屋子,把房门紧关,她的第二个孩子,在屋门口急嚷着“妈妈”。

  “这一包烧黄的,先给你。”

  我一面把刚从火堆里取出来的包谷,撕去一层层的皮,一面招呼着那个快哭的孩子,便听见里面有几本书那样的东西“突地”跌落在地上。我猜疑她在检查我的包袱。该没有东西使她看上吧?她那样健壮,不会把我从后门掀下江去吗?她的丈夫呢?一面胡猜乱想着,一面问小猪:

  “你爸爸呢?”

  他嘟起小嘴巴说道:“没有爸爸,只有个叔叔!”

  “叔叔呢?”

  “叔叔没有住在这里,半夜才……。呵哟,烧焦了!”他赶忙用铁火钳拨开火堆,将一块冒着烟的包谷挟了出来。

  门呀的一声,女主人生气地出现在门口,“看你一刻也离不开,鬼东西!”一边大声叱骂站在门前的二儿子,一边迅速地瞥视我一下。

  “哇——”,她先前抱进屋的孩子在里面哭起来,背在背上的婴儿也跟着哭了起来。她跑进屋里去,把哭着的第三个小孩子提了出来,给他一个耳光之后,随即把大孩子手中啃了一半的包谷抢去,硬塞在小儿子的嘴上,将那张发出哭声的嘴洞,莽撞地堵住。

  这第三的孩子真的不哭了,但背上那个却更加哭得凶横了。小猪躲在黑角落里小声地抽噎,不敢哭出来。

  睡的时候,我被引进店后一间小屋子,里面发出久无人住的霉气。蜘蛛网像吊着许多流苏一样地,现在屋顶下边和四只角上。两条长凳上横放着三四块松木板子,没有草*,没有棉枝。

  “这样怎么好睡呢?老板娘。”

  “真没法呵,将就点吧,客人!”女主人板着面孔。

  “怎么能够将就?谁愿意出钱睡这样的客店?”

  “到了我们这地方,是要受点委屈的哪。”接着冷冷地笑着。

  “难道这地方会穷到这样子吗?”

  见我稍露讥刺,她便吊下两只嘴角,气愤地说道:

  “哪里穷?这样好的地方!那些天杀的东西接二连三来抢,还有你们那些保商队……哼,过一回,光一回……”

  我早已非常疲倦:“算了,算了,就这样睡吧。”

  光板的床上不好睡,壁板缝里又钻入江风,约到半夜以后就醒了,听着江涛打岸。

  “笃笃笃……”店外响起敲门的声音,起初小而低沉,渐渐便大起来:“砰砰砰……”女主人喂喂地急应着去开门。

  “睡死了!老子晚上走这样远来!”

  “呵呵!这是你们山上种的南瓜吗?……小声点,小声点,今晚有过路的人。”

  “包袱大吗?”

  男子突然很小声,女人的回答却低微到听不见。我轻坐起来,带着些微的恐怖。壁板缝里望江上,月色正好,黑油油的江水,碰在江中突出的大石上,溅射出无数灿然的银花。一个旅人,晚上来到这儿息宿,半夜被人推下江去,谁也不会察觉。呵,可怖的地方!我不安地躺着,直到天微明才昏迷迷地睡去。天光大亮醒来,觉得自己还在着,便非常喜悦地做着早上要吃的东西。

  那男子已不见了。女主人将一个壮大黄圆的南瓜连皮切在瓦罐内,三个高矮不齐的孩子围在她身边,睁大贪食的眼睛,舐着带有唾液的嘴唇。

  忽然看见壁上挂着一张小小的像片,就着窗外透进的鲜朗晨光,能从薄尘中分辨出两个年轻军人的雄健姿影。倒边隐约有字,细看始明白:

  “民国八年与徐排长摄于四川之泸州,后徐君阵亡于成都龙泉驿一役,即将此仅存之遗影,敬赠君之夫人惠存。

  陈长元谨赠”

  字迹粗劣,大概是一个排老二之之流写的罢。回头去看见孩子们和母亲还在那里热心地弄煮着南瓜,心里便禁不住黯然起来。

  (选自《漂泊杂记》,有删改)

  1.简要分析第一段在文中的作用。

  2.试推断文章划线句中女主人生气的原因。

  3.简要概括女主人家庭的生存状况。

  4.请探究文章结尾划线句“心里便禁不住黯然起来”的意蕴。

  阅读下面的论述类文本,完成下面小题。

  网络使我们变笨了吗?

  几年前,畅销书作家尼古拉斯·卡尔出版了《浅薄》,副标题是“互联网如何毒化了我们的大脑”。显然,他认为两络使我们变笨了。因为我们在网络时代被信息包裹得喘不过气、选择困难、注意力不集中、无法做深度思考?

  那么,网络真的使我们变笨了吗?

  我们先把这个问题放在一边,来聊一聊戴维·温伯格的《知识的边界》。

  这本书的主题是关于网络时代知识的处境。但戴维·温伯格拒绝技术决定论,他在这本书里所做的更多是在梳理知识网络化的过程和现状。

  如果我们把网络时代和印刷术时代做一个区分,可以看到知识在这两个时代的创造、传播和获取,都有很大不同。

  纸是有限的,而网络是无限的;纸上信息是固定的,而网络上的信息是流动的。

  纸是有限的这一点很重要。首先,纸的固定形式,决定了思考的形式,它必须有开头,有论证,有结尾,它是线性的。其次,也正是因为纸的容量是有限的,所以在印刷术时代,我们拥有一套严格的信息过滤机制,只有真正重要的东西,才能够被出版。但在网络时代,过去的过滤机制失效了,我们被淹没在信息海洋之中。

  纸上的信息是固定的也很重要,正因为此,它获得了一种庄重感和权威感,当然还有确定感,它的潜台词是,我们是可以理解这本书的。而网络是流动的,一切都在快速地更新和变化之中,它**了知识,但没有办法重新收纳它,或者说,它已经没法收纳了。

  这背后,潜藏着两种看待世界的方式。印刷术时代,人们将知识分门别类,相信通过理性思考可以认识整个世界。因为知识在印刷术时代是被控制的,一步一个脚印,我们总能走到最后。但是网络时代显现了一个事实,世界太大了,大到我们根本不可能全部理解,这一点,无疑让印刷术时代过来的人感到恐慌。

  这种种不同,在戴维·温伯格看来各有利弊。比如说,网络时代很容易生成“回音室”,人们容易只靠*和他一致的思想,对其他思想进行排斥,这无疑会加重思想的僵化和极端化。

  另一方面,知识的网络化带来了很多不可预料的东西,比如说*,它调动了许许多多个节点,让知识的生产和传播的形式都发生了改变,用戴维.温伯格自己的话来说就是:当知识变得网络化之后,房间里最聪明的人不是站在讲台前给我们讲课的人,也不是房间里所有人的集体智慧,而是房间本身。

  但是,戴维·温伯格始终没有回答另一个问题:个人,在网络中该怎么自处?

  所以,那个问题又回来了,网络真的使我们变笨了吗?

  我和戴维·温伯格一样不倾向于技术决定论,重要的永远是人,也就是你自己。网络有诸多好处,也有诸多弊端,但这并不妨碍一个人自己的求知欲望和行动。

  是的,这就是我们的时代。所以,木心的那句话在这里又可以用上了,我们只能“在自己身上克服这个时代”。

  (摘自《冒犯经典》魏小河著,有删改)

  1.请简要概括本文的论述层次。

  2.文中说知识在印刷术时代和网络时代的创造、传播和获取有很大不同,请简要概括。

  3.请结合文章,阐释最后一段划线句中“在自己身上克服这个时代”的内涵。

  六、材料作文

  根据以下材料,选取角度,自拟题目,写一篇不少于800字的文章;文体不限,诗歌除外。

  有人说,好人必有好报;也有人说,好人未必有好报。但人们选择做好人善事,是因为回报,还是因为那是对的事情呢?

  七、文言文阅读

  阅读材料,完成下面小题。

  尝谓文者礼教治政云尔其书诸策而传之人大体归然而已。而曰“言之不文,行之不远”云者,徒谓“辞之不可以已也”,非圣人作文之本意也。退之、子厚徒语人以其辞耳,作文之本意,不如是其已也。且所谓文者,务为有补于世而已矣;所谓辞者,犹器之有刻镂绘画也。诚使巧且华,不必适用;诚使适用,亦不必巧且华。要之,以适用为本,以刻镂绘画为之容而已。不适用,非所以为器也。不为之容,其亦若是乎?否也。然容亦未可已也,勿先之,其可也。

  (选自王安石《上人书》,有改动)

  1.用斜线“/”给下面的文言文断句。(限4处)

  尝谓文者礼教治政云尔其书诸策而传之人大体归然而已。

  2.退之、子厚分别是指韩愈和_______,他们同为中唐_______的倡导者。

  3.用自己的话概括本段材料的主要内容

  八、选择题

  下列对有关名著的说明,不正确的两项是

  A.《哈姆莱特》中,波洛涅斯被误杀后,克劳狄斯才觉察到哈姆莱特的精神错乱不像是为了恋爱,也不像是疯狂,心里害怕哈姆莱特会产生危险的后果,决定让他去英国。

  B.老舍先生的《茶馆》以老北京裕泰茶馆的兴衰变迁为背景,共分三幕,向人们展示了从清末到北洋军阀时期,再到新*成立前夕,北京的社会风貌及各阶层人物的不同命运。

  C.《边城》中,老船夫下葬后三天,船总顺顺亲自来碧溪岨商量接翠翠过家里去住,翠翠却想看守祖父的坟山,不愿即刻进城。

  D.《欧也妮·葛朗台》中,欧也妮叫人将自己一直珍藏的金饰熔掉,加上夏尔还给她的八千法郎,铸了一个圣体显供台,送给了她曾为夏尔祈祷过无数次的教堂。

  E.《社戏》中,一群少年在看戏回去的路上偷了八公公田里的豆,第二天,纯朴、大度的八公公不但没有严厉责备,还送了好些罗汉豆给迅哥儿母子吃。

  九、名著阅读

  简答题

  (1)《红楼梦》中,莺儿嘻嘻笑道:“我听这两句话,倒像和姑娘的项圈上的两句话是一对儿。”莺儿所说的“姑娘”是谁,是在怎样的情境下说的。

  (2)请简要分析《三国演义》中关羽失荆州走麦城的原因。

  十、现代文阅读

  阅读材料,完成下面小题。

  文字是文明栽体,记录着国家和民族的历史发展进程。汉字是中华传统文化代代相传并不断繁荣发展的根基,汉字教育是凝神聚力、铸魂固本的战略工程。如果汉字消亡了,国家和民族的根基也就动摇了。

  但自清末以来,在日本“脱亚入欧”的影响下,*一些知识分子也对汉字质疑:“汉字难学”论者主张汉字“拼音化”与“简化”;“汉字落后”论者甚至提出“汉字不灭,*必亡”,认为汉字的出路在于“拉丁化”,主张放弃汉字教育。20世纪80年代,还有人认为“计算机是汉字的掘墓人”“汉字行将就木”。时下,英语是学生的必修课,学*英语可以改变命运的言论甚嚣尘上。中学阶段,在汉语与英语之间,学生更重视英语,一进大学,越发专注于英语的四级六级甚至八级,忙托福、雅思,顾了媳妇忘了娘。随着信息、智能技术的迅猛发展,汉字也变成了电脑键盘、手机屏幕上字母组合的产物,人们书写汉字的机会在变少,提笔忘字的时候在增多。曼妙婀娜、形神兼备的汉字面临危机绝不是危言耸听。

  汉字是象形文字,具有“形、音、义”三要素,与拼音文字不同。汉字教育,应遵循汉字自身的规律和特点。拼音文字是由字母组成的文字,其教学适合用拼音的方法,汉字教学引进并将其作为汉字教学的主要方法,其弊端日渐明显。*年来,一些汉字教育工作者根据“形、音、义”三要素和儿童擅长“形象记忆”的特点,探索“从字形讲字义,从生活讲汉字”和“因形分类、因类施教”等汉字教学的新理念与新途径,尝试把每个汉字的文化基因形象生动地传授给学生,让学生对汉字有兴趣、生爱恋、晓逻辑、会思考,甚至实现以字养德。这类教学实验,对完善汉字教育有积极意义。

  (节选自王殿卿《汉字教育攸关文化传承发展》,有删改)

  1.怎样理解文中“顾了媳妇忘了娘”的意思?

  2.为什么“曼妙婀娜、形神兼备的汉字面临危机绝不是危言耸听”?

  3.从材料看,怎样才能更好地进行汉字教育?


届高三语文试卷 (菁华3篇)(扩展3)

——届九年级中考模拟语文试卷 (菁华3篇)

届九年级中考模拟语文试卷1

  一、选择题

  初中生学*书法,应从楷书入手,学*书法,贵在多练,借鉴前人的经验,掌握一定的方法是必要的,但更重要的还需勤学苦练,持之以恒。王老师向同学们推荐了楷书字帖,符合推荐要求的选项是( )

  A.

  B.

  C.

  D.

  下面词语字形、注音不完全正确的选项是()

  A. 庇bì护 抽噎yē 笃dǔ志 神采奕yì奕

  B. 匿nì笑 缥pāo缈 慎chēn怪 拈niān轻怕重

  C. 蝉蜕tuì 中zhòng伤 干涸hé 怪诞dàn不经

  D. 竦峙zhì 殉xùn职 蹒pán跚 人迹罕hǎn至

  下列加点的词语使用不正确的选项( )

  A. 面对咄咄逼人的特朗普,墨西哥和总统涅托都需要在美国之外找一条新路,为市场带来稳定和信心,为经济带来活力与新增长点。

  B. 当全民健身上升为国家战略后,日常参与健身的人群日益扩大,他们以各种方式强健体魄,而在这其中,获得感、幸福感油然而生。

  C. 大家认为他提出的这条建议很有价值,都随声附和表示赞成。

  D. *驻加拿大大使卢沙野说,*不怕谈人权,只不过我们不想把这个问题和自贸协定谈判搅在一起,而加拿大某些媒体却经常把人权问题同经贸问题混为一谈。

  下列各组词语都属于谦辞的一项是( )

  A. 惠顾 高龄 奉送 贵庚 B. 舍妹 小弟 贤侄 斧正

  C. 奉劝 令郎 垂爱 寒舍 D. 家父 舍弟 鄙人 愚见

  二、句子默写

  填空

  ①岐王宅里寻常见,崔九堂前几度闻。_____,_____。(杜甫《江南逢李龟年》)

  ②________,_____。晴空一鹤排云上,便引诗情到碧霄。(刘禹锡《秋词》)

  ③岁月匆匆,“时间都去哪儿了”,我们不由得想起孔子在河边的那声感叹:“______,____。”《论语 子罕》

  ④古诗文中有许多表现“爱国”的句子,请写出完整的一句。______,____。

  三、名著阅读

  名著阅读

  (1)名著《西游记》中唐僧在松涧火云洞落难,悟空最终请来______(填人物)降伏了红孩儿。取得真经返归途中,老鼋作梗让师徒又经受了一次磨难,他作梗的原因是________。

  (2)①下列文章不属于《朝花夕拾》的是(____)

  A.《范爱农》 B.《风筝》 C.《无常》 D.《琐记》

  ②《五猖会》中回忆童年往事,含蓄地表达了对父母毫不顾及孩子心理的无奈与厌烦,这件事是什么?请用简练的语言叙述出来。

  (3)学过《再塑生命的人》,如果有兴趣,可以课外阅读《______》一书,感受(______)(作者)在逆境中奋进的精神和意志。

  四、诗歌鉴赏

  次北固山下 王湾

  客路青山外,行舟绿水前。 潮**阔,风正一帆悬。

  海日生残夜,江春入旧年。 乡书何处达?归雁洛阳边。

  1.下列对诗句理解分析不正确的一项是( )

  A. 诗题“次北固山下”中的“次”是游览的意思,表明诗人写诗的地点。

  B. 首联先写“客路”后定“行舟”,神驰故里的漂泊羁旅之情,流露于字里行间。

  C. 这是一首五言律诗,诗的颔联颈联对仗工整,这是律诗的一个重要特征。

  D. 尾联的“乡书”、“归雁”让人感受到一种淡淡的思乡愁绪。

  2.请赏析“潮**阔”一句中“阔”字的妙处。

  3.结合全诗分析作者是如何表达思乡之情的。

  五、文言文阅读

  子曰:“学而时*之,不亦说乎 有朋自远方来.不亦乐乎 人不知而不愠,不亦君子乎 ”(《学而》)

  曾子曰:“吾日三省吾身:为人谋而不忠乎 与朋友交而不信乎 传不*乎 ”(《学而》)

  子曰:“学而不思则罔,思而不学则殆。”(《为政》)

  子曰:“饭疏食,饮水,曲肱而枕之,乐亦在其中矣。不义而富且贵,于我如浮云。”(《述而》)

  子夏曰:“博学而笃志,切问而*思,仁在其中矣。”(《子张》)

  1.与“人不知而不温”中的“而”字用法相同的一项:()

  A. 学而时*之 B. 学而不思则罔 C. 曲肱而枕之 D. 切问而*思

  2.翻译下面的句子。

  为人谋而不忠乎 与朋友交而不信乎 传不*乎

  3.《论语》中有不少语句逐渐演化并固定为成语,至今仍活跃在现代汉语中,请列举出四个。

  4.结合选文,从内容和形式等方面概括《论语》的特点。

  陶侃惜谷

  陶侃尝出游,见人持一把未熟稻,侃问:“用此何为 ”人云:“行道所见,聊取之耳。”侃大怒曰:“汝既不田,而戏贼人稻!”执而鞭之。是以百姓勤于农植,家给人足。

  选自《晋书》

  1.下列各句中加括号的字与“执而鞭之”中的“之”字用法相同的一项是( )。

  A. 知(之)者不如好之者 B. 禽兽(之)变诈几何哉

  C. 而两狼(之)并驱如故 D. 久(之),目似瞑,意暇甚

  2.翻译下列句子。 用此何为

  3.作者对陶侃的态度如何 本文表现了陶侃的什么特点

  六、现代文阅读

  最妙的是下点小雪呀。看吧,山上的矮松越发的青黑,树尖上顶着一髻儿白发,好像日本看护妇。山尖全白了,给蓝天镶上一道银边。山坡上,有的地方雪厚点,有的地方草色还露着;这样,一道儿白,一道儿暗黄,给山们穿上了一件带水纹的花衣;看着看着,这件花衣好像被风儿吹动,叫你希望看见一点更美的山的肌肤。等到快日落的时候,微黄的阳光斜射在山腰上,那点儿薄雪好像忽然害了羞,微微露出点粉色。就是下小雪吧,济南是受不住大雪的,那些小山太秀气!

  1.这段话是按什么顺序展开描写的 请写出表示顺序的词语

  2.找出上文中表示色彩的词语,并写下来。说说它们有什么作用。

  3.“等到快日落的时候,微黄的阳光斜射在山腰上,那点儿薄雪好像害了羞,微微露出点粉色”能否改为“到快日落的时候,微黄的阳光斜射在山腰上,那点薄雪微微露出点粉色” 为什么

  4.最后一句话表达了作者怎样的感情?请根据上下文,说说你的理解

  我们跟着月亮走吧 朱成玉

  ①那年我12岁,是家里很不顺的一年,处处弥漫着哀伤的味道。先是祖父去世;然后父亲在工作中受伤,中指被车床绞断;随后是哥哥闯了祸,和几个小混混一起偷铁被派出所抓去,坐了班房还要罚款;紧接着是姐姐被一个男人欺骗了感情,整日里精神恍惚。这些事情几乎是一起涌过来的,母亲像一个太极高手,四两拨千斤,硬生生用她柔弱之躯扛起了这一切。

  ②父亲在医院里并不知晓家里发生的其他变故,母亲刻意提醒我们,因为祖父的去世,父亲心情一直没有缓过来,所以家里的事情必须对他隐瞒,让他安心养伤。

  ③母亲把亲戚朋友甚至邻居家趟了个遍,总算凑够了钱交了罚金,毕竟偷盗数额较小,派出所一通教育之后,就把哥哥放了回来。母亲并没有立即打骂,只是让他跪在院子里,让他自己思过。哥哥双膝跪着,心却直挺着,倔强地噘着嘴不肯服软:“我去偷东西,不也是寻思给弟弟妹妹买点儿好吃的吗?”

  ④“就是饿死,也不能白拿别人东西,何况是偷!”母亲终于爆发,气得脸色惨白如月,“人要清清白白,你就在这月亮地儿好好反省反省吧。”

  ⑤那晚的月亮白油漆一样泼在哥哥身上,像母亲的目光,一遍遍地洗刷着哥哥身上的污浊之气。

  ⑥几天后的一个夜晚,要去医院给父亲送吃的,很远的路,我和母亲走走停停。

  ⑦歇脚的当口,我看见母亲对着夜空轻叹了一口气,仿佛是在对上苍许着愿望,又仿佛是在向无边的黑夜倾诉着憋闷和委屈。眼角似乎有亮晶晶的东西在闪,母亲毕竟是女人,太多的事让她有些难以承受,可是她坚忍着不让泪水落下。

  ⑧乌云一层一层压过来,有些让人窒息,母亲也一直沉默着,让人不免担心一个闪电会不会将我们引爆。我想牵个话头出来,可是张开的嘴却被一阵风塞住。

  ⑨忽然间,天上的乌云裂开一个缝隙,月亮像获得自由的鸟儿,“嗖”一下窜出来!一绺一绺的月光像一把把利剑,霎时间割开夜妖的黑色袍子。母亲终于露出一丝笑容,“‘黑蘑菇’总有散开的时候。”并催促我说:“快,咱就一直跟着月亮走。”

  ⑩人生是由各种烦恼的碎片组合而成,每个人的一生都是不断拼接的过程。人生中那些艰难之事,于我早已不再新鲜,每每历经苦之风暴来袭,我都不会选择躲闪,而是勇敢地扎进去。我记住了母亲的那句话,也相信乌云总有散开的时候,月亮出现,一切就都有了转机和希望。

  跟着月亮走吧。

  这些年,不知是有意还是巧合,母亲总是喜欢在月亮地里对我们进行说教,母亲的“月光疗法”对我们健康成长起到了至关重要的作用。殊不知,母亲才是最温暖,最贴心的那缕月光,我们一生都栖息在她的光华里。

  母亲用强大的母爱叮嘱我,一切烦忧都不必过于介怀,跟着月亮走吧,自会走到柳暗花明处,自会走到鸟语花香间。

  1.文章围绕母亲的 “月光疗法”具体写了哪几件事?请简要概括。

  2.请分析第①段中加点词语的妙处。

  母亲像一个太极高手,四两拨千斤,硬生生用她柔弱之躯扛起了这一切。

  3.第⑾段在全文中有什么作用?

  4.文章标题“我们跟着月亮走吧”带给你怎样的启示?请结合生活实际谈谈你的看法。

  七、小作文

  应用文。下面的征稿启事存在语病,格式和标点符号方面各存在错误,请各找一处将修改建议写在答题纸上。

  征稿启事

  为了迎接“世界读书日”的到来,我校文学社决定举办以《我阅读我成长我快乐》为主题的征文活动。活动要求:征文内容围绕主题,文体不限;用300字稿纸抄写,不超过1000字以内;请将稿件于12月30日前送至我校文学社编辑部。欢迎同学们积极投稿。

  20xx年12月10日

  八、综合性学*

  综合性学*

  读书可以让我们享受和吸取人类文化的成果,让我们的心灵世界逐渐变得广阔,变得丰富多彩。少年正是读书时,热爱读书吧!让书陪伴你一生。现在,同学们准备开展“少年正是读书时”综合性学*活动,请你积极参与:

  ①请你给本次活动确定主题。

  ②为营造活动氛围,推荐两则读书的名人名言:

  ③为配合这次活动,班里还准备办一份读书小报。请你为这份小报拟写一个栏目名称,并简要介绍这个栏目的内容。(栏目名称为4个字)。

  九、命题作文

  以“这天,我回家晚了”为题,写一篇550字左右的作文。

  要求:文体不限,书写规范,不得抄袭套写;不能出现真实的姓名、学校等敏感信息。

届九年级中考模拟语文试卷2

  一、选择题

  下列词语书写及加点字注音全都正确的一项是( )

  A. 惩(chěng)罚 歼(jiān)灭 戛(gā)然而止 相形见绌

  B. 喑(yīn)哑 惬(qiè)意 贻(yí)笑大方 谈笑风生

  C. 酷肖(xiāo) 愧怍(zu ) 言简意赅(gāi) 披星带月

  D. 媲(pì)美 畸(qí)形 满载(zāi)而归 粗制烂造

  二、名句名篇

  请根据要求填写古诗文名句。

  ① 大漠孤烟直,___________________ 。(王维《使至塞上》)

  ② ______________,却话巴山夜雨时。 (李商隐《夜雨寄北》)

  ③人不寐,_________________________ 。(_________)《渔家傲》)

  ④天街小雨润如酥 ___________。 (韩愈《早春呈水部张十八员外》)

  ⑤忽如一夜春风来,______。(岑参《白雪歌送武判官归京》)

  ⑥诵读古代诗词,确实能让你受益终身。当你遇到逆境时,你可以用李白的“长风破浪会有时,______________”来勉励自己;当你与亲友分别时,你可以用王勃的 ________,天涯若比邻”来抚慰自己。

  三、综合性学*

  *的戏曲,源远流长,有着鲜明的民族风格,是人们喜闻乐见的文艺形式,全国许多地方都有着自己的剧种,可谓百花齐放,异彩纷呈。今天我们一同走进民族文化的瑰宝——戏曲天地,去感受它博大的内涵,去品味它悠长的韵味。

  (1)京剧表演中,常用脸谱象征某种人物形象的特点。《说唱脸谱》中“蓝脸的窦尔敦盗御马,红脸的关公战长沙,黄脸的典韦白脸的曹操,黑脸的张飞叫喳喳……”如果让你用黑脸谱来扮演《水浒传》中的一个人,你认为最适合于扮演________________,因为从小说“__________________”这一情节可以看出他的强悍、勇猛。

  (2)我们当地最流行的剧种有什么?请至少举出一种并写出代表作。

  ______________________________________________________________

  阅读下面的短文,按要求回答问题。

  激情、浪漫、梦幻,是对北京奥运开幕式的最好描述。导演们使用了一种*人、外国人都能看得懂的方式讲述了一个特别美丽的*故事,巨大的水墨文化卷zhóu____,数千张世界儿童的笑脸,用世界的语言传递古老神奇的*文化。当罗格的那句“无与lún____比”响彻北京时,我们看到的不仅仅是世界对*的赞美,更是世界对*的重新认识。当焰火、星光、*在鸟巢辉映之时,我却似乎看到数百年来中华民族的种种劫难,从《南京条约》到《辛丑条约》,从火烧圆明园到南京大屠杀,______________,我们用不屈、坚韧、勇敢、自信开创了中华民族的伟大复兴。北京奥运仅仅是开始,开幕式也只是一个宣言。作为*人,我无比自豪!

  (1)根据拼音写出汉字。

  卷zhóu____________ 无与lún________比”

  (2)请仿照画线的句子,根据文意,在横线上续写一句话。

  _____________________________________________

  (3)学校将举办一次“争当文明小市民,争做环保小使者”演讲比赛,如果你是主持人,请你为本次比赛写一段开场白(60字左右)。

  _________________________________________________

  四、文言文阅读

  阅读下面[甲] [乙]两组文言文,完成下列小题。

  [甲] 一箪食,一豆羹,得之则生,弗得则死。呼尔而与之,行道之人弗受;蹴尔而与之,乞人不屑也。

  万钟则不辩①礼义而受之,万钟于我何加焉?为宫室之美,妻妾之奉,所识穷乏者得我欤?向为身死而不受,今为宫室之美为之;向为身死而不受,今为妻妾之奉为之;向为身死而不受,今为所识穷乏者得我为之;是亦不可以已乎?此之谓失其本心。 (节选自《孟子·鱼我所欲也》 )

  [乙]曾子②之妻之市,其子随妻而泣。其母曰:“母还,顾反为汝杀彘③ 。”妻适市来,曾子欲捕彘杀之。妻止之曰:“特与婴儿戏耳。” 曾子曰:“婴儿非与戏也。婴儿非有知也,待父母而学者也,听父母之教。今子欺之,是教子欺也。母欺子,子而不信其母,非所以成教也!”遂烹彘也。

  (节选自《韩非子·曾子杀彘》 )

  [注]①辩,有的版本作“辨”。②曾子:曾参,孔子的学生。③彘:zhì,猪。

  1.解释下列句子中加点的字。

  ①呼尔而与之,行道之人弗受 与:________________

  ②所识穷乏者得我欤 得:________________

  ③妻适市来 适:________________

  ④妻止之曰:“特与婴儿戏耳。” 止:________________

  2.下列句子中加点词语的意义和用法与“行道之人弗受”中的“之”相同的一项是 ( )

  A. 曾子之妻之市 B. 妻止之曰 C. 曾子之妻之市 D. 今子欺之

  3.将下列文言语句翻译成现代汉语。

  ①万钟则不辩礼义而受之,万钟于我何加焉?

  译文:________________________________________________

  ②今子欺之,是教子欺也。

  译文:________________________________________________

  4.古往今来,舍生取义的志士仁人很多,请举出一个人物,并简要概括其事例。

  5.从“曾子杀彘”这个故事,可以看出曾子怎样的教子思想?

  五、诗歌鉴赏

  阅读下面一首诗,回答问题。

  酬乐天扬州初逢*上见赠

  刘禹锡

  巴山楚水凄凉地,二十三年弃置身。怀旧空吟闻笛赋,到乡翻似烂柯人。

  沉舟侧畔千帆过,病树前头万木春。今日听君歌一曲,暂凭杯酒长精神。

  1.“怀旧空吟闻笛赋,到乡翻似烂柯人”中运用典故,分别用来表现诗人怎样的思想感情?

  2.请描述“沉舟侧畔千帆过,病树前头万木春”所展现出的画面,并揭示诗句的含义。

  六、现代文阅读

  品读《*石拱桥》中的精彩语段,回答问题。

  永定河上的卢沟桥,修建于公元1189到1192年间。桥长265米,由11个半圆形的石拱组成,每个石拱长度不一,自16米到21.6米。桥宽约8米,路面*坦,几乎与河面*行。每两个石拱之间有石砌桥墩,把11个石拱联成一个整体。由于各拱相联,所以这种桥叫做联拱石桥。永定河发水时,来势很猛,以前*河堤常被冲毁,但是这座桥却从没出过事,足见它的坚固。桥面用石板铺砌,两旁有石栏石柱。每个柱头上都雕刻着不同姿态的狮子。这些石刻狮子,有的母子相抱,有的交头接耳,有的像倾听水声,有的像注视行人,千态万状,惟妙惟肖。

  1.本段的说明对象是_________ ,它是 _______________典型代表之一。

  2.指出文中划线语句所采用的说明方法,并分析其作用。

  3.“桥宽约8米,路面*坦,几乎与河面*行。”中的“约”“几乎”能否去掉?为什么?

  4.*的石拱桥很多,都有相同的特点,作者为什么选择赵州桥和卢沟桥作为说明的例子

  阅读下面的文字,完成下列小题。

  失败是个未知数

  ①你品尝过失败的滋味吗

  ②测验不及格,升学考试不进分数段,比赛取不上名次……这都叫失败,失败的滋味当然是苦的。

  ③人的一生,不可能永远一帆风顺,不可避免要遭受这样或那样的失败,只不过有的人栽跟头栽得多些,有的人栽得少些罢了。可以说,人生就是不断栽跟头,而又不断爬起来前行的漫长过程。

  ④失败的下一站是“痛苦”,但却不是终点站,而是岔道口。这岔道口岔出两条路:一条是心灰意冷,一蹶不振的路,这条路通向彻底的失败;另一条是汲取教训,再接再厉的路,这条路通向再失败或失败的反面--成功,但只有踏上这条路,才有成功的希望。因此,一个人遭到了失败,并不意味着这就是最终的结果。问题在于:站在"痛苦"这个岔道口的时候,自己选择的是哪一条路。

  ⑤十九世纪初,有一位英国将军在战场上吃了败仗,落荒而逃,躲进农舍的草堆里避风雨,又痛苦,又懊丧。茫然中,他忽见墙角处有一只蜘蛛在风中拼力结网,蛛丝一次次被吹断,但蜘蛛一次次拉丝重结,毫不气馁,终于把网结成。将军深受激励,后来重整旗鼓,终于在滑铁卢之役打垮了自己从前的对手拿破仑。这位将军,就是历史上赫赫有名的威灵顿。威灵顿将军的故事,不正好说明失败并不尽是最终结果这个道理吗

  ⑥失败并不可怕,痛苦并不可怜;只有怯懦才是可悲的。失败是一个未知数。当我们在失败之余,勇于化痛苦为动力,那么,就有可能从"痛苦"这个"岔道口",找到通向成功之路。我说的这番话,你认为对吗

  1.“失败是个未知数”中的“未知数”在本文中的意思是什么

  2.请用一句话概括第四段讲述的故事内容,并说一说作者讲述这个故事的目的。

  3.根据第四段的内容,再补写一个论据(事实论据与理论论据皆可)。

  4.本文是围绕什么问题展开论述的 结合你的实际,谈谈你对这个观点的看法。

  阅读下面文章,完成下列小题。

  我生命中的那簇野菊花 赵宁

  ①成长本是一个漫长的过程,历经岁月的不断锤炼打磨,才能由幼稚走向成熟,由怯弱走向勇敢。然而,有时候蜕变却会在瞬间完成,尤其是关键的那一步,就像作家或画家长久地苦思冥想,仍不得其法,而在某一境况下灵感顿生,一挥而就。

  ②我是母亲第四个孩子,出世没几天,便被送进医院。医生告知,孩子心脏发育不好,有肺炎、先天性气管炎,很难养活。而我的父母亲,始终不肯放弃我,一天天的打针吃药,细心的照管,使我终于幸存下来。

  ③我所能记起的是五六岁前后的事情。当别的孩子在村里像兔子一样欢快追逐时,我躲在医院的病床上;当一群孩子相约戏水时,我被关在家里;小伙伴们爬树、捉知了,我只能远远地看者。我常搬着小凳子,坐在家门口,看他们跳皮筋、打沙包,时常心生悲哀,我甚至自己一点也不喜欢自己。

  ④深秋的一天,我睡得迷迷糊糊的感到嘴里干渴,嚷着要喝水。父亲过来*惯地摸我的额头,“这么烫,又发烧了。”他迅速地给我穿戴好,叮嘱了母亲几句,就去推自行车,领我去镇上的医院。当我坐在父亲的自行车的后座的时候,才知道,此刻天刚蒙蒙亮,路上几乎没有人。深秋的早晨寒气袭人,父亲飞快地骑着自行车,我则昏昏沉沉地靠在他的背上。镇医院离家并不远,很快就支了,听诊、化验、取药、打针……这一系列过程,小小的我已是非常熟悉了……出了医院已是9点多了,小镇上早已热闹起来,父亲带着我往家赶,晴朗的天,阳光是金色的,照在一张张兴奋的脸上,我无力地靠在父亲的背上,什么也不想说,也不想看,我只觉得自己与这充满活力的景象格格不入,我像是被快乐遗弃的孩子,不知道自己还能不能长大。我无声地哭泣着,泪水将父亲的外套弄湿了一大片。他感到了什么,车子骑到前面一个转弯处忽然改变了方向,父亲对我说:“咱们抄*路能*点。”我从来不知道这条小路也能通向村子。说是一条小路,其实是灌溉渠的渠岸,这条水渠很深,也很宽,渠岸大约1米宽,并不是很*坦。我坐在后座上,觉得有点紧张,便坐直了身子,也没有了一丝困意。

  ⑤我的眼前忽然一亮,就在前方渠边斜坡上竟然有一大片一大片白色的花,在阳光的照耀下,开得那样鲜艳,那样精神。“爸爸,那是什么花?你放我下来吧。”我从自行车上跳了下来。父亲说:“这是白菊花,像是野生的。”我蹲在了路边兴奋地看着这些花,一丛丛,一簇簇,紧密地挨着,矮矮的,却精神抖擞,一些小虫在上面跳跃飞舞,花朵不大,可开得那样灿烂,没有一丝倦怠之意,露水还没有散去,在花朵上面打转。我已无法准确地描绘那个时刻小小的我的心境,我只记得那一刻有一幅画面深深地刻印在我脑海里:清凉的风,湛蓝的天,金闪闪的阳光,地上一大片白色的野菊花明艳美丽,清香袭人,一个瘦弱的小女孩看得痴迷,看得绽放了久违的笑颜。而就在那一刻,她坚定地告诉自己,她要生长得像这花一样美丽……

  ⑥那一天回家后,我破天荒头一回吃药没有让父母催促,那一大碗药,我第一次面无惧色,一饮而尽。只有我知道,我饮下去的是希望,是力量,是渴望生命如花绽放的动力,那一刻一个六岁的小女孩觉得自己长大了。

  ⑦多年以后,我终于摆脱了疾病,健康地长大,并没有留下一丝疾病侵扰的痕迹。如果说生命是一件完美的艺术品,那么在我生命的这幅画卷中,那路边的野菊花就该是神来之笔吧!

  1.读第⑤段中两处对野菊花进行描写的划线句,请从中分别找出表明野菊花外在和内在特点的词语。

  外在特点的词语:_____________________;内在特点的词语:______________

  2.第⑥段中写道,“破天荒头一回吃药没有让父母催促”,是一件什么事情让我发生了这样的变化?请你根据文章的内容用简洁的语言概括这件事。

  3.从全文看,我能够“幸存下来”并“健康地长大”的因素有哪些?

  4.下面对文章写法分析不正确的一项是: ( )

  A. 文章第①段先写自己对人生经历的回顾和感悟,由此引出下文对往事的叙述。

  B. 第③段划线句使用了对比手法,突出了“我”对正常儿童生活的向往和内心的悲哀。

  C. 去掉首尾两段,文章仍然结构完整,中心突出。

  D. 文章主要通过动作描写来表现父亲对我的关爱和照顾。

  5.在你的生命中,一定也有这样的“野菊花”给了你人生感悟,请你采撷记忆中的那片“野菊花”,将她和她给你的感悟一起写在下面。

  七、命题作文

  阅读下面的文字,按要求作文

  站在青春的.路口,回望成长的足迹,你是否发现,在成长的过程中,你所走的每一步,都有很多人相伴左右:含辛茹苦的父母、慷慨无私的朋友,循循善诱的老师,甚至擦肩而过的陌生人……请拿起手中的笔,写下你一路走来的感受吧! 请以“有你同行”为题写一篇文章。

  要求:①除诗歌、戏剧外,文体不限。②文章不少于600字 ;③文中不得出现真实的班级、人名等。

届九年级中考模拟语文试卷3

  一、选择题

  下列各组词语中加点字的书写或注音有误的一组是()

  A.狡黠(xié)雾霭 恹恹欲睡(yān)蓬首垢面

  B.尴尬(gān)褴褛拈轻怕重(zhān)诩诩如生

  C.魁梧(wú)褶皱重峦叠嶂(zhàng)钟灵毓秀

  D.慰藉(jiè)悲怆万贯家私(guàn)荒谬绝伦

  依次填入下面这段话空白处的词语,最恰当的一组是( )

  阅读优秀作家的优秀作品,________作品的语言,感受作品的思想、艺术魅力,不仅可以________大自然和人生的多姿多彩,还可以激发我们________自然、热爱生活的感情。

  A.品味体味珍爱

  B.品评体验珍惜

  C.品评体验珍爱

  D.品味体味珍惜

  下列各句中有语病的一项是( )

  A.*期武汉市*正在大力加强廉租房、公租房建设,用这种方式解决当前存在的房源不足、房价上涨的问题。

  B.湘、鄂、皖、赣四省地域相邻,山水相连,在非物质文化遗产的保护、传承等方面开展深度合作,既可整合旅游资源,也有助于形成极具特色的区域文化生态圈。

  C.为了更好地提高服务质量,我们必须坚持以人为本,最大限度地为旅客创造和谐的候车环境和人性化服务。

  D.在全球化成为世界发展大趋势的今天,只有学会欣赏不同的文化,才能实现包容互鉴的共赢。

  下列各句标点符号使用不规范的一项是( )

  A.食无定味,适口为佳。美好语言的标准必须遵循两条:有效,悦耳。在人人想着创新的年代,自觉认知与坚守语言的底线,或许才能让我们的母语故乡变得更加美好。

  B.蒲松龄尝言:“书痴者文必工,艺痴者技必良。”读好书,就等于把生活中寂寞的辰光换成巨大享受的时刻。

  C.把阅读作为一种追求、一种爱好、一种健康的生活方式,这样的人一定拥有开阔的精神版图;建设书香社会,不断提升全民阅读的品质,这样的国家一定拥有勃勃生机和巨大潜力。

  D.有人在纸质书中嗅到了书香,感受到了阅读审美;也有人将视野浓缩于方寸之间,同样找到了阅读的乐趣,在互联网时代,却非要固守黄卷青灯不可,未免显得有些迂腐。

  二、现代文阅读

  文化的生与死

  费孝通

  每个人都是有生有死的,但一个社会是可能不死的。“社会”为什么能长久存在?因为有“文化”。而文化是如何起作用的?是基于人的群体性。群体可以超越个体的局限。每个个体的人有生有死,但不同的人的生与死是有时间差的,生不同时,死不同刻,而不同时间生死的人,不同代际的人,会有共处的时间。在这段时间里,个体的人生经验、知识、感觉、发现、发明等等,可以互相交流,互相传递,可以变成别人的东西,保存在别人那里。一个人的生命会逝去,但是他一生的知识积累,不一定随他的生命结束而消失,它们会传递给继续活着的人,可以传递给很多人。这种不断传递,就成为社会很多人共同的知识即文化,保存在很多人的头脑中,形成一个不断增加的、动态的、分散的“信息库”,这个信息库又反过来不断塑造着新的社会成员的态度和行为,这就是文化的传承。同时,由于各种信息载体(石刻、竹筒、书本、磁带、光盘等)的存在,人们可以把知识记录下来,储存起来,留给后人,这样,即使一个社会真的消失了,一个文化中断了,但后来的社会,其他文明活着的人可以从那些很久以前死去的人那里学*各种知识——人和人可以跨越时间、空间的障碍,进行交流和学*,分享知识和经验。

  文化传承中,有很多这种跨越时间、空间继承的例子。比如我们今天经常说“西方文化来自古希腊罗马文明”,实际上,尽管古希腊罗马本来就属于欧洲,但他们的很多文化成就,并不是通过他们自己生物性的后人直接传到*代欧洲的,而是通过*人“转手”的。因为在中世纪,欧洲本身的很多古典文化的东西中断了,而这些东西保存在*人那里,后来“文艺复兴”,欧洲人不是从自己的前辈手里,而是从*人那里又“取回”了很多古希腊罗马人创造的知识。

  社会和文化可以使人“不朽”。像唐朝的诗人李白,是一个具有有限生命的人,而他保存在各种文献中的诗和诗的风格,则是“文化”。“人”是会消失的,但“文化”保留下来了,社会长存,文化不死,创造文化的人也就“不朽”了。一个人创造的文化不仅能保留,还能传递,还能影响别人,能激发别人的灵感,实现“再创造”,所以传统可以成为新文化生长的土壤。

  文化的传递,必须是一种历史过程,所有文化都必须是积累的,没有积累,没有超越生死、时空的这种积累,文化就不可能存在。

  从“个人和群体”的角度理解文化,“文化”就是在“社会”这种群体形势下,把历史上众多个体的、有限的生命的经验积累起来,变成一种社会共有的精神、思想、知识财富,又以各种方式保存在今天一个个活着的个体的生活、思想、态度、行为中,成为一种超越个体的东西。当一个新的生命来到这个世界上时,这套文化传统已经存在了,这个新的生命体就直接生活在其中,接受这种由很多人在很长时间里逐步创造、积累的文化,所以文化具有历史性。

  文化的历史性是广义的,不仅具体的知识和技能是在历史长河中积累传承的,更深层、更抽象的很多东西,比如认识问题的方法、思维方式、人生态度等,也同样是随文化传承的。进一步说,文化的传承,也同样包含了“社会”的传承。一个社会基本的结构,夫妻、父母、社区结构,都是文化的一部分,是先人传下来的,是晚辈向长辈、后人向前人学来的。学*,继承中不断有修正和创新,但只有在继承中才可能有创新。没有一个社会结构是完全凭空构建的,它总是要基于前一个社会结构,继承其中的某些要素,在此基础上建立新的东西。比如,即使像美国这样一个“人造”的国家,其社会结构也不是从美国建立时突然开始的,而是来自欧洲的移民从欧洲移植过去的,实际上是欧洲文化的延伸和变体。

  (费孝通《文化的生与死》有删改)

  1.下列关于“文化”的说法,不符合文意的一项是()

  A.如果没有人的群体性,文化就不会得以保存和传递。

  B.一个人可以“不朽”,因为有文化的作用;一个社会可以不死,因为有文化的存在。

  C.文化是社会共有的精神、思想和知识财富,与个体的人生经验、感受无关。

  D.欧洲的“文艺复兴”和美国社会结构的构建,其文化传承的具体方式是不同的。

  2.下列各项关于“文化的历史性”说法不正确的一项是()

  A.文化的传递是一种历史过程。

  B.个体接受在历史上逐步积累的文化。

  C.历史文化不仅包含具体的知识和技能在历史长河中的传承,还包含“社会”的传承。

  D.学*文化,继承文化中要不断修正和创新,但只有在继承中才能有创新。

  3.下列推断不符合文意的一项是()

  A.现代社会每个个体的思想和行为,都会受到前人思想的影响。

  B.一个国家失传的史料,有可能在其他国家的文献中找到。

  C.尽管社会面貌已有了巨大的变化,但在社会基本的结构上,我们今天与唐代并没有什么不同。

  D.传统的宣纸生产工艺,既是前人生产经验的总结,也体现了前人对客观世界的认知。

  三、文言文阅读

  阅读下面的文言语段,完成下面小题。

  汉六年,人有上书告楚王韩信反。高帝问诸将,诸将曰:“亟发兵坑竖子耳。”高帝默然。问陈*,*固辞谢,曰:“诸将云何?”上具告之。陈*曰:“人之上书言信反,有知之者乎?”曰:“未有。”曰:“信知之乎?”曰:“不知。”陈*曰:“陛下精兵孰与楚?”上曰:“不能过。”*曰:“陛下将用兵有能过韩信者乎?”上曰:“莫及也。”*曰:“今兵不如楚精,而将不能及,而举兵攻之,是趣之战也,窃为陛下危之。”上曰:“为之奈何?”*曰:“古者天子巡狩,会诸侯。南方有云梦,陛下弟出伪游云梦,会诸侯于陈。陈,楚之西界,信闻天子以好出游,其势必无事而郊迎谒。谒,而陛下因禽之,此特一力士之事耳。”

  高帝以为然,乃发使告诸侯会陈:“吾将南游云梦”。上因随以行。行未至陈楚王信果郊迎道中高帝豫具武士见信至即执缚之载后车。信呼曰:“天下已定,我固当烹!”高帝顾谓信曰:“若毋声!而反,明矣!”武士反接之。遂会诸侯于陈,尽定楚地。

  上还,至洛阳,赦韩信,封为淮阴侯。信知汉王畏恶其能,多称病,不朝从。

  (选自《史记·留侯世家》)

  1.下列对句中加点词的理解,不正确的一项是()

  A.*固辞谢固:坚决

  B.上具告之具:准备

  C.将不能及及:赶得上

  D.此特一力士之事耳特:只,仅

  2.为文中画波浪线的语句断句,正确的一项是( )

  行未至陈楚王信果郊迎道中高帝豫具武士见信至即执缚之载后车。

  A.行未至陈楚/王信果郊/迎道中高/帝豫具武士见信/至即执缚之载后车

  B.行未至陈/楚王信果郊迎/道中高帝豫具/武士见信/至即执缚之/载后车

  C.行未至陈/楚王信果郊迎道中/高帝豫具武士/见信至/即执缚之/载后车

  D.行未至陈楚/王信果郊迎道中/高帝豫具/武士见信/至即执缚之载后车

  3.下列对文本内容的理解,不正确的一项是( )

  A.有人上书告发韩信谋反,陈*认为不可攻打韩信,因为诸将中没有谁打得过韩信。

  B.刘邦听从了陈*的意见,假装出游,召见韩信,从而将他抓获。

  C.刘邦到洛阳后赦免了韩信,但把他从楚王降为淮阴侯。

  D.韩信被赦免后,常假称生病不参加朝见,因为他知道刘邦厌恶自己的才能。

  4.将第三大题文言文材料中画横线的句子翻译成现代汉语。

  举兵攻之,是趣之战也,窃为陛下危之。

  四、名句名篇

  填空。

  (1)一鼓作气,再而衰,__________。(《左传·曹刿论战》)

  (2)_____________,千里共婵娟。(苏轼《水调歌头·明月几时有》)

  (3)出师一表真名世,____________。(陆游《书愤》)

  五、现代文阅读

  年年依旧的菜园

  迟子建

  (1)外祖母家有一片菜园。春天一到,最先种上的是菠菜、生菜和白菜,之后种香菜、水萝卜和土豆,再之后种那些爬蔓的植物:豆角、倭瓜、黄瓜等。当然,如果弄到茄子秧、柿子秧、辣椒秧,它们也一定会被恰到好处地栽种在园子里,那时候菜园中的蔬菜品种可就丰富多了。时间一到,菜园里黄瓜顶花带刺,或翠绿细长或粗壮脆嫩,鲜亮得让人垂涎欲滴;一嘟噜一嘟噜的豆角从叶片的缝隙间垂挂下来,白里带紫的豆角花默默地绽放。肥硕的茄子挤满秧枝,紫嘟嘟的像脸蛋似的彰显笑容;壮实、疯长的倭瓜秧爬满了整个栅栏,形成了一道绿色的墙,黄色的喇叭形花朵正芬芳吐蕊呢:好一派生机盎然的景致。

  (2)太阳刚下山了,菜园中还散发着阳光留下的余温,待到月亮升起的时候,菜园完全是另外的景致了。分不清哪里是花,哪里是菜,只是见月光像泉水一样倾泻下来,把那些开花的不开花的植物全都镀上一层银光。这时候蜜蜂和蝴蝶都不见了,只是听得见水边青蛙的叫声,像是在歌颂月夜下菜园的美景。而当天色微明、菜园种的植物沾染了浓重的露水、太阳忽然跃出山顶将露珠照散的时候,农人们也就下田干活了。

  (3)如果看厌了菜园的景致,当然还可以走出园子到自留地去。自留地的面积可要比菜园大多了,它大多种苞谷和麦子。我喜欢啃青苞谷吃,那滋味甜丝丝的,感觉是在吃糖,可又比糖的味道柔和多了。而我喜欢麦子并不喜欢它的果实,我喜欢麦芒,那些像胡茬子一样的麦芒可以用来挠痒痒。

  (4)外祖母对外祖父说:“你去给园子锄锄草。”我便跟着外祖父到园子中锄草。外祖父对外祖母说:“你去园子里给我弄点葱来蘸酱。”我便跟着外祖母到园子中拔葱。

  (5)我常常在帮助外祖父锄草的时候将苗也锄了下来,我也往往在帮外祖母拔葱的时候将葱根断在土里。我总是帮倒忙,但外祖父和外祖母从不责备我,我是太爱菜园了。

  (6)外祖父和外祖母都是农民。农民是土地真正的主人。我扯着外祖父的手时感觉那手是粗糙而荒凉的,我扯着外祖母的手时感觉那手也是粗糙而荒凉的。外祖父摆弄那些农具的时候我便也跟着摆弄,外祖母给地施肥时我便也跟着施肥。

  (7)我不喜欢谷子。外祖母就说:“谷子是粮食啊,人是靠它才活命的啊。”我就渐渐喜欢上了谷子。外祖父说:“别小看我这片菜园和自留地,它可以养活城里的几十条人命哪。”

  8)我便知道城里其实是个很贫乏的地方。

  (9)外祖母告诉我,我生活的地方就是农村,我便知道农村是广大的,我也知道那些菜地和麦田都是农民的**子。我跟着他们用双手学会了打垄、锄草、间苗、施肥和收割,所以直到如今我的手仍然缺乏女性的细腻和柔美,它们同样是粗糙而荒凉的。

  (10)当我的双手远离那些农具的时候,我就很自然地用手拿起笔回忆那些让人感到朴实和亲切的消逝了的日子。回忆那菜园,菜园里的蚂蚱和蜻蜓,却无法描绘出它们翩然的姿态;回忆麦田,丰收后有稻草人屹立在麦田里的情景,却难以企及那岿然神韵。我的文字是粗糙而荒凉的。

  (11)当我坐在城市的.咖啡厅里听着那些饱食终日的人发着空虚的牢骚,我便会想到外祖父劳累一天后吃罢晚饭沿着菜园散步的情景。外祖父呼吸着真正的空气,所以无论在他生前或死后,他的睡眠都是安详的。如今他在他种过黄豆和玉米的土地上安息了。

  (12)外祖母依然健在,她仍然用她粗糙而荒凉的手忙碌在菜园里。外祖母种的菜外祖父如今是吃不到了,就由她的儿孙来吃,而到了她的儿孙也吃不到了的时候,外祖母肯定早就不在人间了。而菜园总要有人种下去。人一代代地老去,菜园却永远不老。

  (13)冬天来了。冬天来了的时候菜园就被雪覆盖了。那些好看的蚂蚱和蜻蜓不见了,那些花和碧绿的菜蔬也都死灭了。白雪覆盖着生长过茂盛植物的土地,白雪同样覆盖着为耕种这些植物而死去了的人的灵魂,那些寂寞而宽厚的依附着土地的灵魂。

  1.作者在第(1)段分别描写了外祖母家菜园的哪些特点?请简要概括。

  2.依据上下文,解释下面句子中加点部分在文中的语意。

  (1)我的手仍然缺乏女性的细腻和柔美,它们同样是粗糙而荒凉的。

  _____________________________________________________________

  (2)我的文字是粗糙而荒凉的。

  ________________________________________________________________________________

  3.文中“人一代代地老去,菜园却永远不老。”联系文本说说怎样做才能让菜园“不老”?

  4.概括本文所写的中心内容和作者在文中表达的主要思想感情。

  六、综合性学*

  根据要求完成下面小题。

  为让学生树立正确的科学观,让科学为人类造福,促进人类幸福,而不致成为祸害。语文老师准备带领同学们开展一次关于主题为“现代科技给我们带来了什么?”的综合性学*活动。

  在活动过程中,第一小组想了解现代科技对社会发展的影响,请你为小组草拟三个研究选题。

  在活动过程中,第二小组开展辩论赛的形式让同学们了解现代科学技术对我们的影响。

  辩论会上,甲方队员说:“如果没有电脑和网络,就不会有青少年因迷恋上网而厌学。所以,网络的普及与发展,不利于青少年的茁壮成长。”如果你是乙方辩手,会怎样反驳呢?

  乙方队友说:“_____________________________________________”。(120字以内。)

  七、材料作文

  阅读下面的材料,按要求作文。

  1973年,科莱特和比尔·盖茨同时进入大学。大二,科莱特本想和比尔·盖茨一起创业,但科莱特想到自己没有学位,就放弃了。比尔·盖茨却在心里规划好了自己未来的路,最后他毅然退学选择创业。

  1977年,比尔·盖茨开发出了软件,注册了自己的公司,比尔·盖茨再次邀请科莱特加入自己的商业帝国。但科莱特觉得自己的知识不够,再次谢绝了邀请。这时比尔·盖茨已经开始计划如何让公司上市的问题。不久,比尔·盖茨拥有了数十家上市公司。

  1983年,科莱特成为计算机系的博士。可就在这一年,比尔·盖茨进入了《福布斯》亿万富翁排行榜。

  生活中,我们需要排除心中顾虑,落实行动,才有机会收获成功。

  请你根据对上述文字的理解和思考,写一篇文章。

  要求:依据材料的整体语意立意,自拟标题,不少于600字。文中如果出现真实的姓

  名或校名,请以化名代替。


届高三语文试卷 (菁华3篇)(扩展4)

——三年级上册语文期末试卷3篇

  班级: 姓名: 等级:

  第一部分: 基础知识积累与运用

  一、读拼音,写词语。

  zī shì nà mèn xīn shǎnɡ shì yàn sū xǐnɡ

  ( ) ( ) ( ) ( ) ( )

  zhǔn bèi bō luó kuān yù jiān ɡù xiū jiàn

  ( ) ( ) ( ) ( ) ( )

  二、比一比,再组词。

  辫( ) 推( ) 侧( ) 据( ) 插( )

  瓣( ) 堆( ) 测( ) 锯( ) 播( )

  娇( ) 壮( ) 姿( ) 糖( ) 郊( )

  骄( ) 状( ) 资( ) 塘( ) 胶( )

  三、词语填空。

  1、在下面括号里填上合适的字,组成成语。

  ( )然大( ) ( )颜( )色 高( )大( )

  没( )打( ) 日( )兼( ) ( )不( )传

  ( )不转( ) 相( )并( ) 千( )万( )

  2、按要求写四字词语。

  ①栩栩如生(AABC式) 、 、

  ②四面八方(含有数字的) 、 、

  ③黑糊糊 (ABB) 、 、

  ④确确实实(AABB) 、 、

  ⑤又细又长(又A又C) 、 、

  3、词语搭配

  匆匆地( ) ( )的世界 水绕( )

  轻轻地( ) ( )的石级 大陆( )

  快活地( ) ( )的铁罐 山花( )

  四、查字典填空。

  “端”字用音序查字法,应先查大写字母( );用部首查字法,应先查部首( )。它在字典中的解释有:①头;②开头;③项目;④端正;⑤*举拿着。“变化多端”中的“端”的意思是( );“五官端正”中的“端”的意思是( )。(填序号)

  五、古诗句、名句展示台。

  (1)明月松间照, 。

  (2) ,水村山郭酒旗风。

  (3)欲把西湖比西子, 。

  (4)*青山相对出, 。

  (5)五岳归来不看山, 。

  (6) ,眼见为实。

  (7)玉不琢, 。 , 。

  六、按课文内容填空。

  (1)本学期,我们学*了《刻舟求剑》、《 》、《 》等成语故事,还学*了《女娲补天》、《 》等神话故事。

  (2)这学期,我看了许多课外书,其中我最喜欢的是《 》,因为 ,所以我喜欢这本书。

  七、句子练*。

  (1)在原句上修改病句。

  A.学校组织我们观察了一部有教育意义的电影。

  B我们全校师生和校长都参加了这次活动。

  (2)按要求改写句子。

  A.漳河发大水,把天地淹没了。(改写“被”字句)

  B.这美丽的奇观怎么不是真的呢?(改成陈述句)

  C树叶落下来。(把句子写具体)

  第二部分:快乐阅读。(阅读短文,完成后面的作业。)

  (一)《和时间赛跑》

  时间过得飞快,使我的小心眼里不只是着急,还有悲伤。有一天我放学回家,看到太阳快落山了,就下决心说:“我要比太阳更快回家。”我狂奔回去,站在庭院里喘气的时候,看到太阳还露着半边脸,我高兴地跳起来。那一天我跑赢了太阳。以后我常做这样的游戏,有时和太阳赛跑,有时和西北风比赛,有时一个暑假的作业,我十天就做完了。那时我三年级,常把哥哥五年级的作业拿来做。每一次比赛胜过时间,我就快乐得不知道怎么形容。

  1. 从短文中找出下列词语的近义词和反义词。

  快乐 近义词:( ) 反义词:( )

  2、用“ ”画出文中拟人的句子。

  3、填空。

  “我”小心眼里“着急”、“悲伤”的原因是( )。

  “我”快乐、高兴的原因是:( )。

  (二)我家的小园

  我家的东南边有一块长方形的小园,里面种着各种蔬菜、花草和树木。这里一年四季都是美丽的。

  春天到了,大地像铺上了一条绿毯子。园里一片绿油油。一群小鸟停在树枝上,叽叽喳喳地叫个不停。静静的小园开始热闹起来了。

  夏天,成行的向日葵长(的 地 得)茁壮,一朵朵金黄色的花,迎着朝阳怒放。火红的石榴花散发着阵阵芳香,引来一群群蜜蜂。一排排玉米威武地挺立着,结出了一个个玉米棒子。地里,茄子紫了,西红柿红了,顶花带刺的黄瓜挂满了架。望着这些丰硕的果实,我感到小园的可爱。

  秋天,喇叭花开了,在绿叶丛中伸出了一只只红色的小喇叭,正在为小蜜蜂吹奏着动听的歌。石榴树上的石榴熟了,红色的果皮裂开了,露出了珍珠般的颗粒,迎着我咧嘴傻笑。这一切怎能不惹人(喜爱喜欢)?

  冬天,一场大雪过后,石榴树披上了白雪,地上也盖上了厚厚的白雪,整个小园成了白雪的世界。我情不自禁地赞叹道 好美 小园

  我家的小园如此(骄 娇)美,真是一幅诱人的四季画。

  1、划去括号内不恰当的字、词。

  2、联系上下文解词。

  丰硕:______________________________________

  3、给第5自然段的空白处补上标点。

  4、文章是围绕哪句话写的?用“ ”画出来。

  5、文章分别从( )、( )、( )、( )几方面描写小园的( )。

  6、文中描写了好几种花草树木,你最喜欢哪一种?为什么?

  ____________________________________________________________

  第三部分:*作天地

  可以写各种各样的人或事,可以写自己喜欢的景或物,可以写自己的快乐或烦恼……写的时候,先想一想,要把想表达的意思写清楚。题目自拟。

  一、 看拼音写词语。

  qí zhì

  yān mò

  bái fèi

  liú tǎng

  wēi xiǎn

  二、 给下面带点的字选择正确的读音(正确的打√)。

  暑假(jià jiǎ)  磨刀(mò mó)

  挨打(āi ái)   空白(kòng kōng)

  三、 比一比,再组词。

  移( )

  宜( )

  探( )

  叹( )

  迅( )

  讯( )

  努( )

  怒( )

  喘( )

  端( )

  四、给下列词语选择正确的解释(在括号里打√))

  欣赏   ①奖赏( )  ②观赏( )

  突围   ①忽然( )  ②冲破( )

  勤劳   ①勤务( )  ②尽力多做( )

  五、 用直线把下面的词语搭配起来。

  成群结队的   树叶

  密密层层的   大海

  又松又软的   小鸟

  无边无际的   雪花

  六、 选词填空。

  (1)“110”的警察叔叔( )地抢救受伤的群众。

  争分夺秒   争先恐后

  (2)我( )王小华学*成绩赶上来,因为他现在变得( )时间,发愤学*了。

  爱惜   珍惜   发现   发明

  (3)我看到老奶奶( )到在地上,就立刻( )过去,把她( )起来,( )去她身上的尘土,帮她( )起掉在地上的东西,( )她有没有受伤?

  拣   扶   摔   跑   问   拍

  一、 看拼音写词语。

  二、 给下面带点的字选择正确的读音(正确的打√)。

  暑假(jià jiǎ) 磨刀(mò mó)

  挨打(āi ái) 空白(kòng kōng)

  三、 比一比,再组词。

  四、给下列词语选择正确的解释(在括号里打√))

  欣赏 ①奖赏( ) ②观赏( )

  突围 ①忽然( ) ②冲破( )

  勤劳 ①勤务( ) ②尽力多做( )


届高三语文试卷 (菁华3篇)(扩展5)

——上海高三语文作文3篇

  *是四大文明古国之一,自古以来道德就是人们为人处事的行为准则。与此相反,西方国家却用规则作为人们与人处世的行为准则。历史的列车运转到了今日,我认为,应让道德与规则并行。

  中华文明源远流长,从古至今都是以道德着称的文明国度。古时的“君子不乘人之危”“君子不夺他人之爱”和今天的“爱岗敬业”“诚实守信”等都是很好的道德信条,正因如此,*才被称为礼仪之邦。西方文明虽不及*文明博大精深,但其规则意识则为世人所褒扬。有些西方国家在建国之初,就在社会生活的方方面面规定和许多法律规则,西方人也形成了浓厚的遵守法规的氛围。

  一次,几名*学生到加拿大体验生活。当他们同加拿大同学一起去餐厅吃饭时,*学生看到学生打饭窗口前拍着长长的队伍,而教师打饭窗口前却空无一人。于是,他们很快跑到了教师打饭窗口前,早早的盛到了饭,而加拿大学生却仍排着队等待。有人问加拿大学生为什么不到教室窗口盛饭,加拿大学生说是学校校规有规定,学生不能到教室窗口去打饭。在他们看来,规则就是用来遵守的,没有规则就没有道德。

  哈佛大学的图书馆藏书几十万,学校规定学生只能在图书馆里看书,不得外借,若违反规定,一律开除。一名学生曾偷偷把一本书偷偷带出了图书馆。后来一场大火把图书馆的藏书都烧毁了,那名学生意识到他手中的书本是的幸存品,于是他把它交给了学校。学校非常感谢他,接受了书本,却把他开除了。因为他违反了校规。他们大规则看得高于一切,违反了校规就是失去了道德。

  由此看来,我们在保留传统的中华文明的同时,还要接受西方文明的规则意识。其实,在现实社会中,道德与规则是紧密相联的。如果说规则是人的骨骼,那么道德就是血和肉,这二者合一才能构成一个活生生的人。没有规则的社会,将十分混乱;没有道德的社会,人与人之间缺乏最起码的信任,整个社会将失去安全感。道德与规则是密不可分的,有了道德,才会遵守规则;遵守了规则,才会拥有高尚的道德修养。

  让道德与规则并行吧!

  我一翻开《海市蜃楼》这本书,就被它优美的、诗意的、感动心灵的语言深深震撼了,它让我们感悟到了道德的`力量、审美的力量、艺术的力量、语言的力量,滋养着我们少年儿童精神生命的成长。

  第一篇文章《大青石圆梦记》映入我的眼帘,我带着好奇之心读完了整篇文章。

  它讲了这样一个故事:在荒郊野外待了很长时间的大青石无意中听孩子们说自己向一条鲸鱼,便开始打听变成鲸鱼的方式,并按照这些方法努力去做,但当她发现如果要实现他的愿望便会给更多的生命带来痛苦时,他放弃了自己的理想。虽然没有成为一条大海中真正的鲸鱼,但他成了一条可以和孩子们一起玩耍的“鲸鱼”,世界上唯一一条可以接受孩子们最热情地包围和拥抱的“鲸鱼”。这样的变化足以让大青石心满意足,因为那是更适合他的空间。

  童话里的主人公似乎都有些不切实际的想法,但是只要经过努力,虽然会历尽千辛万苦,他们最终总能实现理想和愿望。这样的经历给了我们莫大地鼓舞和激励。

  我们每个人都有自己美好的理想,有的想当科学家,有的想当作家,有的想当医生……但都要经过自己的努力,付出汗水和心血,并且要向同一个目标奋进,这样才能够实现我们美好的理想。我也有一个美好的理想就是当一个宇航员,因为可以遨游太空,探索宇宙的奥秘。火星上可不可以住人?如果地球上的人多了能不能移居到月球上?紫外线为什么会越来越强烈?这些稀奇古怪的问题经常萦绕在我的脑海中,它鞭策着我:现在要努力学*,多看科普书籍,丰富自己关于天文地理方面的知识。

  然而,像大青石这样,为了保护他人,保护更多的生命而放弃了理想和愿望,这样的行为更令我们感动与崇敬。

  生活中,这样的事例不是很多吗?我们的妈妈就是这种任劳任怨、为自己的孩子甘愿放弃青春和梦想的人。她们为了抚育我们,简直操碎了心。

  而有些人为了个人的利益,却损害了大多数人的利益。比如,在车里,一

  些人吃完香蕉,只图方便,随手把香蕉皮向窗外扔了出去。一旦行人不注意,就会滑到。他们只注意自己位置的洁净,却破坏了环境。试想这样的行为难道不令人厌恶吗?

  其实,我们也能像这块大青石一样,为了保护他人,而放弃了自己,这

  样的行为最让人感动和敬佩。

  没有规矩,不成方圆。规矩就是规则,规则就是规矩。考场上有考场规则,运动场有运动规则,在斑马线上也有交通规则。过斑马线不能闯红灯,这种规则应该是所有人都要遵守,可还是有许多人选择在生命线上进行冒险。

  “*式闯红灯”,这被称为是最自由的过马路方式,马路两边,只要凑够一撮人就可以走了,和红灯绿灯无关,反正目标大,汽车肯定不敢撞过来。正是因为这种观念,我国每天都有人在斑马线上发生交通事故,安全线成了危险线。

  这回,我也成为了违反交通规则中的一人。我和妈妈在郑州游玩时,当我和妈妈到达路口,那里已经聚集了许多人,而这时,左右几乎没有汽车在通行,似乎空气静止了。人们似乎都在想:是闯过去,还是等一等这三十秒,不跨越这条规则红线。

  终于,一位有点微胖、身穿白色衬衫的男士敢为天下先,径直穿过人群,走向马路对岸。接着,人群松动了,我们都开始了“*式过马路”,无视红灯,鱼贯而出。我冲出绿化带,脖子扭来扭去观察身旁的汽车,踏着大步,挥舞双手,昂首挺胸,如同**仪仗队走过—广场。突然,一个黑色的身影从我面前呼啸而过,我吓得呆住,只感觉有一只手把我拉过了马路,我到现在都不记得,那个黑影,是小汽车还是卡车。这次的闯红灯经历给我敲响了警钟,也让我明白规则的重要性。

  回到家乡仙桃,我依然看到了许多“*式过马路”的现象。那天,我放学来到马路旁,和许多人一起等待绿灯亮起。我看到了身边这样一位大姐,她左右看看,毫不犹豫地离开马路边沿的台阶,准备带头闯过红灯。她已经走过了三格斑马线,前面还有十几格,但是,我没有动,我身边的其他人也没有动。这位大姐停下了,十分犹豫,但还是没有退回来。就在这时,左右两边的车来了,没有一辆减速,大姐只好在车流中尴尬地站着,多危险啊!可是,等车流一断,她还是突然加速,跑到了对面。红灯还有大概三四秒,我身边的这些人终于忍耐不住了,一起阔步走向对面,而我却一直等绿灯亮了,我才抬脚。我在心中为那些越过规则红线的人而羞愧,他们不仅拿自己的生命开玩笑,还拿他人的生命开玩笑。他们想过后果的可怕吗?

  回首往事,我看到用自己生命闯红灯、越过规则的人,实在是屡见不鲜,我想到一句让人深思的话:治标不治本。光靠红绿灯的提醒,光凭*人自己的自觉,这些表面约束工作是做不到让“*式过马路”遁形的。那么,我们该如何强化规则意识,从根本上解决“*式过马路”呢?


届高三语文试卷 (菁华3篇)(扩展6)

——六下语文试卷答案3篇

  一、抄写句子。做到笔画正确,结构匀称,书写工整,行款整齐。(2分)

  什么是路,路是从荆棘的地方开辟出来的,从没有路的地方踩出来的。

  二、看拼音写词语。要求把每个字写端正、匀称。(5分)

  duàn liàn fěi cuì kǔ xíng chōu yē méi guī

  ( ) ( ) ( ) ( ) ( )

  三、按要求写词语或句子。(19分)

  1.把词语补充完整并选择合适的词语填在下面的空格中。(6分)

  ( )拥而至 集思( ) 炎( )子孙 ( )无虚*

  ( )有趣味 发愤( )强 司空( ) ( )旁通

  2010年5月1日,第41届世博会在*上海隆重开幕。这是亿万 的骄傲。在此之前各地举办的与世博会有关的讲座都是 。开幕后,黄浦江边240多个的展馆,更是吸引了世界各地的游客 。

  2.根据提示,写出带有“海”字的成语。(2分)

  (1)泛指全国各地: (2)比喻危险之地:

  (3)形容东西难找: (4)比喻没有消息:

  3.用“然”组词,分别填在括号里。(3分)

  (1)客家民居和傣家竹楼各有特点,建筑风格( )不同。

  (2)两三百人聚集在一个会议室,秩序( ),毫不混乱。

  (3)那一座座碉堡,经历无数次地震撼动和风雨侵蚀而( )无恙。

  4.用恰当的关联词把下面三个句子连起来。(2分)

  桃花心木已经适应了环境 种树的人不再来了 桃花心木不会枯萎了

  5.修改病句。(2分)

  止咳祛痰片的主要成分是远志、桔梗、贝母、氯化铵等配制而成的。

  6.根据“味道”一词的不同含义,写两句话(2分)

  7.用最恰当的句式改写下面这句话。(2分)

  为了生活,九岁的凡卡来到莫斯科当学徒。

  四、按课文内容或要求填空。(19分)

  1.“一人虽听之, ,思援弓缴而射之。”句中的两个“之”分别指 、 ;后一人的学*态度我们可以用 、等成语来形容。

  2.太阳他有脚啊, 地挪移了;我也 跟着旋转。

  3.“‘ ’这一条意见,就是党外人士 先生提出来的,他提得好,对人民有好处,我们就采用了。”这里用了 论证的方法,说明__________。

  4.《卖火柴的小女孩》作者是 ,他被誉为“ ”。我还读过他写的童话 。

  5.名篇名著总能给我们带来人生的启迪,《百年孤独》告诉我们诚信的重要,书中那句话说得好“ 。”

  6.《浣溪沙》这首词中,最能看出词人不服老,旷达乐观的句子是:____________。

  7. 通过本册课文的学*,我感受到了老北京春节的 ;认识了流落荒岛,

  的鲁滨孙;还知道了镭的母亲就是居里夫人,她那 的科学精神真让我敬佩。(填入四字词语)

  五、阅读分析,完成练*题。(22分)

  (一)句子(3分)

  “这点美丽的淡蓝色的荧光,融入了一个女子美丽的生命和不屈的信念。”

  这里的“淡蓝色的荧光”指的是 ,说它融入了“美丽的生命,是因为 ,这句话是对居里夫人 的赞颂。

  (二)片断(4分)

  《十六年前的回忆》(片断)

  在法庭上,我们跟父亲见了面。父亲仍旧穿着他那件灰布旧绵袍,可是没戴眼镜。我看到了他那乱蓬蓬的长头发下面的*静而慈详的脸。

  “爹!”我忍不住喊出声来。母亲哭了,妹妹也跟着哭起来了。

  1.用修改符号在原文上改正两个错别字。

  2.片段中的“父亲”指的是 ,通过 和 描写,表现了父亲的革命精神。

  3.法庭上的父亲让我想到了一句诗:

  (二)课外短文(15分)

  改变一生的闪念

  ①一个关于我的老师——李老师的故事,珍藏在我心里,久久不能忘怀。

  ②多年前的一天,李老师正在家里睡午觉,突然,电话铃响了,她接过来一听,里面传来一个陌生粗暴的声音:“你家小孩偷书,被我们抓住了,你快来!”话筒里传来一个小女孩的哭声和七嘴八舌的呵斥声。

  ③李老师回头看着正在睡午觉的惟一的女儿,心中立刻明白了是怎么回事。肯定是有个女孩偷书被售货员抓住了,又不敢让家里人知道,所以胡说了一个电话号码,却碰巧打到这里。她本可以放下电话不理,甚至也可以斥责对方,因为这件事和她没有任何关系。 但通过电话她隐约想到,那个一念之差的小女孩,现在一定非常惊慌害怕,正面临着也许是人生中最困难的境地。于是,她问清了书店的地址,匆匆忙忙赶了过去。

  ④正如李老师预料的那样,书店里站着一位满脸泪痕的小女孩,旁边的售货员正恶狠狠地大声斥责着她。李老师一下子冲上去,将那个小女孩搂在怀里,转身对售货员说:“有什么事就跟我说吧,我是她妈妈,不要吓着孩子。”在售货员不停的责备声中,她交清了罚款,领着这个小女孩走出了书店。看着那张被泪水和恐惧弄得一塌糊涂的脸, 她笑了笑,将小女孩领到家中,好好清理了一下,什么都没有问。小女孩临走时,她特意叮嘱:如果你要看书,就到阿姨这里来,阿姨有好多书呢。 惊魂未定的小女孩,深深地看了她一眼,便飞一般地跑掉了,从此再也没有出现过。

  ⑤一晃十几年过去了,一天中午,门外响起了一阵敲门声。李老师打开房门,看到一位年轻漂亮的陌生女孩,满脸笑容,手里还拎着一大堆礼物。“你找谁?”她疑惑地问。女孩激动地说出了一大堆话。好不容易,李老师才从陌生女孩的叙述中恍然明白,原来她就是当年那个偷书的小女孩,大学毕业了,特意来看望自己。女孩眼睛泛着泪光,轻声说道:“当年情急之下的那个电话,幸亏打到您的家里。虽然我至今都不明白,你为什么愿意充当我的妈妈,帮我摆脱困境,但这么多年来,一直好想喊您一声妈妈。”望着女孩脸上幸福的笑容,李老师也笑了。

  1.请结合上下文,说说下列词语的意思。(4分)

  一念之差:

  惊魂未定:

  2.文章第②段写道“里面传来一个陌生粗暴的声音”,在第④段有两处描写与“粗暴的声音”相照应,请找出来写在下面。(2分)

  3.概括短文主要内容。(要求在80字以内)(3分)

  4.从第④段中画“ ”的句子中,你体会到了什么?(2分)

  5.文中有两处地方写到了李老师的“笑”,这两次笑的原因分别是什么?(2分)

  第一次笑(第④段)

  第二次笑(第⑤段)

  6.女孩说:“……这么多年来,一直好想喊您一声妈妈。”句中“妈妈”一词表达了女孩怎样的情感?(2分)

  六、开放题。(3分)

  1.从下面几个事物中任选一个,用恰当的诗句赞美它。

  西湖 春雨 柳树 友情

  ___________________:

  2.明代解缙常以对联嘲讽权贵。一日,一权贵出上联讥笑解缙,解缙不慌不忙予以反击。二人用谐音双关互嘲,请你写出谐音的字。

  上联:二猿断木深山中,小猴子也敢对锯?

  下联:一马陷足淤泥内,老畜生怎能出蹄?

  对联中的“锯”、“蹄”分别与 、 二字谐音。

  七、语言实践。(30分)

  1.口语交际。(3分)

  在《改变一生的闪念》一文中,李老师将小女孩领回家后,“什么都没有问”,就让小女孩离开了;如果当时李老师对其进行严肃批评教育后再让她走,教育效果会不会更好呢?你赞成哪种做法,说明理由。

  2.仔细观察漫画,用简洁的语言写一写你的想法。(2分)

  3.写作。(25分)

  在我们的成长过程中,既有阳光雨露的呵护,也有风霜雨雪的相伴,它们都是人生的一大财富。幸福和谐的生活,让我们品评甜蜜,懂得了珍惜;苦难艰辛的处境,使我们志怀高远,实现了超越。在经历和体验中,我们逐渐摆脱了蒙昧、自私、幼稚……我们长大了。

  请你以“在 中成长”为题,写一篇不少于400字的作文。(横线上可以填“逆境”、“磨炼”、“呵护”、“关爱”等。)

  要求:

  ③ 题目补充完整; ②写清楚事情的经过,内容具体,重点突出;

  ③有自己的真情实感; ④不出现真实的校名、姓名。

  答案

  一、略 。

  二、锻炼 翡翠 苦刑 抽噎 玫瑰

  三、1.( 蜂 ) ( 广益 ) ( 黄 ) ( 座 ) ( 饶 ) ( 图 )( 见惯 ) ( 触类 )

  炎黄子孙 座无虚* 饶有趣味 蜂拥而至

  2.五湖四海 刀山火海 大海捞针 石沉大海

  3.截然 (或“迥然”) 井然 安然

  4.虽然种树的人不再来了,但是桃花心木也不会枯萎了,因为桃花心木已经适应了环境

  5.删除“配制而成的”

  6.“味道”一词含义有(气味;滋味;氛围,气氛),意思正确即可。

  7.为了生活,九岁的凡卡不得不来到莫斯科当学徒。

  四、1.一心以为有鸿鹄将至 弈秋的教导 天鹅(鸿鹄) 三心二意 一心二用

  2.轻轻悄悄 茫茫然

  3.精兵简政 李鼎铭 举例 我们接受任何人只要是正确的批评

  4.安徒生 童话故事大王 《海的女儿》《丑小鸭》《皇帝的新装》《拇指姑娘》等

  5.守信是一项财宝,不应该随意虚掷。

  6.谁道人生无再少?门前流水尚能西!休将白发唱黄鸡。

  7. 热闹非凡 积极乐观 锲而不舍(恰当即可)

  五、(一)居里夫人用三年零九个月的时间提炼出来的0.1克镭发出的光;这项成就的取得是以居里夫人终日的烟熏火燎、身体的疲劳为代价获得的;坚持不懈的科学精神

  (二)1.“绵”改“棉”;“详”改“祥”

  2.* 外貌 神态 坚贞不屈(视死如归、大义凛然等)

  3.千磨万击还坚劲,任尔东西南北风

  (三) 课外短文(18分)

  1.一念之差:指在事情的发展过程中,由于思想上的一点偏差,结果产生了决定性的改变。

  惊魂未定:指受惊后心情还没有*静下来。

  2.旁边的售货员正恶狠狠地大声斥责着她;在售货员不停的责备声中

  3.李老师帮助了一个因“一念之差”而偷书的小女孩,使她感受到了温暖,从而改变了她的一生。

  4.体会到了李老师善良、高尚、有责任心,还有她的善解人意。

  5.第一次笑:为了减轻小女孩的恐惧和愧疚感。(或者为了安慰小女孩)


届高三语文试卷 (菁华3篇)(扩展7)

——初三语文试卷质量分析3篇

  反省是我们温故知新的基础,是我们学以致用的标准。

  一、试题分析:

  本套试卷的考试范围,难易适中,重点突出,考察全面,与中考题型较为吻合,虽然在课外阅读题上有一定的难度,但还是能够全面反映学生的语文学*情况。通过试卷分析,我们可以看出老师在*时的教学中突出抓基础知识的教学,对课文的讲述也到位,围绕着教材教参以及相应的材料,在备课上课方面做的较好,重视作文教学,重视复*课。所以学生试卷中基础知识的题目以及作文方面做的都比较好。

  二、出现的问题:

  古诗文默写题很多学生知道填写哪一句,但是容易写错别字;文言文知识没有彻底识记,不该扣分的还是丢分了;课外阅读部分做得不是很好,学生不懂不同题型的解题方法,下笔没有方向;名著缺背漏背的也不少;综合性学*得分不高,学生练*过少;作文存在审题不清、选材陈旧、观察力不强,知识面狭窄,书写潦草等问题。

  三、具体改进方案:

  1、积累运用部分(古诗填空以及文言文)

  (1)强化文学基础常识的训练;

  (2)定期组织“古诗文默写”“文言实词译句”等练*,强化基础的巩固。

  2、阅读部分:针对学生的特点和年级的特点,准备每周“课外阅读一测验”,强化考试两种文体的阅读。

  3、名著部分:强调学生课外阅读名著,并以考试的形式来检测学生的阅读进程以及阅读效果,以督促学生真读书。。

  4、综合性:强化题型的训练,注重解题方法,做到对症下药。

  5、作文:

  (1)强化学生作文的修改环节,让学生在修改作文中学会写好作文。

  (2)优秀文章示范。每周抽出30分钟(占用自*课)教师或学生推荐一篇适合阅读、有助于写作的好文章,字数在千字左右,注明推荐原因,查找文章赏析等与学生评议,促使学生积累好词妙句、写作方法,拓展学生写作素材。

  (3)每周一练,小到片段写作,大到周记或者日记,培养学生的写作水*。

  一、试题评价

  1、试题结构:

  全卷包括文言阅读、现代文阅读、写作三大部分,共26道小题,

  2、试题特点:

  (1)文言文阅读主要考查默写、词语解释、句子翻译及内容理解,也适应中考改革方向,增加了对虚词的考查。但在题型设置上还不全面,如:对课内要求背诵的诗文可作理解性默写的考查;对虚词可设置选择题进行考查。

  (2)注重对现代文阅读能力的考查,与中考接轨,两个语段都是课外的,分别为记叙文和,注重考查学生对语言材料的整体把握,准确提炼和表达的能力。

  (3)试题注重考察学生的知识运用和掌握,题干表述严密。

  二、试卷分析

  总得分情况:

  分数段129-120119-110109-10099-9089-8079-7069-60

  人数30504012531

  比例21.28%35.46%28.37%8.51%3.55%2.13%0.07%

  问题分析:

  从总体看,一是考生试卷*均分仅为109.70,得分率仅为73.13%,二是高分少。三、优良率21.28%;及格率94.33%。

  具体来说:

  第一部分文言文阅读

  课内文言得分率:83.71%;课外文言得分率:61.42%;文言文常见实词和默写掌握得不错,翻译句子还有待规范。

  第二部分现代文阅读

  议论文语段得分率:80%;记叙文语段得分率:63.65%;表述不严密,不能捕捉关键信息;整体把握能力、概括能力还要训练。

  第三部分作文

  作文得分率:71.92%;缺乏真情实感;选材无新意;缺乏点题意识,中心不明;书写潦草,标点不准确。

  三、改进措施

  我们三位备课组老师对这届学生应该说是非常了解,*行班几乎都有半数的学困生,另外还有为数不少的懒惰学生,这些都是我们教学落实的.巨大障碍,针对这一生源的现实,我们采取了一些措施。根据09年中考的考查内容,我们备课组从文言文、现代文和作文这三方面拟订计划:

  1、利用假期和放学补课时间进行现代文阅读的复*训练,现代文阅读的基本阅读方法、基本题型和答题规范作好人人落实关;

  2、对九年级的教材作全面分析调整,准确把握教材的,的积累和运用今年要单独列块,根据考查内容作系统全面的复*。

  3、尽量将中考文言文篇目提前上,以确保学生(特别是学有困难的学生)能有足够的时间进行落实,对文言文及古诗的默写采取人人过关检测;文言诗文按考试要求分级落实,(要考阅读理解的、要考理解性默写的、只考读读背背的、考句子翻译的、考诗词理解的)

  4、对学生的作文采取面批,指导学生修改,做好初三级阶段学生作文积累。对学困生采取把一件事记清楚确保基本达标。

  为了更好的为学生减负增效,我们备课组老师自己多下功夫,多做研究,如:

  1、文言文教学在已有的较为扎实的基础上,进一步做好归纳工作,对中考考试篇目从重点字词、句子翻译到阅读理解都作了详尽的归纳整理,有利于更好的落实。

  2、对现代文教学分三种文体注重整体理解,多交给学生一些阅读技巧,让学生掌握阅读方法的基础上进行阅读训练,引导学生多读、多思考、多感悟。

  3、加强写作教学的研究,从作文题的选择、积累方法的运用到面批落实方面都作精心的准备。

  总结:面临着中考有些同学乱了分寸,不知如何着手做试卷了。简单分析试卷类型来提高分数,何乐而不为呢?

  本次语文试卷由三部分组成:积累与应用;阅读本事;写作本事,基本与中考题型相吻合,现就这份考卷及本校学生完成情景作一些分析。

  一、考卷与考情分析

  本次试卷兼顾基础与提高,试卷由积累与应用;阅读本事;写作本事,三大板块组成,分别考查学生知识积累本事、语言应用本事、体验感悟本事。同时这份试卷充分体现了知识与技能、过程与方法、情感态度与价值观的三维原则,异常注重语言的实际运用。

  就从学生答题情景看,积累与应用部分学生完成得较好,阅读本事部分其次,写作本事完成得较差。这能够看出学生对于语文基础积累还是较为重视,阅读部分学生主要问题是回答不准确,语言表达较不规范。如第16题大部分学生是回答了“难受”字样,却没有回答“无可奈何”这个感受。写作本事部分完成较差,有一部分学生只是仓促作答,甚至有一部分学生作文没有完成,这可能是与学生*时动笔较少、阅读面不广和考试时间较短有关。很多学生都不能构思十分完整,逻辑条理十分严密清晰地完成作文,表达中心,只是天马行空地想到哪,写到哪,文章文采也较差。

  二.反思与经验总结

  1、狠抓语文只是积累,让学生每一天多读、多想、多记,日积月累积少成多。

  2、要充分利用教材培养学生阅读本事。教材中的课文都是文质兼美的妙文!如果在教学中注重课文例文作用,那么学生就会根据这些例子的字范作用去举一反三、触类旁通。突破学生阅读障碍瓶颈。同时培养学生的语言表达本事和整体感知本事,并告诉学生杂把握文章主旨基础上联系实际分析。

  3、文言文教学要强化。要借助工具书和课下注释整体感知文章大意,让学生主动认知文言实词、虚词,引导学生诵读诗文,使学生构成良好的文言语感。

  4、作文教要定格。今后作文教学要引导学生写定格作文,按照三大文体有条不紊的训练,作文既要在“通顺”上下功夫,也要在文体上强调,杜绝学生写“三不像”的文章,可根据每册每单元训练重点,进行作文训练。还要培养学生“有感而发”、“文为心声”的作文*惯,写自我真正感受过的资料,必须会更加得心应手。

  三、下阶段措施:

  1、整顿纪律,扫歪风,扶正气,让学生养成勤奋、踏实的作风。

  2、借中考引发学生的危机意识,促发他们的学*主动性。


届高三语文试卷 (菁华3篇)(扩展8)

——高三语文作文 (菁华6篇)

  喧闹的屋子里,大人们正在热切的谈论着什么,他们一边谈着一边笑着,还有人毫不顾忌地吞云吐雾。我实在受不了,就打算出门散散心。

  青石板路,缓缓延伸,我沿着一条静谧的小河缓缓向前走去。

  突然,我的视线被一片茂密的竹林吸引,碧绿的竹子仿佛将此地围城了一片绿色的海洋。竹子高耸入云,给我带来一种缥缈的感觉。我抑制不住内心的好奇,缓缓抬脚,向前迈进。

  “汪汪……”一阵狗吠声惊得我停下了脚,我揉了揉眼睛发现前方有一处小小的房屋还有一只大狗。我有些许害怕,不敢前进。这时,从屋内出来了一个老爷爷,他安抚着那条狗,一边让我过去。

  许是被他慈祥的笑容打动了吧,我坐到了他的小屋中与他聊了回天。

  老爷爷告诉我,自己原本是个商人,一直在名利场上追逐着权势与财富,后来当妻子突然离世之后,他开始真正地看清自己的本心,开始认识到钱财权势一切都只是一场空,唯有真正地拥有自己才是真。

  然后他就毅然放弃了自己原本蒸蒸日上的事业,来到此处,种了一片竹林,在竹林中搭了一座小屋。当然,还带了那条和他妻子曾经一起抚养过的狗。

  说完之后,老爷爷像个顽皮的小孩子一样呵呵一笑,又转身从厨房里端来了两杯茶。给了我一杯,又继续说:“这茶是我妻子在世时最喜欢喝的,现在她不在了,我就多带了几包来,每次想她时,就喝上一杯。你尝尝,看怎么样?”

  轻轻摇动着,一股茶香扑面而来,从这茶香中,仿佛能看见老爷爷自己一直坚守着的本心。轻轻一抿,茶香蔓延于心中,一股淡淡的清苦弥漫于舌间。

  轻轻闭上眼,细细感受这清苦。是啊,老爷爷放弃了权势与财富,只愿去坚守那真正的自己,这过程虽然有一丝清苦,但我相信,一定会回味无穷。

  喝完了老爷爷的茶,他送我到了竹林外,又是来时的青石板路,依旧沿着河流缓缓前进,只是心境发生了变化。来时心中被争权夺利的喧闹充斥着,然而现在我知道了坚持自己,勿忘本心,要去坚守那真正的自己。我想大声地对心中的自己说:“前方的路虽然坎坷,但我一定会努力坚守真正的自己,坚守本心!”

  高尔基说过:“书是人类进步的阶梯。”对于我来说,那本书开启了我的启蒙教育。

  在我童年的时候,由于身体原因,我未能和同龄孩子一样顺利进入幼儿园。这注定我将于别人不一样,妈妈经常带我去我妹妹家玩,已记不清是哪一次,我去妹妹家玩时,正巧我妹妹在看一本书,名叫《幼儿画报》。我便好奇地问她在看什么。她回答道:“《幼儿画报》呀!可好看了!不如我们一起看吧!我家里有很多,我去给你拿一本!”她很快从书房拿出了一本。我顿时被书里五彩缤纷的颜色给吸引了,便拿起书翻阅起来。一面,两面……很快一本书被我翻完了。妹妹惊奇不已:“你就看完了?”我摇摇头,将手一摊,无奈地说:“我没有看字,我只是把上面的图画看了一遍,因为我不认识几个字。”“不认识可以学嘛!我可以教你!我们一起看一本吧!”于是,我们一边看,妹妹一边读文字,我一边听。很快,我发现书中的故事比图画更有趣。离开的时候,我带了几本她已经看过的《幼儿画报》,打算和妈妈一起读。

  “妈妈,我们一起看《幼儿画报》吧!”我兴奋地对妈妈说。妈妈自然同意了。可是,几本很快便读完了。通过去向妹妹家拿这种方法早已满足不了我对书的渴望。妈妈便订了《幼儿画报》这本杂志。这本杂志是月刊,于是每个月去报刊亭领杂志便成了我兴的事。

  这本杂志除了本身的《幼儿画报》外,还配送了《时尚好妈咪》和光盘,还有一些小玩具。妈妈通过《时尚好妈咪》改进了对我的教育方法,我看不懂的故事就通过光盘来看,然后再猜这个字,并问妈妈是否正确。很快,我的识字量飞速提升。

  一两年后,《幼儿画报》举办了一个活动:写一封信给*,并以赠送一期《幼儿画报》特刊作为奖励。当时,我兴奋不已,便让妈妈买来信纸、信封、邮票,自己一笔一画写下了我人生的的第一封信,并郑重地将信塞进信封。当时的我真以为*会看我的信呢!最终,我也如愿以偿地收到了特刊。虽然貌似买信纸加上信封和邮票的费用比书的标价还要贵,但这是我学会了很多字的有力证明啊!

  现在,我已不再看《幼儿画报》了,只是因为那本杂志,我受益终身。于是,我将所有的《幼儿画报》和光盘送给了小学同学的妹妹,希望她也能因此受益。

  许嘉璐欠了学生半堂课

  许嘉璐是当今*的“训诂学”大师,虽然这门学科很冷僻,也很枯燥,但由于当时他在北京师范大学任教期间治学严谨,一个学期选修这门课的学生竟达200多人,以讲《论语》出名的北师大才女于丹也是其中之一。

  谈起恩师许嘉璐,于丹说了记忆犹新的一件事。许老师上课都提前到教室,然后站在那里一讲就是50分钟。只有一次例外,他进教室后表情很凝重,讲了20多分钟后说:“实在对不起,同学们,今天我不能讲下去了。因为陆达松先生病危,他是我上大学时得益最多的老师,也是章太炎的再传弟子,我必须马上赶过去。

  这种情况下,我站在这里也魂不守舍,讲不下去了,只好欠同学们半堂课了,以后我会找机会补上的。”说完,许老师就走了。于丹说,许老师走后,同学们都哭了。大家都能理解许老师的心情,他本来就不愿耽误我们的课,所以没有调课,坚持要给我们上课,然而,他对老师的感情又使他难以坚持。

  *充满好奇心的童年

  波兰伟大的天文学家*,小时候就非常喜欢问问题。他对世界充满了好奇,经常缠着爸爸妈妈问这问那:太阳为什么总是从东边升起,从西边落下?晴朗的夜空有那么多星星,为什么到了白天却无影无踪?小鸡为什么从鸡蛋里出来,而不从母鸡的肚子里出来……*后来对科学奥秘的不懈探求,正是从这些稀奇古怪的“为什么”开始的。

  梵高

  梵高生前曾有一个心愿:“总有一天我会找到一家咖啡馆展出我自己的作品。”但是生前,就连这么一点点卑微的梦想,最终也还是化为泡影。然而今天,梵高的出生地荷兰和梵高的安息地法国,争相把梵高认作自己的国民,争相为他建造精美的美术馆。在巴黎的奥塞博物馆、在伦敦的国家美术馆、在荷兰的梵高美术馆……他的作品永远被摆放在最显著的位置。在他的作品前,永远是人头攒动,人们向他的作品投出虔诚的眼神,啧啧赞叹,表现出对他超常的崇敬和膜拜。

  素材运用角度:价值、时间会沉淀出真正伟大的东西

  三毛

  三毛的好,一半在文字,一半在她独特壮阔的生活方式。她满足了我们对自身生活的幻想——从撒哈拉沙漠的生活,到和荷西的爱情。后来,我自己去过沙漠,我才发现在沙漠生活很大的组成部分是寂寥和恐惧。我看《温柔的夜》里三毛的日记,发现她和荷西的爱情也充满了关于金钱物质琐碎的争吵。

  看肖全给三毛拍的照片——她直面镜头,苍白惨淡,不带一丝笑容。也许那才是她真实的样子。后来三毛自杀,我非常冒昧诛心地猜测,大概也是因为太多读者把对生活的美好幻想投射到她身上,她被我们绑架,太沉,太重了。

  素材运用角度:*常心对待别人,对待自己;没有人是完美的

  每个人都得有自己的朋友圈,社交圈。你的人际关系越好,你的生活就越有趣,越不乏活力。所以说“友谊搭建生活”。而尊重,便是能使一个人的人际关系树更健康地成长的营养。

  “尊重”一词的意义就是以*等的态度去对待每个人。想想看:当别人对一件事持有自己的看法,尊重他们的看法便使一件事情拥有了多方面,多角度的看法。构成这世界的多姿多样的文化的因素也是建立在尊重之上的。而且是尊重,使我们拥有了更多朋友。当

  我们尊重了别人,别人也就开始尊重我们。互相的尊重,于是建立起最基本的友谊。为日常人们熟悉的友谊,其亦源于此。

  许多人知道尊重他人,却不付诸行动。将白糖融入咖啡里必然更加有味,同样,将尊重融入生活也必然使其更不乏趣味。只可惜有些人可怜到这种程度:知道咖啡拌白糖更可口居然还在”苦的爱抚“中艰难地喝。我相信大家一致地认为这种人是傻子,也只可惜他们不会反思自己:我有具备这种可笑的”精神“吗?有,那么请改变改变自己吧。学会尊重,于是就成了威风凛凛的.大将军,掌控了生活之基本策略。

  尊重体现在生活的每个细节中。简单来说,你每天在学校上课前对老师行礼便是尊重教师的一种表现。当别人有了属于自己的作品或物品时,你不去破坏,不去对其做一些令别人不爽的作为就是尊重他人财产,劳动的体现。别人有生理缺陷,忽略不计;别人对你有所关爱,不忘回报;别人有自己的观念意识,不妨接受;别人在自己不擅长的领域十分在行,反对嫉妒。如果对十分优秀的人产生嫉妒以至于“恨”的心理,那么你就要好好地反思一下人品了。并不是所有人都把尊重做的很完美,甚至对那些十分优秀的人产生”恨“的情绪的人也历历在目。总之,你要学会尊重,就首先要从自己生活中的种种细节下手,一步一步地反思自己,再一步一步地改正,这样提高你的素质道德,然后充实人生。

  当这世界都做到了尊重,于是人们互相尊重包容,最终,温暖会照亮这世界的每一个角落;这世界的每一份子,都会尝到尊重的甜蜜;于是,许许多多的战争,恶人就不会有了,就可以避免了啊!

  我们就像是人生轨道上的列车,不期而遇,停留了一会儿又匆匆离别。回首相望,-不能忘怀的就是你——我的同桌。

  同桌两年,就在那一瞬间分别了。离别时,猛然觉得,相聚的时光是那么短暂。柔和的春风里,我们曾经携手在同一片蓝天下尽情嬉戏打闹;而在这潇潇凉风中,又要依依不舍地挥手相别。别离的泪水飘零我们的衣襟,也润湿了我们空空的心底,但你还是握着我的手,笑一笑说:“让友情永驻心中,天涯若比邻。来日方长,日后再聚!”

  多少个风霜雪月,你的音容笑貌时常浮现在我的眼前,翩然于我的梦乡。物是人非事事休,欲语泪先流……也许我不该叹息什么。不求天长地久,但求曾经拥有,——“海内存知己,天涯若比邻”嘛。我们还拥有无数个明天呀,期待着明天的重逢,期待着欢聚一堂,畅所欲言,欢歌笑语,让心灵获得舒展和满足。日复一日,让这种美好的期待丰富我们的人生。是的,我们的友谊之花,会因思念而盛开得绚丽多彩。

  曾记否,你把仅有的一颗糖非要塞给我吃,我不吃,你就猛得把包开的糖块儿硬是塞进我嘴里,而后“咯咯咯”笑着跑开;曾记否,我因成绩的猛然“下跌”而黯然神伤,你就顽皮地一揪我的耳朵,说:“你想把‘成功’占为己有怎么可能?失败一次会使你更加清醒!”结果逗得我开怀一笑。曾记否,那一次我受了委屈就伏在你肩头哭,你呢,慷慨地陪着我泪落纷纷,哭过了,你长舒一口气:“哇,哭过的感觉真好!云开雾散,阳光灿烂。”又惹得我喜笑颜开。曾记否,那一次上课我开小差儿,给你画了一张“速描”,面目有点儿惨不忍睹,上书“傻丫头---”,结果被老师发现了,你就帮我求情:“老师,都怪我模样太引人注目,太对不住观众。您就原谅她这一次,好吗?我保证以后……”诱得老师一脸的灿烂,我总算逃过一劫。

  还记得我们同唱的《同一首歌》吗?就让我们在歌声中寻一丝慰藉,让离别之情沉淀为更深厚的友情,永驻心中,抚慰心灵。

  拥有友情的人生,真好!

  那一天,我不得已上路,为自尊地生存,为自我的证明……”

  当这首歌回荡在耳畔,它如船桨一样在我的心海中激起阵阵涟漪。透过空蒙的海面,我看到张瑞敏的顽强,看到牛根生的执著,看到马云的从容与镇定。

  走在路上,曾经我不解“常将有日思无日,莫待无时思有时”,不解“水到穷处待云起,危崖旁侧觅坦途”,不解“卧薪尝胆,三千越甲可吞吴”。那时的我,过多地贪恋身边的“风景”,在不知不觉中滑向抛物线的减区间,直到坠地的那一刻,我才明白自己已偏离轨迹许久。

  托起疲惫的身躯我又一次走在路上,这一次与成熟携手,与坚定同行。

  走在路上,真的需要坚定。

  泰戈尔说:“只有经过地狱般的磨炼,才能炼出创造天堂的力量,只有流过血的手指,才能弹出世间的绝唱。”我不需去下地狱,却要爱烈火中百炼成金,去获得创造天堂的力量,流泪流汗,去完成一次壮阔的远航!而这一切,需要有坚定的信念作翅膀,坚定的勇气做后盾。

  走在路上,我一路感受坚定。

  是谁在高唱“人到万难需放胆,事当两可要*心”?那是张大千。他告诉我:遇到困难要有坚定的意志,有胆量去同困难作斗争,遇到左右为难的事情,要静下心来认真权衡利弊,以坚定的信念做最明智的选择。

  是谁在唱“为人树起脊梁铁,把卷撑开眼海银”?那是谭嗣同,他告诉我:为人处事要有原则,以坚定的目标走自己的路, 立志做自己的主人,不做奴才。

  是谁在唱“心有三爱,奇书骏马佳山水,青松翠竹白梅兰”?他告诉我:做人要像松竹那样坚定有节,根深深扎入大地,渴饮甘泉,未出土时便有节,枝摇星月,叶拍苍天,直到凌云高处,仍然虚心。

  走在路上,坚定的信念在左,坚定的意志在右,坚定如种子,我走在路上手握坚定,时时播种,时时开花!

  坚定,永恒的主题!

  走在路上,坚定为我开路!


届高三语文试卷 (菁华3篇)(扩展9)

——初三语文试卷质量分析(精选五篇)

  本次语文试卷由三部分组成:积累与应用;阅读本事;写作本事,基本与中考题型相吻合,现就这份考卷及本校学生完成情景作一些分析。

  一、考卷与考情分析

  本次试卷兼顾基础与提高,试卷由积累与应用;阅读本事;写作本事,三大板块组成,分别考查学生知识积累本事、语言应用本事、体验感悟本事。同时这份试卷充分体现了知识与技能、过程与方法、情感态度与价值观的三维原则,异常注重语言的实际运用。

  就从学生答题情景看,积累与应用部分学生完成得较好,阅读本事部分其次,写作本事完成得较差。这能够看出学生对于语文基础积累还是较为重视,阅读部分学生主要问题是回答不准确,语言表达较不规范。如第16题大部分学生是回答了“难受”字样,却没有回答“无可奈何”这个感受。写作本事部分完成较差,有一部分学生只是仓促作答,甚至有一部分学生作文没有完成,这可能是与学生*时动笔较少、阅读面不广和考试时间较短有关。很多学生都不能构思十分完整,逻辑条理十分严密清晰地完成作文,表达中心,只是天马行空地想到哪,写到哪,文章文采也较差。

  二.反思与经验总结

  1、狠抓语文只是积累,让学生每一天多读、多想、多记,日积月累积少成多。

  2、要充分利用教材培养学生阅读本事。教材中的课文都是文质兼美的妙文!如果在教学中注重课文例文作用,那么学生就会根据这些例子的字范作用去举一反三、触类旁通。突破学生阅读障碍瓶颈。同时培养学生的语言表达本事和整体感知本事,并告诉学生杂把握文章主旨基础上联系实际分析。

  3、文言文教学要强化。要借助工具书和课下注释整体感知文章大意,让学生主动认知文言实词、虚词,引导学生诵读诗文,使学生构成良好的文言语感。

  4、作文教要定格。今后作文教学要引导学生写定格作文,按照三大文体有条不紊的训练,作文既要在“通顺”上下功夫,也要在文体上强调,杜绝学生写“三不像”的文章,可根据每册每单元训练重点,进行作文训练。还要培养学生“有感而发”、“文为心声”的作文*惯,写自我真正感受过的资料,必须会更加得心应手。

  三、下阶段措施:

  1、整顿纪律,扫歪风,扶正气,让学生养成勤奋、踏实的作风。

  2、借中考引发学生的危机意识,促发他们的学*主动性。

  3、加强对临界生、上层学生进行指导和督促,对后层生加大力度监控,对中层学生也及时做好沟通了解的工作。

  4、不断强化高考意识,增强危机感。

  期中考试已落下帷幕,本次考试为泰州市模拟考试,我们东校区语文成绩在全姜堰市排名第22,在张甸区排名第2,应该说是稳中有进。本学期以来,我校大力倡导新课程改革,采用先学后教、当堂训练的模式实施课堂有效教学,坚持每日一练,每练必改,每改必讲。从目前考试情况来看,应该说初见成效,当然在本次期中考试中还是暴露了不少的问题,具体分析如下。

  一、试题分析

  该试题能体现出中考命题思路,兼顾基础和能力的考察,比较客观地检验了学生的语文知识水*。选题紧扣教材,面向全体学生,涵盖教材要求,并在考查基础知识的同时,着重加强对能力的考查,符合初三年级学生期中检查的要求,试卷结构合理,难度适中。既考查课内基础知识,也注意课外的拓展延伸,利于培养学生的能力。

  二、考试中反映出的主要问题

  这次考试全市集中阅卷,而且是首次网上阅卷,从阅卷情况看,暴露的问题比预料的还要多,总均分也比预想的低。主要问题如下:

  1、语基训练不到位,学生基础不牢。我们在这学期还加强这方面训练,保证这块少丢分,结果学生对修改病句等知识点掌握还不够到位,第1题汉字拼音得分率只有20%,古诗赏析题失分较多,当然这次考的是辛弃疾的词,稍微难一点。

  2、现代文阅读能力明显偏低。现代文阅读共32分,一篇是科技小品说明文,另一篇是文化散文,大多为阅读常规题。结果是只有极少数同学能得到20分以上,得分率较低,尤其是理解词语在选文中的意思和在语境中的含义(第20题),这一题总分4分,可是我们大多数学生都得不到分,可能是我们在*时教学中未真正重视学生的阅读理解能力的培养,没有把文段的概括,有效信息的筛选和优美语句的欣赏等方面作为课堂教学的重点,应当引起高度重视。

  3、课外文言文阅读分析题,其知识点是课内知识的迁移,在于考察学生几年来课内文言知识的积累和知识迁移的能力。总的来看,较之以前还是有进步的,当然有少数学生因为基础较差,所以失分较多。本学期以来,我们初三语文备课组同仁加大了对学生课外文言文的辅导的量,以《初中文言文精品阅读》为抓手,精选典型篇目,在不耽误正常教学进度的前提下挤时间有计划地对学生进行课外文言文的训练,弥补了文言知识的匮乏,同时也培养了学生对文言词语的感悟能力。

  4、关于作文,从得分率来看,我们校区只有少数学生能得到45分以上,大部分学生得分在42分左右,极少数学生写作态度不端正,作文写不全,只能得到20分左右,从卷面看,学生在开头结尾的润色方面稍有起色,但缺乏生活素材的积累,缺乏对生活的理解和感悟。暴露出的问题是材料不鲜活,老生常谈。错字多,语言不生动,*铺直叙,人物形象干瘪,没有感染力。一些卷面不整洁,涂改处较多,标点符号的运用也不规范。

  5、因为首次实行网上阅卷,故有少数同学不适应答题卡的书写要求,把答案写错了位置,还有同学在答题卡上用箭头标示,这样即使答对了,也看不到答案,所以以后要加强规范答题的辅导。

  三、下一阶段教学措施

  1、对初三学生加强训练,研究如何上好复*课,要教给学生审题、答题等方法及注意事项,着眼点不能只放在提高答题技巧上,而应放在提高答题能力上。

  2、重视语言积累和运用的训练。积累训练要坚持不懈地抓,语言运用要重视材料探究题的训练。

  3、加强现代文阅读的训练。教给学生有效的阅读方法,重视对文学作品阅读欣赏的指导和训练,提高现代文阅读水*。

  4、文言文阅读一定要重视能力的迁移,加强课外迁移文言文的阅读训练。

  5、加强作文的应试训练,作文教学中要重视审题、立意的训练;进一步强化学生的文体意识;扩大学生的阅读面,拓宽视野,充实底蕴,丰富素材积累;强化语言运用能力的训练,指导学生如何让语言“亮”起来,切实提高学生作文水*。同时,要特别重视学生的良好书写*惯的培养,以适应中考作文阅卷的要求。

  6、加强对开放性试题的训练指导。指导学生运用正确的情感态度价值观去解答开放性试题。

  中考迫在眉睫,本备课组必将努力不止,拼搏不息,抓好尖子生和后进生力争在xxxx年中考中考出优异成绩。

  一、总体分析

  这一份模拟考卷,可以说是命题者花了一定心思,出的非常成功。考卷突出了检查学生初中三学年所掌握的知识和所具有的语文能力,重视语文基础知识的考查,突出对学生语文素养的考查,对今后的中考复*具有较强的指导性。虽然难度系数有一定提升,但符合今年中考的命题导向。本试题满分为120分,考试时间为150分钟。在参加的考生中最高分为101分,最低分为6分,其中以80----90分的居多,较往常相比确实有点难度。

  二、试卷分析

  本次试卷在结构上,主要分为以下几大块:

  (一)语文知识积累与运用。

  主要考查了汉字的书写,拼音的正确读法、病句的修改、古诗文名句默写、运用、名著积累、语言的实际运用等几个方面,很有针对性地考查学生对语文基础知识的掌握。

  本次试卷在结构上,主要分为以下几大块:

  1、汉字书写。这一题不是简简单单的规范抄写而是要求学生从多个词语中选择拼音、写法正确的一项综合性较强。

  2、病句修改注重与课本接轨,既考查了对文本的熟悉又巧妙的考察了学生的修改病句的知识。

  3、名句填空。第1----4题是机械型默写,即给出上句,要求写出下句,或给出下句,要求填出上句。难度较小,除个别的有错别字,其余得分不错。第5题和第6题是理解性填空还是比较简单的,也容易得分。最后一个是考查学生的筛选能力。有新意的是加进了对现代文名篇名段的考察,体现了中考的立足课本,以本为主的思想。

  4、名著积累。名著题不是简单的考名著知识而是将语言的运用与其结合起来,可谓一举二得。

  5、语言运用题。这道题既联系了实际,又考察了学生的语法知识,是一道综合性较强的题。学生要想选对须仔细思索。

  (二)阅读

  (三)古诗文阅读

  文言文阅读延续了传统的比较阅读,选择了《爱莲说》与《岳阳楼记》进行比较阅读,在题型的设计上也是由易到难,由课内的知识识记到课外的理解运用。特别是第7小题前面考文题后面考作者有一定的迷惑性。第10题在文言文中考察了修辞的作用,可谓巧妙。美中不足的是如能在题中加“两段主要运用了哪种修辞”,就更严密,更完美。例外忽视了文言文的阅读节奏这个知识点。

  (四)现代文阅读

  现代文阅读依然是以课外阅读为主的考查形式。这次与中考全部考课外的命题导向相一致。从题目的考查点上看,重在培养学生在整体把握文章基础上的分析概括能力以及词不离句,句不离篇;仔细推敲,深入思考的良好阅读*惯。从题目设计上看,无论从设计点上看和获取答案的途径上看都有点高、有点难。特别是记叙文和议论文的题有点偏难。如继续稳重的22题和24题,对于初中生确实有点难。议论文的16题和19题,16题概括难度较大,19题综合性太强,学生往往只从一个角度去答,这个题可删去前半部分让学生联系实际谈就可以了。说明文出的较好,紧扣山西特色,既考察阅读能力又联系生活实际。总之今次的阅读理解题有点难,学生得分也不高。其实初中学生阅读能做到在整体把握文章基础上的分析概括能力,能联系上下文去寻找答案,能运用简洁的语言概括理解文章就行。

  (五)作文

  作文形式从以前的话题作文中解脱出来,采用二选一的形式,与当下的作文考试题目类型的大致走向是一致的。不设审题障碍,淡化文体要求,出题者设置文题:第一个题《心中的风景》对于这个作文题目,主要体现了“人人有话说,人人说真话”的作文命题意图。学生有较深刻的体验,有话可说,感情也较为真挚。第二个题要求学生先读那段提示,从提示中感悟自然,感悟生活。是学生能更深层次的表达自己的内心世界,考察学生观察生活的能力,真正体现语文“从生活中来,到生活中去” 的思想。

  三、对今后教学的走向

  1、强化语文积累及运用训练,尤其强化字词、名句的默写和开放性创新题型训练,以学会运用为目的进行练*。

  2、强化综合性学*、口语交际的复*及名著阅读的拓展,以语文学*与现实生活之间的联系为主。

  3、强化阅读训练,包括现代文阅读(记叙文、说明文和议论文)和古诗文阅读。

  4、强化写作训练,进行作文储备,了解考场作文的技巧。

  选择题

  题量六道,东城与西城一样,题型上稍有创新,将关联词替换成了病句。其六种题型中,字音字形与西城海淀一样,都涉及到对新增二十个词语的考察。其他题,病句题不难,而标点、语意衔接及修辞则是非常难为各位考生了。标点所考察的书名号用法及引号与其他符号的位置关系会难倒一批考生;语意衔接考察的是上下文内容及语法结构的严格一致,语感稍差的同学都选不出来;修辞手法中所附的材料可用“不知所云”来评价,材料过于晦涩,且主题太不明显,中考不太会出这样的材料。

  填空题

  默写题考察了《过零丁洋》《诗经・蒹葭》《岳阳楼记》和《浣溪沙》(一曲新词酒一杯),四篇都是重点备考的对象,不多说。

  名著阅读考察了《孟子》,也是学生需重点注意的名著之一。其考查形式给各位考生的备考提供了一个方向,即名著的相关文学常识、涉及到名著的课文的具体内容(包括提到的重要地点、时间、人物、事件、主题)都应该成为重点记忆的对象。

  综合性学*

  第一题,整合并概括材料信息。材料形式为“表格+文字材料”,文字材料中概括两个要点并不难,表格的概括相对较难,表现在表头不够清晰,能做出的同学需有较好的逻辑思维。

  第二题,概括采访记录。思路跟2011年中考一致,基本上需概括的关键词都在各个材料的第一句话中,要概括的自己的理解也只是各个看法的整合,这使得本题很容易。

  第三题,考察学生对材料进行高度概括的能力,比前两题难,但若掌握了方法,本题也容易。仔细审题可知,标题中已知部分对应的是材料最后一段,因此学生需概括的即是一二段的内容,一段写“爱心分队”诞生,二段写“分队用爱心传递温暖”,将这两个意思概括为与标题已知部分格式一致的句子即可。

  文言文阅读

  从篇目上看,考察的是《陈涉世家》,就预测性来说,不算成功。题型则是一贯的实词解释、句子翻译与文意理解,可分析之处不多。

  现代文阅读

  文学类阅读:王蒙《新疆的歌》

  相对海淀、西城文学类阅读都选择“亲情”主题的文章,东城选取的主题比较晦涩,王蒙的《新疆的歌》,是一篇主题偏向于地域文化的抒情散文,理解上来说,会比《苏醒中的母亲》及《母亲》都难。但地域文化类的抒情散文是北京*几年中考文学类阅读的重要选文方向,如曾考过的《又临黄河岸》《西皮流水》等等,都属于偏向于文化的抒情散文。东城的选文方向应得到广大考生的注意。从题型上来说,则非常传统:

  第一题,行文思路题,要找作者三次在当地听《黑黑的眼睛》的地点和当时歌声的特点,答案都在原文中,只要能定位好原文,并不难。

  第二题,句子含义题,稍有难度。答好这道题必须紧扣两方面:一是句子与全文内容的关系,二是句子与全文主题的关系。从内容看,“她”指《黑黑的眼睛》这首歌,她与伊犁的联系有二:一是“我”与伊犁人民之间的联系由它作载体;二是她贯穿“我”在伊犁那段岁月(也即见证“我”那段岁月);从主题看,本文主要是通过作者对这首歌的向往与热爱来表现出作者对伊犁丰富深邃的民族精神的热爱。上述四个要点即是本道题的答案。

  第三题,全篇赏析题,跟西城的题类似,解题思路也是一致的。本文不多说。

  说明文阅读:“北京中轴线与北京城的格局”

  选文特点看,与西城、海淀一致,为文化类小品文。非常巧合的是,海淀与东城都关注到了北京城的中轴线,然而从题型设置看,则除材料链接题外,东城与海淀截然不同:

  第一题,梳理说明层次。出得比较难,答案都不能从原文找,需自己先梳理说明层次,再加以概括。本题型应得到学生充分重视,因为是否能梳理清楚说明层次能看出是否真正看懂了文章。这种题型是非常有价值的。

  第二题,语言准确性题。与海淀西城不同,不是问两句不能调换的理由,而是问“短语”不能删去的理由,考察的是副词性短语“有计划地”,需将本词与原文加以联系,明确副词所表示的文中含义,才能答好本题。

  第三题,材料链接题,出得不甚成功。基本上与原文关系不大,只需要仔细阅读材料,对材料加以概括即可得出答案。

  议论文阅读《匠人与大师》

  本文在2009年燕山区的期末考试中考过,东城用了文章,但换了题,题比原来难了。

  第一题:概括(提取)中心论点。在议论文中,这是一道最基础的题型,按理说应该不难。但很多学生反映找不到中心论点。这只能说明同学们对议论文的阅读方法还不甚熟悉,需要加强练*。本题答案是第六段第一句话。

  第二题:寻找使用比喻论证的句子中各喻体所对应的本体。对应原文即可,不难。

  第三题,论据作用题。定位论据,寻找它所证明的分论点即可,不难。

  作文

  东城出的是半命题作文《,给了我》。从形式看,比较让人跌眼镜,姑且算创新,而实际在*时考试中也有区县使用过这种方式――作文题居然要填前后两个空,主语跟宾语都要填上,但并不难。

  需注意的是,按照这个作文题的逻辑而言,补题时前边一空补的是主语,姑且可理解为写作对象(理解为写作材料也未尝不可),后一空补的应是跟材料相关的立意。也就是说,出题思路是让学生在补题的时即确定写作内容及文章立意,东城考生若能多花些时间在补题这一步,将题目补好,下边的写作过程将会非常顺利。

  东城的一模相对来说比较难,很多东城孩子都表示很受伤。但不管是否受伤,下一步都不是自怨自艾的时候,成绩发下后用心分析试卷,找出薄弱环节,制定更科学的复*计划,才是重中之重。同学们,祝大家复*顺利!

  反省是我们温故知新的基础,是我们学以致用的标准。

  一、试题分析:

  本套试卷的考试范围,难易适中,重点突出,考察全面,与中考题型较为吻合,虽然在课外阅读题上有一定的难度,但还是能够全面反映学生的语文学*情况。通过试卷分析,我们可以看出老师在*时的教学中突出抓基础知识的教学,对课文的讲述也到位,围绕着教材教参以及相应的材料,在备课上课方面做的较好,重视作文教学,重视复*课。所以学生试卷中基础知识的题目以及作文方面做的都比较好。

  二、出现的问题:

  古诗文默写题很多学生知道填写哪一句,但是容易写错别字;文言文知识没有彻底识记,不该扣分的还是丢分了;课外阅读部分做得不是很好,学生不懂不同题型的解题方法,下笔没有方向;名著缺背漏背的也不少;综合性学*得分不高,学生练*过少;作文存在审题不清、选材陈旧、观察力不强,知识面狭窄,书写潦草等问题。

  三、具体改进方案:

  1、积累运用部分(古诗填空以及文言文)

  (1)强化文学基础常识的训练;

  (2)定期组织“古诗文默写”“文言实词译句”等练*,强化基础的巩固。

  2、阅读部分:针对学生的特点和年级的特点,准备每周“课外阅读一测验”,强化考试两种文体的阅读。

  3、名著部分:强调学生课外阅读名著,并以考试的形式来检测学生的阅读进程以及阅读效果,以督促学生真读书。。

  4、综合性:强化题型的训练,注重解题方法,做到对症下药。

  5、作文:

  (1)强化学生作文的修改环节,让学生在修改作文中学会写好作文。

  (2)优秀文章示范。每周抽出30分钟(占用自*课)教师或学生推荐一篇适合阅读、有助于写作的好文章,字数在千字左右,注明推荐原因,查找文章赏析等与学生评议,促使学生积累好词妙句、写作方法,拓展学生写作素材。

  (3)每周一练,小到片段写作,大到周记或者日记,培养学生的写作水*。


届高三语文试卷 (菁华3篇)(扩展10)

——高三英语试卷分析实用5份

  上学期期末考试已经结束。为了及时查缺补漏,对于考试中反映出来的问题及时进行改正,总结好的经验和做法,为以后的教学找出指导性的意见,以便在接下来的教学过程中给予更好的指导,达到进一步提高学生英语水*和成绩的目的。根据本次考试的实际情况特作如下分析:

  一、成绩分析

  本次期末考试从整体上看学生的成绩还可以。二年八班共有学生27人,其中八十分以上的同学有1人,七十分以上的同学有7人,六十分以上的同学有16人。及格率为59%,优秀率为4%。

  二、学情及教情分析

  本次期末考试重点考察语文教育出版社出版的《英语》(基础模块下册),考试范围为第一单元至第八单元所要求掌握的单词、短语以及每单元后的常用语法。从学生试卷的总体作答情况来看,学生对于课本中所要求的单词、短语以及课文翻译掌握的非常好,涉及到课本内容的题目基本上都能较好地掌握。针对考察学生综合能力的完形填空及阅读理解题目,少部分学生掌握的较好能够做到学以致用,大部分学生缺乏知识的实际应用能力。

  三、题型设计及失分原因分析

  1、选择题:主要考查学生对课本知识的掌握和应用情况,试题难易程度中等,学生作答情况不是十分好。25个选择题,每题1分,

  其中最高分12分。选择题的设计应用性比较强,部分内容学生刚刚接触掌握的还不是特别牢固。另外一部分题考察单元课文中所学的单词短语及语法,学生本部分掌握的还可以,但还有部分同学掌握的不是特别好,对一些固定单词和短语的掌握程度还不够,个别单词短语记得不太牢固。

  2、完形填空题:难易程度中等。学生整体把握短文的能力还不是十分强,部分学生在作答中理解出现偏差,导致失分比较严重。另外学生对固定搭配、介词的使用及部分单词的用法掌握不够牢固,在做题应用时不能将所学知识学以致用。

  3、阅读理解题:本题较难。三篇阅读理解,学生反映最集中的问题是对文章内容理解不清。另外学生词汇量明显较少导致不能很好理解文章内容。另外部分学生做题速度有待提高。

  4、短语、句子翻译:本题比较容易。短语翻译10个,每个1分。句子翻译10题,共20分。短语翻译全部来自课文,从学生的作答情况来看绝大部分学生对所学过的单词和短语都能够熟练掌握。句子翻译因为都来自课本,因此学生掌握的也可以,但个别学生对部分单词和短语掌握的还不够熟练。

  四、反思和改进措施

  针对目前学生英语学*的现状,我们一定要注重课堂45分钟的效率,要确保每个学生都能做到对知识的巩固、灵活运用和独立解题能力的培养。结合实际,我做了具体计划,如下:

  1、加强基础训练,扩大学生的词汇量。

  在各种考试中,基础知识都是考查的重点。我们对词汇、短语、句型和语法等基础知识的教学,不能仅仅停留在让学生机械记忆阶段,在加强对单词、短语的听写和背诵的同时,应注重把词汇教学融入特定的语言环境中,强调对基础知识的理解和运用。 由于班级学生整体的英语底子比较薄,因此要重点检测基础词汇,对不同类型的单词,可以有不同的要求,避免学生负担过重,失去学*英语的兴趣。这点是学好英语的基础和根本,一定要下大力气,在课堂上抽出相应时间进行检测,强化学生的英语词汇的掌握。

  2、加强学生阅读能力的培养。

  阅读能力的提高是长期训练的结果。阅读在整个考试中所占有的比例也比较高,因此,培养学生的阅读能力对于提高学生的整体成绩是非常必要的。在下一阶段的教学中要把握好阅读课的课型,有意识地加强学生阅读能力的训练与培养,课堂上要指导学生阅读教材,课后也要指导学生多阅读内容健康,时代感、思想性、知识性、趣味性较强的短文,教会学生略读和范读等方法,通过大量阅读来培养学生的语感以及通过上下文捕捉信息的能力,拓宽学生的知识面,开阔学生的视野。

  3、面向全体学生,关注不同层次学生的学*。

  本次测试反映出学生整体水*不高,学生成绩参差不齐,部分学生还是不能对英语产生浓厚的兴趣。因此,在日后的教学过程中要学会面向全体学生,关注不同层次学生的学*状况。在我看来,设法提高每一个学生学*英语的积极性是非常重要的。一定要鼓励学生积极、主动地参与课堂教学,帮助学生找到适合自己的英语学*方法,为今后的学*和发展打下良好的基础。

  高三英语试卷评析课,是指教师对英语高考模拟试卷的评讲。在高考前的总复*阶段,试卷评析是一个重要课型。但在*时的教学过程中,对试卷评析课的研究,许多老师对此都不太重视,也不太讲究“评析”的方式。那么,在高三阶段如何上好试卷评析课呢?本人结合自己的教学实践,就英语试卷评析课中应注意的几个问题进行了初探。

  一、评析课存在的问题

  在高三阶段,试卷评析课既频繁,又重要。但许多教师要么一份试卷不分主次,从第一题讲到最后一题,题题分析,面面俱到;要么因时间紧、内容多只是去对对答案,对重难点问题不做详细评析,甚至有时对学生所完成的试卷根本不进行评析,最后导致英语课堂费时低效。也有的教师只是注重一两种题型的讲解,如单项选择题和单词拼写题,以致误导学生只重视语言知识的识记和掌握,轻视解题能力的培养和提高。

  二、解决问题的方法

  1.评析的及时性

  每次测试后,都应当及时批改试卷,并且及时讲评。最好是第一天测试,第二天分析。首先,就学生而言,考试时其思维最集中、最活跃,此时他们对于试卷所考查的知识点是非常熟悉的。其次,对教师来说,刚阅完卷,对学生存在的问题了如指掌,此时讲评必然事半功倍。

  2.评析前应做好仔细分析

  考试的目的在于了解学生的学*情况,发现问题。而考试后的试卷评析,则是一次重要的补救教学。只有经过充分的课前准备,把握学生答题情况,评析时才能有的放矢。

  (1)要分析试卷内容。对试卷的题型,知识能力的考查,试题的难度都要做到心中有数。除了对所考的知识点和难度值进行分析外,还应对各题的命题思路进行分析。尤其对于问题的设置,解题的思路更要仔细分析。

  (2)要分析学生答题情况。要具体统计哪些学生在知识记忆方面失分较多,哪些学生在运用能力题方面失分较多。只有完全了解学生的答题情况,才能有针对性地分析讲评,使学生明确自己对所学知识的掌握情况,以便在今后的学*中加以改进,以达到最佳效果。

  (3)认真备好讲评课。重点应当集中在学生出错率高,问题相对集中的题目上,还要确定用什么方法讲,讲到什么程度;确定学生出错关键及思维障碍所在,怎样才能在今后不出或少出错,措施如何。

  三、试卷评析中应注意的五个原则

  1.师生共同参与

  一堂成功的试卷讲评课,教师应该给予学生表述自己思维过程的机会,增加教师与学生、学生与学生的交流时间。在讲解错误率较高的试题时,可适当地让思路较好的学生做些示范介绍,然后由学生讨论,教师进行总结。只有师生双方共同参与,互相交流,在彼此的探索、合作学*中,才能有所感悟,有所收获。

  2.知识的拓展和归纳

  教师讲评时,应从“点”出发,把“面”带出来呈现给学生。把这一题目所考查的知识网络化、系统化,将某一类所学的知识做一个系统的梳理归纳,形成知识链,尽可能地构建知识间的广泛联系。那么,学生学到的就不仅只是题目本身,而是该题所涉及到的一系列内容,会大大提高教学效果,也使学生在相关知识的联系和区别上得到深刻的理解。

  例如,compare这个词在单项选择、完型填空及短文改错中出错率极高,讲评时,就可以把它放在不同的句式结构中,让学生尝试找出决定它不同形式的信息:

  a.If you__________these books, you will find them different.

  b.____________these books,you will find them different.

  c.____________these books, and you will find them different.

  d.____________with that book, this book is much more interesting.

  A. comparing B. compare C. compared D. to compare

  通过讨论,能够总结出a、b、c句中的主句主语均为compare的执行者“人”,而d则是compare的动作的承受者,故很容易地得出d答案为C。而a句中的有效暗示为“If”是连词,所以应选择B。b句没有任何连词,并且与d)句形成对比,就可选出答案是A。c句中的关键词是“and”,故应选择祈使句B。然后,根据其各自的特征,学生就会有一个完整的“compare”。

  3.指导解题的技巧

  “最有价值的知识是关于方法的知识”,这是古人及当今许多学者的共识。在试卷评析时,不仅让学生掌握基础知识,而且应重视学*能力、解题能力的培养,重视解题技巧的指导。因此,评析试卷的关键是把解题思路、方法、技巧教给学生,针对不同的题型特点,进行相应的解题技巧的点拨。例如,这样一道选择题:Does the way you thought of_________the water clean make any sense?

  A. make B. to make C. making D. made

  学生认为选C,因为of是介词,也有学生认为选择A,因为does开头。面对学生的困惑,宣布:“答案是B。”听到答案,学生个个都觉得不可思议。于是,要一步步引导他们,先分解句子:

  a. You thought of the way.

  b. the way to make the water clean

  c. Does the way make any sense?

  然后,把a和b合并:You thought of the way to make the water clean.最后,把这个句子与c合并成一个定语从句:Does the way (that) you thought of to make the water clean make any sense?

  此时,学生脸上都露出了满意的表情。于是,要乘胜追击,又给了学生另一道题:The day I was looking forward to at last.

  A.comingB.to come C.came D.has come

  这时,几乎所有的学生都能选出正确答案C。所以,经过层层引导,步步深入,学生进一步加深对所考知识点的印象,实现理解上的“第二次飞跃”。

  4.讲评方式多样化

  在评析试卷时,应注意讲评方式的多样化,既要有教师讲评,又要有学生参与,还要有师生共同讨论。不同的题型评析方法应当不尽相同,如单选、完形、阅读、书面表达等不同题型要区别对待,教师应有自己不同的评析方法。在讲评时,教师应做好归纳工作,如“哪些短语后to为介词,哪些动词的-ing和-ed形式可做形容词,可表达不同的含义,等等。对于学生的易错题和难理解的题,要作分析,教会方法。

  5.学生的情感体验

  在试卷评析中,不断鼓励学生,维护学生的自尊和自信是非常必要的。鼓励应贯穿试卷讲评的始终。遇有创新独到之处的学生及时肯定,并积极点燃他们智慧的火花,激发他们内在的潜能。对成绩不理想的学生,尽可能挖掘其闪光点,加以赞赏,予以鼓励。总地说来,在试卷评析中,学生和教师要实现语言知识、解题方法、情感等多方面的良性互动。

  总之,要想上好高三英语试卷评析课,教师不仅要注意评析课存在的问题,还要评析前做好分析,并坚持评析中应注意的五个原则。只有这样,才能提高试卷评析课的教学效果,才能提高复*备考的质量。因此,对于高三英语试卷评析课,是值得我们每一位高中英语教师,在今后的教学中必须继续努力探索和研究的。

  一、整体情况分析

  本次考试听力为20分,笔试为130分。本次考试的命题坚持了“贴*高考,注重语言的情景性、运用性”的试题设计原则,注重考查学生的语言基础知识积累和在语境中运用语言的能力。试卷在选材上注重题材、体裁的多样性,文章内容贴*生活、贴*实际,具有积极的情感态度导向,完全体现了天津卷英语学科高考的命题特点。命题从总体上讲有较好的区分度,能反映出学生的语言运用能力,对下一阶段高三的英语备考工作也有着正确的导向作用。

  三、具体题型分析

  1、听力部分整体得分较低,原因是听力能力差,次要信息较多,学生易受干扰。总体而言,题目较易、较浅显,听力得分率相对较高。从主要的失分题型中,说明我们在以后的教学中不仅要进一步加强听力训练和方法指导,培养学生的预测和捕捉关键信息,以及综合理解语言信息的能力,还得夯实学生基础,提高学生的语言基本功,从而提高语言理解能力。

  2、单项选择:学生对语法方面能力欠缺,其主要原因为高三学生对语法知识有些遗忘,而有些学生干脆就一直未掌握;学生尽管每天都在背词汇,但效果较差,更不能对词汇进行深层掌握。为此,在余下的两个月时间内,我们尽量加强重点词汇的检查力度,并及时归纳总结各种词汇辨析,力求少失分。

  3、完型填空:完型是考查学生在阅读理解的基础上对语篇、词汇、语法的综合运用情况。从失分率高的题目可以看出,一方面,个别学生未真正把握整个语篇含义就做题导致失分;另一方面,大部分学生在名词和动词的词义辨析和固定搭配方面掌握仍不牢固,与单选似乎有着同样的问题,今后既要强化完形答题技巧训练又要多方面搜集两大实词的辨析与用法。

  4、阅读理解:此次4篇阅读难度适中,学生在查找个别细节、目的归纳、猜测词义、推理判断题上的能力仍然较弱,原因是学生不善于利用有效的信息进行排除和合理的推断,没有掌握如何从整体上、宏观上把握文章的脉络,明确段意,理解文章中心和作者的写作意图的技巧。其它原因:1)部分学生基础薄弱,阅读速度不够,造成阅读时间不足,文章来不及细读。2)学生词汇量不够,造成理解难度增大,解题正确率低。3)分析信息,处理信息能力差,无法合理分析关键信息,提取有效信息,耗费了很多时间,结果事倍功半。4)知识面陕窄。今后应更加注重学生读的能力的训练。

  第五部分:任务型阅读

  这篇任务型阅读无论是从选材还是题目的设计上来讲都是较为成功的,能够综合考查学生阅读中寻找信息及对信息进行总结、推理以及进行相应的变化等方面的相关能力。难度一般,题目中只有个别根据关键词在文中相关位置直接找出答案的细节题,大部分都需要学生对句子理解后加以适当转换的题型。

  第六部分:书面表达

  本次书面表达内容话题与学生实际密切相关,关于心理状态和对考试的认识。但从阅卷的情况来看,书面表达中还存在以下几点问题:

  1 .要点不齐全。有点要点容易被学生遗漏或表达不清,反映第一轮复*还不够扎实。

  2.基本功不扎实,出现了很多语法错误和汉语式英语的表达方式。更不能使用一些较高级或较复杂的句法结构来提高文章的档次。

  3.很多学生没有养成写好后复读检查一遍的*惯,存在着诸如主谓一致、标点符号的低级错误。

  4.连词的使用欠缺。体现在句与句,段与段之间缺少必要的连词和高级词汇的运用,文章读起来不连贯,不流畅,条理不清晰。这也是得不到高分的原因之一。

  除此以外,还有学生字数不够(少于100字)或者不注意大小写及标点符号等小的细节,导致扣分较多。

  三、今后复*中应注意的问题

  1.加强复*以巩固所学知识。

  试卷中学生的失分,很大一部分源于他们的词汇量太小,知识体系建构不牢,不会灵活运用所学知识等问题。我们应坚持学新,在学新中复*旧知识,加大知识的输入量,则学生词汇量会渐减,知识体系就不会牢固。因此,我们可以精选一些趣味性较强的泛读文章,在整体理解文章后,挑出文章中出现的较重要又常见的词汇、词组,对其用法进行讲解,即可复*已学知识,也可适当补充拓展。或者在进行专项阅读和完型训练时,在将文章作为限时训练做完后,然后再对文中重要的词和词组的用法进行归纳总结,这样既达到训练学生阅读的目的又可复*已有的知识。

  2.狠抓词汇教学不放松。

  这张试卷的一大特点就是词汇量大,这一点在完型中体现的尤为突出。很多学生因为词汇量的不足而不能理解句子的意思,严重影响学生的阅读速度和信心,导致完形填空得分不高。所以词汇的丰富不丰富,决定着能力的高低和英语成绩的好坏。*时除了复*已学单词外,还须注意有些词的一词多义。除此以外,还须对阅读中出现频率较高的单词,让学生记在笔记本上,经常翻阅和检测。对于这些单词可以适当降低要求,只要学生认识这些词汇即可。

  3.加强写作基本功,提升书面表达能力。

  从阅卷情况看,这次作文的主要失分点除了要点不全外,还有错句较多,文中存在的大量拼写、词汇、语法、句法错误,说明学生基本功没有过关。因此应加强基础知识和基本技能的训练,为写作打好基础,要求侧重背一些连词和英语固定表达的句型,以及常用的短语和有特点的动词词组。

  高考英语写作想得高分,除了内容要点齐全外,表达正确外,还要求作文中有一定较高级的词汇和语法结构。所以一定要借鉴参考范文中好的表达方式,要坚持背诵一些优秀范文和一些语言纯正的文章,多读、多背,注重语言积累。同时要注意不同文体作文的写作格式和技巧。

  一、试卷分析

  本次试卷分听力和笔试两部分。内容选自外研版七年级下学期英语教材,试题的难易度适中,基本按照7:2:1的难易度命制试卷,不仅能考查出学生英语基本基础知识的掌握程度,更能检测出学生的综合语言运用能力。本次试卷内容全面,知识覆盖面广,涵盖了七年级下课标规定的绝大部分内容,包括话题、语法、词汇等内容,话题涉及旅游、家乡、交通、故事、过去生活等。

  本次试卷的特点有许多,许多试题源于教材,其中六篇阅读语篇有四篇源自于教材,这不仅引导学生重视教材,更有利于指导教师充分和合理使用教材;试题题型全面,像词语运用和动词填空试题有利于考查学生对词汇的灵活运用和语法的掌握,而阅读试题的设置则从不同角度考查了学生对阅读技巧的掌握,如填表格、选择、图文搭配、回答问题等。情境设置具有真实性;注重跨文化意识的熏陶;选题时代感强。

  二、成绩分析

  成绩:校*均分102.55,及格率74.14%,优秀率15.59%。

  分析:*均分低是因为考十几,二十几的学生较多,这也是造成及格率低的原因。

  三、学生作答主要问题及归因

  1、失分率高的答题有:单词辩音的23、24、25小题;单选的28、34小题;动词填空50题;完型填空的63、64、70题;阅读的74-78题、87-90题;写作失分多。

  2、失分的主要原因分析:

  1)辩音训练不够,学生缺乏规律;

  2)缺乏对语境的把握;

  3)学生答题不仔细,表格阅读技巧没掌握;回答问题没有把握文章大意和作者写作意图

  4)写作审题错误运用过去时,逻辑关系有问题,语言表达能力差,书写不规范。

  四、改进措施及建议

  1、改进教学措施及建议

  1)深入研究课标和教材,明确重点、难点和考点、

  2)加强集体备课与研讨,合作并交流。

  3)加强辩音训练,多进行朗读。

  4)指导答题技巧。

  2、提高学生成绩及建议

  1)培养学生良好的学**惯,培养学*兴趣

  2)加强基础训练,继续通过竞赛等形式促进学生夯实基础

  3)加强阅读技巧的训练

  4)加强审题能力训练,强调规范书写。

  5)分层辅导,加强监督检查。

  3、改进命题质量的建议

  研究课标和教材,制定双向细目表,预估学生答题存在的问题,提高命题质量。

  第一题 听力 一、二、三、听力部分:难度不是很大,但是初一学生训练少,很少得到满分,尤其第四部分听力填空内容,绝大多数学生听不出来。

  第二题 单项选择 绝大部分学生都全做了,但有些题学生没有接触到因此有些失分。从卷面上看,学生对语法题不是很好,而且有些语法忘记,或者根本就不懂。

  第三、四题 完形填空和阅读理解:这些题目都和学生的阅读能力和掌握的词汇量有关,*时还需在阅读方面加强训练。同时还要掌握一些阅读和做题技巧,而这些对初一来说是个难点。

相关词条